ML111870274

From kanterella
Jump to navigation Jump to search

Draft Written Exam (Folder 2)
ML111870274
Person / Time
Site: Oyster Creek
Issue date: 05/26/2011
From:
Exelon Nuclear
To: Todd Fish
Operations Branch I
Hansell S
Shared Package
ML110030666 List:
References
TAC U01831
Download: ML111870274 (299)


Text

EXAMINATION ANSWER KEY ILT 10-1 NRC RO Exam 1 ID: 10-1 NR01 Points: 1.00 The plant was at rated power when the following annunciator alarmed:

  • EMRV POWER LOST/DISABLED The Operator reports that there are NO lit indicating lights for EMRV NR108A (the bulbs were changed with tested/good bulbs with no change in indication).

Which of the following states whether EMRV NR108A is available to perform its RPV pressure protection function and its ADS function?

Available for RPY Pressyre Available for ADS?

Protection?

A. Yes Yes B. No Yes C. No No D. Yes No Answer: C Answer Explanation I QID: 10-1 NR01 Question # I 1 I Developer / Date: JJR /7-11-11 Knowledge and Ability Reference Information Importance Rating K&A RO SRO 218000 ADS K1.08 - Knowledge of the physical connectio.,s and/or cause- effect relationships 3.9 3.9 between AUTOMATIC DEPRESSURIZATION SYSTEM and the following: Safety/relief valves Level I RO J Tier 2 J Group I 1 General BR2002 ADS Lesson RAP-B5g References Sh.1 of 4 Plan ILT 10-1 NRC & ,l,UDIT EXAM Page: 1 of 218 26 May 2011

EXAMINATION ANSWER KEY ILT 10-1 NRC RO Exam C is Correct. The indications provided show a complete loss of power to EMRV NR108A. The EMRV solenoid must energize to open the EMRV. Thus, the means the power to open the EMRV, both automatically from any Signal and manually from the control switch, has been removed and the valve will stay shut in the current configuration. Position indication for the valve comes from the same power Explanation supply as that for the valve, and thus there is no power, normal or alternate, to the valve. Thus, the valve is unavailable to perform either the RPV pressure protection function or ADS function.

All distractors are Incorrect but plausible if the applicant does not recall the resultant effect of an EMRV that is disabled on the ADS system.

References to be None provided durintJ exam:

Lesson ptlan 2621.828.0005, Automatic Depressurization System Learning ADS-368, Describe the EMRV initiation logic for Objectivel both overpressure operation and operation in the ADS mode.

Question Source (New, Modified, Bank) I Bank If Bank or Modified:

VISION S)rstem/Question 10 666142 Question :Source ADS-513 Memory or Fundamental X Comprehension Cognitive 1 :1 or Analysis Knowledge Level NUREG 1021 Appendix B: Interlocks, setpoints, or system (singular) response 10CRF55 55.41 (b) I 3 I 55.43(b) I Content Mechanical components and design features of the reactor primary system.

Justificati*on for LORT questions N/A with KIA values < 3.0 Time to Complete: 1-2 minutes I Point Value: 1 System 10 No.: I 218000 I PRA: I NO ILT 10-1 NRC & AUDIT EXAM Page: 2 of 218 26 May 2011

EXAMINATION ANSWER KEY ILT 10-1 NRC RO Exam Safety 3

r8J Initial License Level Functic>n: D LORT ILT 10-1 NRC & AUDIT EXAM Page: 3 of 218 26 May 2011

EXAMINATION ANSWER KEY ILT 10-1 NRC RO Exam 2 ID: 10-1 NR02 Points: 1.00 The plant is in COLD SHUTDOWN with the Shutdown Cooling (SOC) System in service. Plant conditions include the following:

  • SDC Loop A is in service
  • SDC Loops Band C are secured
  • All RECIRC PUMP SUCTION TEMPS indicate 188°F and rising
  • USS 1B2 is de-energized due to maintenance lAW 305, Shutdown Cooling System Operation, what operator action is required to LOWER RPV temperature?

A. Place the B or C SOC Loop in service at the discresion of the Unit Supervisor.

B. Throttle OPEN V-5-107, IAI SO CLG CCW INLET, to allow more RBCCW flow through the SDC A heat exchanger.

C. Throttle OPEN V-5-106, SO CLG CCW OUTLET, to allow more RBCCW flow through the SDC A heat exchangers.

O. Throttle OPEN V-17-55, SHUTDOWN COOLING A DISCHARGE, to allow more SDC flow through the SOC A heat exchanger.

Answer: C Answer Explanation I QID: 10-1 NR02 Question # I 2 I Developer 1 Date: JJR 17-11-11 Knowledge and Ability Reference Information Importance Rating K&A RO SRO 205000 Shutdown Cooling K1.05 - Knowledge of the physical connections andlor cause- effect relationships 3.1 3.1 between SHUTDOWN COOLING SYSTEM (RHR SHUTDOWN COOLING MODE) and the following: Component cooling water systems Level I RO I Tier 2 I Group I 1 General BR3002 305 References Sh. 20f4 ILT10-1 NRC & AUDIT EXAM Page: 4 of 218 26 May 2011

EXAMINATION ANSWER KEY ILT 10-1 NRC RO Exam C is Correct. The question stem provides a condition where the SOC System is cooling down the RPV with the A SOC Loop. The stem also states USS 1B2 is de-energized, which powers SOC Pumps B &

C, therefore SOC loops B & C are unavailable for cooldown. Procedure 305, SOC System Operation, requires the operator control cooldown rate by throttling RBCCW flow via V-5-106, SO CLG CCW OUTLET. In order to LOWER RPV temperature, V-5 106 must be throttled OPEN.

Explanation A is Incorrect. This distractor is plausible if the applicant does not recall that SOC Pumps would be unavailable due to a fault on USS 1B2.

B is Incorrect. This distractor is plausible if the applicant does not recall that the outlet to the heat exchanger is throttled, not inlet.

o is Incorrect. This distractor is plausible if the applicant does not recall that cooldown rate is controlled by throttling the CCW flow, not SOC flow.

References to be None provided durintl exam:

Lesson Plan 2621.828.0.0045, Shutdown Cooling System Learning SOC-10453, Explain or describe how this system is Objective/ interrelated with other plant systems.

Question Source (New, Modified, Bank) I New If Bank or Modified: N/A VISION System/Question 10 Question Source Memory or X Comprehension Fundamental 2:RI or Analysis Cognitive Knowledge Level NUREG 1021 Appendix B: Recognizing Interaction between systems (plural), including consequences and implications 10CRF55 55.41 I 10 I 55.43 I Content Administrative, normal, abnormal, and emergency operating procedures for the facility.

ILT 10-1 NRC & AUDIT EXAM Page: 5 of 218 26 May 2011

EXAMINATION ANSWER KEY ILT 10-1 NRC RO Exam Justification for LORT questions with N/A KIA values < 3.0 Time to Complete: 1-2 minutes I Point Value: 1 System ID No.: 205000 PRA: I NO Safety ~ Initial License Level 4

Function: o LORT ILT 1().1 NRC & AUDIT EXAM Page: 6 of 218 26 May 2011

EXAMINATION ANSWER KEY ILT 10-1 NRC RO Exam 3 10: 10-1 NR03 Points: 1.00 The plant was at rated power when the the BOP notices the OPEN and CLOSED indications forV-5-166, CCW OUTLET ISOLATION. are extinguished (assume both lightbulbs are working as designed).

A loss of which ONE of the following power supplies would cause the indications observed on V-5-166?

A. DCB B. USS 1A3 C. MCC 1B21B D. 4160V Bus 1A Answer: C Answer Explanation ,

QID: 10-1 NR03 Question # I 3 I Developer 1 Date: JJR 17-11-11 Knowledjle and Ability Reference Information Importance Rating K&A RO SRO 400000 Component Cooling Water K2.02

  • Knowledge of electrical power 2.9 2.9 supplies to the following: CCW valves Level I RO I Tier 2 I Group I 1 General BR 3004 References Sh. 4 of6 C is Correct. The question stem is stating that v-s 166 has lost valve indication. The power supply for this CCW (RBCCW) valve indication is MCC 1B21 B.

A loss of MCC 1B21 B will cause a loss of indication Explanatioln to V-S-166.

A, B, and D are Incorrect. This distractor is plausible if the applicant does not recall the power supply to V-S-166.

References to be None provided during exam:

ILT 10-1 NRC & A,UDIT EXAM Page: 7 of 218 26 May 2011

EXAMINATION ANSWER KEY ILT 10-1 NRC RO Exam Lesson Plan 2621.828.0.0035, RBCCW System Learning RBC-CT1, Demonstrate satisfactory knowledge of Objectivel the RBCCW System.

Question Source (New, Modified, Bank) I New If Bank or Modified: N/A VISION System/Question 10 Question Source Memory or X Comprehension Cognitive Fundamental 1:F or Analysis Level Knowledge NUREG 1021 ApP4tndix B: facts 55.41 I 7 I 55.43 I 10CRF55 Design, components, and functions of control and safety Content systems, including instrumentation, signals, interlocks, failure modesJ and automatic and manual features.

Justificati~on for LORT questions with N/A KIA values < 3.0 Time to Complete: 1*2 minutes I Point Value: 1 System IDI No.: 400000 PRA: I NO Safety [gJ Initial License Level 8

Function: o LORT ILT 10-1 NRC & AUDIT EXAM Page: 80f218 26 May 2011

EXAMINATION ANSWER KEY ILT 10-1 NRC RO Exam 4 ID: 10-1 NR04 Points: 1.00 The plant was at rated power. The 125 VDC Distribution System is in a normal lineup.

An event then occurred and the following annunciators came into alarm:

  • BUS AlB UV
  • 1B1 DC LOST Which of the following loads IS affected by this event?

A. Remote Shutdown Panel B. Containment Spray System 1 C. Isolation Valve Motor Control Center DC-1 D. Main Generator Field Excitation Switchgear Answer: D Answer Explanation I QID: 10-1 NR04 Question # I 4 I Developer 1Date: JJR 17-11-11 Knowledge and Ability Reference Information Importance Rating K&A RO SRO 263000 DC Electrical Distribution K2.01 - Knowledge of electrical power 3.1 3.4 supplies to the following: Major D.C. loads Level I RO I Tier 2 I Group I 1 General BR3028 References Sh.10f2 ILT 10-1 NRC & AUDIT EXAM Page: 9 of 218 26 May 2011

EXAMINATION ANSWER KEY ILT 10-1 NRC RO Exam D is Correct. The question stem provides indications (annunciators) for a loss of DC-A. Due to the combined annunciator alarm of BUS AlB UV, the applicant must analyze the other annunciators received and determine there is a loss of DC-A, not DC-B. The MG Field Excitation swithgear is powered Explanation from DC-A.

A, B, and D are Incorrect. These distractors are plausible if the applicant does not recall major loads powered from DC-A. The RSP is powered from DC D, DC-1 from DC-B, and Containment Spray System 1 Panel ER8A from DC-F. All still have power and are unaffected.

References to be None provided durin ~ exam:

Lesson fltlan 2621.828.0.0012, DC Distribution Learning DCD-1106, Draw a one-line diagram of the 125V DC Objective/ Dist. system including: Major Buses (A, B, and C Battery Systems), Battery charging power supplies, Major Breakers, Automatic bus transfer switches, Manual bus transfer switches, and Major loads for each DC panel.

Question Source (New, Modified, Bank) I Modified If Bank or Modified:

VISION System/Question ID 505782 Question Source DCD-06 Memory or X Comprehension Fundamental 3:SPK Cognitive or Analysis Knowledge Level NUREG 1021 Appendix B: Solve a eroblem using Knowledge and its meaning 55.41 I 7 I 55.43 I 10CRF55 Design, components, and functions of control and Content safety systems, including instrumentation, signals, interlocks, failure modes, and automatic and manual features.

Justification for LORT que!stions with N/A KIA values < 3.0 Time to Clomplete: 1-2 minutes I Point Value: 1 System 1[' No.: I 263000 I PRA: I NO ILT 10-1 NRC & AUDIT EXAM Page: 100f218 26 May 2011

EXAMINATION ANSWER KEY ILT 10-1 NRC RO Exam Safety ~ Initial License Level 6

Function: D LORT ILT 10-1 NRC & AUDIT EXAM Page: 11 of 218 26 May 2011

EXAMINATION ANSWER KEY ILT 10-1 NRC RO Exam 5 ID: 10*1 NR05 Points: 1.00 The plant is at rated power with Core Spray Main Pump NZ01B in Pull-To-Lock for maintenance.

A seizmic event then occured resulting in a simultaneous LOOP and LOCA in the Drywell. Plant conditions include the following:

  • Drywell Pressure indicates 3.5 psig and rising
  • RPV water level indicates 85 inches and lowering TWO minutes later, which of the following describes which EDGs are powering which Core Spray System pumps?

EDG*1 EDG*2 A. NZ01DONLY NZ01 C, NZ03C, and NZ03B B. NZ01 D and NZ03D ONLY NZ01 C and NZ03C ONLY C. NZ01A, NZ01D, and NZ03A NZ03B ONLY D. NZ01A,NZ01D,NZ03A,and NONE NZ03D Answer: C Answer Explanation QID: 10*1 NR05 Question # I 5 I Developer 1 Date: JJR 17*11*11 Knowledge and Ability Reference Information Importance Rating K&A RO SRO 209001 LPCS K3.03

  • Knowledge of the effect that a loss or malfunction of the LOW PRESSURE CORE 2.9 3.0 SPRAY SYSTEM will have on following:

Emergency generators Level I RO I Tier 2 I Group I 1 General 341 References ILT 10-1 NRC & AUDIT EXAM Page: 12 of 218 26 May 2011

EXAMINATION ANSWER KEY ILT 10-1 NRC RO Exam C is Correct. The question stem provides a condition where Main Core Spray Pump NZ01 B is unavailable and in PTL lAW 341, EDG Operation, during a combined LOOP & LOCA event, if NZ01 B does not start, then the Alternate Pump (NZ01 D) which is powered from EDG*1 will start instead.

NZ01A, NZ03A (both powered from EDG*1) will start and NZ01 B (powered from EDG-2) will start as Explanation expected. The KIA examines the effect on the EDG start logic when a Core Spray component is out of service.

A, B, & D are Incorrect. These distractor are plausible if the applicant does not recall Core Spray pump start logic with NZ01 B not available.

ReferencEts to be None provided during exam:

Lesson Plan 2621.828.0.0010, Core Spray System Learning SDC*10444, Describe the interlock signals and Objecth'el setpoints for the affected system components and expected system response including power loss or failed components.

Question Source (New, Modified, Bank) I New If Bank or Modified: N/A VISION System/Question ID Question Source Memory or X Comprehension Fundamental 2:RI or Analysis Cognitive Knowledge Level NUREG 1021 Appendix B: Recognizing Interaction between systems (plural), including consequences and implications 55.41 I 7 I 55.43 I 10CRF55 Design, components, and functions of control and Content safety systems, including instrumentation, signals, interlocks, failure modes, and automatic and manual features.

Justification for LORT questions with N/A KIA values < 3.0 Time to Complete: 1*2 minutes I Point Value: 1 ILT 10-1 NRC & AUDIT EXAM Page: 130f218 26 May 2011

EXAMINATION ANSWER KEY ILT 10-1 NRC RO Exam System ID No.: 209001 PRA: I NO Safety ~ Initial License Level 2&4 Function: o LORT ILT 1()..1 NRC & AUDIT EXAM Page: 14 of 218 26 May 2011

EXAMINATION ANSWER KEY ILT 10-1 NRC RO Exam 6 ID: 10-1 NR06 Points: 1.00 The plant was at rated power when an event occurred requiring entry into RPV Control no ATWS Level Restoration. The event also required the manual isolation of both Isolation Condensers (lCs).

Consider the EOP step below concerning ICs (which can no longer be performed) .

CONFIRM INITIATION OF THE ISOLATION CONDENSERS What is the EOP basis for the step 'CONFIRM INITIATION OF THE ISOLATION CONDENSERS' and what is the impact now that the ICs are unavailable?

(1) Basis (2) Impact A. (1) To provide a source of makeup water to the RPV.

(2) A significant quantity of cold water is no longer available to help submerge the core.

B. (1) To lower pressure to allow low pressure systems to inject into the RPV.

(2) Reducing pressure will now take longer using alternate pressure control systems.

C. (1) To ensure operability of the Isolation Condensers for subsequent steps in the Level Restoration procedure.

(2) Alternate methods of pressure control that are less desireable than the ICs will be required during Level Restoration.

D. (1) To commence RPV cooldown by reducing RPV pressure as directed by the Pressure Control Leg of RPV Control - no ATWS.

(2) Cooldown lAW the Pressure Control Leg will be required with less desireable alternate methods.

Answer: A Answer Explanation I QID: 10-1 NR06 Question # I 6 I Developer 1 Date: JJR 17-11-11 ILT 10-1 NRC & AUDIT EXAM Page: 15 of 218 26 May 2011

EXAMINATION ANSWER KEY ILT 10-1 NRC RO Exam Knowledge and Ability Reference Information Importance Rating K&A RO SRO 207000 Isc)lation (Emergency) Condenser K3.02 - Knowledge of the effect that a loss or malfunctic)n of the ISOLATION (EMERGENCy) 3.8 4.0 CONDEN!SER will have on following: Reactor water level (EPG's address the isolation condenser as a water source): BWR-2,3 Level I RO I Tier 2 I Group I 1 General EOP User's References Guide A is Correct. lAW the EOP User's Guide, the Isolation Condensers (ICs) contain a significant quantity of relatively cold water within the condenser tube bundles and condensate return piping. Initiating the ICs releases this water, thus providing a source of makeup to the RPV which may help submerge the core for a sufficient length of time to allow other injection systems to be brought on-line. The question asks what impact and why it is a concern that both ICs have been isolated to examine the KIA.

Explanation B is Incorrect. This distractor is plausible since the Isolation Condensers are used to lower RPV Pressure later in the Level Restoration procedure however the question asked the basis for the specific step of 'Confirm Initiation of the Isolation Condensers',.

C and D are Incorrect. *rhese distractors are plausible if the applicant does not recall the basis for

'Confirm Initiation of the Isolation Condensers' and is attempting to make a logical guess.

References to be None provided during exam:

Lesson Plan 2621.845.0.0052, RPV Control- no ATWS Learning ENA-3055, Given a copy of RPV Control, describe Objective/ in detail each step or conditional statement, including technical basis, and how to perform each step as required.

ILT 10-1 NRC & AUDIT EXAM Page: 160f218 26 May 2011

EXAMINATION ANSWER KEY ILT 10-1 NRC RO Exam Question Source (New, Modified, Bank) I New If Bank or Modified: N/A VISION System/Question 10 Question Source Memory or X Comprehension Cognitive Fundamental 1:B or Analysis Level Knowledge NUREG 1021 Appendix B: Bases or purpose 10CRF55 55.41 I 10 I 55.43 I Content Administrative, normal, abnormal, and emergency operating procedures for the facility.

Justification for LORT questions with N/A KIA value~; < 3.0 Time to Complete: 1-2 minutes I Point Value: 1 System I[t No.: 207000 PRA: I NO Safe~v 4

I:8l Initial License Level Function: o LORT ILT 10-1 NRC & AUDIT EXAM Page: 17 of 218 26 May 2011

EXAMINATION ANSWER KEY ILT 10-1 NRC RO Exam 7 10: 10-1 NR07 Points: 1.00 The plant was at rated power when the following annunciator alarmed:

  • DC PWR LOST - BUS C UV The Operator reports the following indications:
  • BATf CAMPS - Downscale
  • BATT C VOLTS - Downscale Which of the following states the impact on the MSIVs and the required action?

Impact on MSIVs Required Action A. ALL MSIVs close Execute ABN-1, Reactor Scram, immediately B. ONLY the outboard MSIVs Execute ABN-1, Reactor close Scram, immediately C. ALL MSIVs remain open, but Manually transfer the DC ALL auto isolation capability is isolation logic to Bus DC B lost D. ALL MSIVs remain open, but Reset the MSIV isolation when SOME position indication is DC power is restored lost Answer: D Answer Explanation I QID: 10-1 NR07 Question # I 7 I Developer 1 Date: JJR 17-11-11 E

Knowledge and Ability Reference Information K&A Importance Rating

______________________~_R_O~~I~S_R~O~

ILT 10-1 NRC & AUDIT EXAM Page: 18 of 218 26 May 2011

EXAMINATION ANSWER KEY ILT 10-1 NRC RO Exam 223002 PCIS/Nuclear Steam Supply Shutoff K4.06 - KI110wledge of PRIMARY CONTAINMENT ISOLA-rlON SYSTEM/NUCLEAR STEAM SUPPLY SHUT 3.4 3.5 OFF design feature(s) and/or interlocks which provide fair the following: Once initiated, system re:set requires deliberate operator action Level I RO I Tier 2 I Group I 1 General ABN-55 237E566 She 12 301.1 References D is Correct. The plant is at rated power when 125 VDC Bus C is lost. DC C supplies 125 VDC Power Panel F, and there is no supply transfer scheme.

When DC C is lost, so is DC F. DC F supplies the electrical power to the DC solenoids used in the MSIV automatic isolation logic. There are also AC powered solenoids in the isolation logic. For the MSIVs to auto close, both the DC powered and AC powered solenoids must de-energize. Therefore, Explanation with the loss of only 1 set of solenoids, all MSIVs remain open. lAW ABN-55, DC Bus C and Panel Failures, when DC power is restored to DC F, then reset the MSIV isolation.

A and B are Incorrect but plausible if the applicant believes the MSIVs will close.

C is Incorrect but plausible if the applicant believes the auto isolation logic will no longer function.

References to be None provided durin ~ exam:

Lesson Plan 2621.828.0.0030, NSSS Learning NSS*3956, List the signals which initiate automatic Objectivel closure of the MSIVs and the setpoints of these signals.

Question Source (New, Modified, Bank) I Bank If Bank or Modified:

VISION Sy'stem/Question ID 667523 Question Source ILT 08-1 RO Audit Exam ILT 10*1 NRC & AUDIT EXAM Page: 19 of 218 26 May 2011

EXAMINATION ANSWER KEY ILT 10-1 NRC RO Exam Memory or X Comprehension Fundamental 3:PEO Cognitive or Analysis Knowledge Level NUREG 1021 Appendix B: Predict an Event or Outcome 55.41 I 7 I 55.43 I 10CRF55 Design, components, and functions of control and Content safety systems, including instrumentation, Signals, interlocks, failure modes, and automatic and manual features.

Justification for LORT questions with N/A KIA values < 3.0 Time to Complete: 1*2 minutes I Point Value: 1 System ID No.: 223002 PRA: I NO Safety [gI Initial license Level 5

Function: o LORT ILT 10-1 NRC & AUDIT EXAM Page: 20 of 218 26 May 2011

EXAMINATION ANSWER KEY ILT 10-1 NRC RO Exam 8 ID: 10-1 NR08 Points: 1.00 Which one of the following is the reason for the KIRK KEY INTERLOCK associated with Transformer PS-1 disconnect switches SW-733-169 and SW-733-170?

To ensure tlhat...

A. the RPS MGs cannot be synchronized at any time.

B. Protection Panel PSP-2 has a redundant source of power.

C. the RPS MGs are synchronized prior to being transferred to PS-1.

D. PS-1 cannot be energized from VMCC 1A2 and VMCC 1B2 atthe same time.

Answer: D Answer Explanation I QID: 10-1 NR08 Question # I 8 I Developer / Date: JJR / 7-11-11 Knowledge and Ability Reference Information Importance Rating K&A RO SRO 212000 RFtS K4.03 - Knowledge of REACTOR PROTECTION SYSTEM design feature(s) and/or interlocks 3.0 3.1 which provide for the following: The prevention of supplying power to a given RPS bus from multiple sources simultaneously Level I RO I Tier 2 I Group I 1 General 913E911 ABN-48 Referenct:ts ILT 10-1 NRC & AUDIT EXAM Page: 21 of 218 26 May 2011

EXAMINATION ANSWER KEY ILT 10-1 NRC RO Exam o is Correct. The question stem asks for the purpose of the Kirk Key Interlock associated with PS*1 disconnect switches SW-733-169 (which provides PS-1 power from VMCC 1A2) and SW-733 170 (which provides PS-1 power from VMCC 1B2).

The intelocks design function is to prevent both disconnect switches from being closed at the same time, thereby allowing only one RPS bus to be powered from its power source at any given time (and also prevents powering a single RPS bus from multiple power sources at the same time). 'rhis question is operationally relevent since these actions are taken whenever the crew is recovering from a loss of power to an RPS bus (PSP-1 or PSP-Explanation 2).

A is Incorrect. This distractor is plausible if the applicant does not recall the RPS electrical power supply lineup.

B is Incorrect. It is true that PSP-2 has a redundant source of power however the question is asking what the purpose of the Kirk Key Interlock is. This distractor is plausible if the applicant does not answer the question being asked.

o is Incorrect. This distractor is plausible if the applicant does not recall the RPS electrical power supply lineup.

References to be None provided during exam:

Lesson Plan 2621.828.0.0037, Reactor Protection System Learning SOC-10436, Using plant procedures and electrical Objective/ drawings, determine electrical power supply for system equipment and any associated/applicable logic, including power loss effects.

Question Source (New, Modified, Bank) I Bank If Bank or Modified:

VISION System/Question 10 506946 Question Source 192 Memory or Cognitive X Comprehension Fundamental Level 1:B or Analysis Knowledge ILT 10-1 NRC & AUDIT EXAM Page: 22 of 218 26 May 2011

EXAMINATION ANSWER KEY ILT 10-1 NRC RO Exam NUREG 1021 Appendix B: Bases or purpose 55.41 I 7 I 55.43 I 10CRF55 Design, components, and functions of control and Content safety systems, including instrumentation, signals, interlocks, failure modes, and automatic and manual features.

Justification for LORT que'stions with N/A KIA valuel; < 3.0 Time to Complete: 1-2 minutes I Point Value: 1 System I[) No.: 212000 PRA: I NO Safety ~ Initial License Level 7

Functi()n: D LORT ILT 10-1 NRC & AUDIT EXAM Page: 23 of 218 26 May 2011

EXAMINATION ANSWER KEY ILT 10-1 NRC RO Exam 9 ID: 10-1 NR09 Points: 1.00 The plant was at rated power with EDG-1 out of service for maintenance.

Subsequently, a combined LOOP and LOCA occurred simultaneously. Plant conditions include the following:

  • Drywell pressure is 5 psig and rising
  • RPV water level is 70 inches and lowering Which one of the following occurs when the EDG-2 output breaker closes?

A. NZ03B will start in 0 seconds B. NZ01 B will start in 10 seconds C. NZ01 C will start in 10 seconds D. NZ03C will start in 20 seconds Answer: C Answer Explanation I QID: 10-1 NR09 Question # I 9 I Developer 1 Date: JJR 17-11-11 Knowledge and Ability Reference Information Importance Rating K&A RO SRO 264000 EDGs KS.06 .. Knowledge of the operational implications of the following concepts as they 3.4 3.S apply to EMERGENCY GENERATORS (DIESEUJET) : Load sequencing Level I RO I Tier 2 I Group I 1 General 341 References ILT 10-1 NRC & AUDIT EXAM Page: 24 of 218 26 May 2011

EXAMINATION ANSWER KEY ILT 10-1 NRC RO Exam C is Correct. The question stem provides a condition where there is a LOOP*LOCA with EDG*1 OOS. In order for the LOOP-LOCA EDG logic to irlitiate, Drywell pressure must be > 2.9 psig and RPV level must be < 90 inches (which is why this must be stated in the stem). lAW 341, EDG Operation, the only statement that is true is that NZ01 C will start 10 Explanatic)n seconds after the EDG*2 output breaker closes.

A, B, and D are Incorrect. These distractors are plausible if the applicant does not recall LOOP-LOCA EDG load sequences with 1 EDG OOS. All pumps would be powered from EDG-2, however the times they start are incorrect.

References to be None provided durin ~ exam:

Lesson Plan 2621.828.0.0013, Emergency Diesel Generators Learning EDG*813, Explain the differences between normal Objective/ EDG start sequence and fast start sequence, including trip bypasses and automatic fault resets.

Question Source (New, Modified, Bank) I New If Bank or Modified: N/A VISION System/Question 10 Question Source Memory or X Comprehension Fundamental 1:1 Cognitive or Analysis Knowledge Level NUREG 1021 Appendix B: Interlocks, setpoints, or system (singular) response 55.41 I 7 I 55.43 I 10CRF55 Design, components, and functions of control and Content safety systems, including instrumentation, signals, interlocks, failure modes, and automatic and manual features.

Justification for LORT que:stions with N/A KIA valuesi < 3.0 Time to Cc)mplete: 1*2 minutes I Point Value: 1 System ID No.: 264000 PRA: I NO Safety ~ Initial License Level 6

Function: o LORT ILT 10*1 NRC & AUDIT EXAM Page: 25 of 218 26 May 2011

EXAMINATION ANSWER KEY ILT 10-1 NRC RO Exam ILT 10-1 NRC & AUDIT EXAM Page: 26 of 218 26 May 2011

EXAMINATION ANSWER KEY ILT 10-1 NRC RO Exam 10 10: 10*1 NR010 Points: 1.00 The Main Generator is in the process of being synchronized to the grid via GD1 lAW 315.1, Turbine Generator Startup.

Panel 8F/9F Main Generator Synchronizing indications are as follows:

  • RUNNING voltage indicates 230 KV
  • INCOMMING voltage indicates 231 KV
  • FREQUENCY indicates 60.0 CYCLES (Hz)
  • SYNCHROSCOPE is rotating in the SLOW direction (counterclockwise) lAW 315.1, which of the following actions are REQUIRED before GD1 can be closed?

A. RAISE RUNNING voltage until it equals INCOMMING voltage.

B. RAISE Main Generator speed using the Speed Load Changer.

C. LOWER Main Generator speed using the Speed Load Changer.

D. LOWER INCOMMING voltage until it is less than RUNNING voltage.

Answer: B Answer Explanation I QID: 10-1 NR010 Question # I 10 I Developer I Date: JJR 17-11-11 Knowledge and Ability Reference Information Importance Rating K&A RO SRO 262001 AC Electrical Distribution K5.01

  • Knowledge of the operational implications of the following concepts as they 3.1 3.4 apply to A.C. ELECTRICAL DISTRIBUTION:

Principle involved with paralleling two A.C.

sources Level I RO I Tier 2 I Group I 1 General 315.1 References ILT 10-1 NRC & AUDIT EXAM Page: 27 of 218 26 May 2011

EXAMINATION ANSWER KEY ILT 10-1 NRC RO Exam B is Correct. The question stem provides a condition where the Main Generator is about to be synchronized to the grid (paralleling two AC power sources). Part of the requirements are to have Generator INCOMMING voltage slightly higher than RUNNING voltage, which it is. Another requirement is to ensure the synchroscope is rotating slowly in the FAST direction (the stem states it's rotating in Explanati()n the SLOW direction. 315.1 directs raising Generator speed using the Speed Load Changer which will cause the synchroscope to change direction.

B, C and D are Incorrect. These distractors are plausible if the applicant does not recall the requirements of 315.1 and that of paralleling two AC power sources.

References to be None provided during exam:

Lesson Plan 2621.828.0.0016, Electrical Distribution Learning SDC*10447, Given normal operating procedures Objective/ and documents for the system, describe or interpret the procedural steps.

Question Source (New, Modified, Bank) I New If Bank or Modified: N/A VISION System/Question ID Question Source Memory or X Comprehension Fundamental 1:P Cognitive or Analysis Knowledge Level NUREG 1021 Appendix B: erocedure steps and cautions 10CRF55 55.41 I 10 I 55.43 I Content Administrative, normal, abnormal, and emergency operating procedures for the facility.

Justification for LORT questions with N/A KIA values < 3.0 Time to C()mplete: 1-2 minutes I Point Value: 1 System ID No.: 262001 PRA: I NO Safety ~ Initial License Level 6

Function: D LORT ILT10-1 NRC & AUDIT EXAM Page: 28 of 218 26 May 2011

EXAMINATION ANSWER KEY ILT 10-1 NRC RO Exam ILT 10-1 NRC & AUDIT EXAM Page: 29 of 218 26 May 2011

EXAMINATION ANSWER KEY ILT 10-1 NRC RO Exam 11 10: 10-1 NR011 Points: 1.00 The plant was at rated power when the following annunciators alarmed:

  • PROT SYS PNL 2 PWR LOST
  • CIP-~I PWR XFER

A. RWCU Isolation B. Full Reactor Isolation C. APRMs 1-4 fail downscale D. APRMs 5-8 fail downscale Answer: 0 Answer Explanation I QID: 10-1 NR011 Question # I 11 I Developer / Date: JJR / 7-11-11 Knowledge and Ability Reference Information Importance Rating K&A RO SRO 262002 UPS CAC/DC)

K6.01 - Knowledge of the effect that a loss or malfunction of the following will have on the 2.7 2.9 UNINTERRUPTABLE POWER SUPPLY (A.C./D.C.) : A.C. electrical power Level I RO I Tier 2 I Group I 1 General ABN-51 References ILT 10-1 NRC & AUDIT EXAM Page: 30 of 218 26 May 2011

EXAMINATION ANSWER KEY ILT 10-1 NRC RO Exam NOTE: This question refers to the loss of power to a Vital AC distribution system bus since Oyster Creek does not have a designated ups. This has been an approved method of examining this KIA on the previous two NRC exams.

o is Correct. The question stem states a loss of power to VMCC 1B2 (which feeds RPS MG Set 2, IP-4 and CIP-3), which includes loss of power to RPS MG set 2 to PSP-2. When RPS MG Set 2 output is lost, APRMs 5-8 fail downscale.

A is Incorrect. This distractor is plausible if the applicant does not recall that the RWCU system is Explanatic)n unaffected by the loss of VMCC 1B2 or does not recognize from the conditions in the stem that VMCC 1B2 was lost.

B is Incorrect. This distractor is plausible if the applicant believes the conditions in the stem (SCRAM CONTACTOR OPEN annunciator) indicates a full Scram and Reactor Isolation have occurred.

C is Incorrect. This distractor is plausible if the applicant does not interpret the indications provided correctly to determine that PSP-2 lost power (which powers APRMs 5-8) and not PSP-1 (which powers APRMs 1-4).

References to be None provided durin ~ exam:

Lesson Plan 2621.828.0.0056, Vital AC Distribution Learning VAC-10436, Using plant procedures and drawings, Objective/ determine electrical power supply for system equipment and any associated applicable logic, including power loss effects.

Question Source (New, Modified, Bank) I Modified If Bank or Modified:

VISION System/Question 10 607666 Question Source VAC-500 ILT 10-1 NRC & AUDIT EXAM Page: 31 of 218 26 May 2011

EXAMINATION ANSWER KEY ILT 10-1 NRC RO Exam Memory or X Comprehension Fundamental 3:SPK Cognitive or Analysis Knowledge Level NUREG 1021 Appendix B: Solve a Problem using Knowledge and its meaning 55.41 I 7 I 55.43 I 10CRF55 Design, components, and functions of control and Content safety systems, including instrumentation, signals, interlocks, failure modes, and automatic and manual features.

Justification for LORT questions with N/A KIA values < 3.0 Time to Complete: 1-2 minutes I Point Value: 1 System ID No.: 262002 PRA: I NO Safety [gl Initial License Level 6

Functic)n: o LORT ILT 10-1 NRC & AUDIT EXAM Page: 32 of 218 26 May 2011

EXAMINATION ANSWER KEY ILT 10-1 NRC RO Exam 12 10: 10-1 NR012 Points: 1.00 A plant startup has just commenced. An event then occurs which results in the loss of power to thle SRM 24 drawer.

Which of the following is correct for these conditions?

A. SRM indication digital recorder on Panel 4F has lost power.

B. SRM Channel 24 SRM PERIOD on Panel 4F indicates infinity.

C. SRM Channel 24 PERIOD meter on Panel 5R indicates downscale.

D. SRM Channel 24 COUNTS PER SECOND meter on Panel 5R indicates upscale.

Answer: C Answer Explanation I QID: 10-1 NR02 Question # I 2 I Developer 1 Date: JJR 17-11-11 Knowledge and Ability Reference Information Importance Rating K&A RO SRO 215004 Sc)urce Range Monitor K6.04 - Knowledge of the effect that a loss or malfunctilon of the following will have on the 2.9 2.9 SOURCE RANGE MONITOR (SRM) SYSTEM :

Detectors Level I RO I Tier 2 I Group I 1 General 706E812 References Sh.4 ILT 10-1 NRC & AUDIT EXAM Page: 33 of 218 26 May 2011

EXAMINATION ANSWER KEY ILT 10-1 NRC RO Exam C is Correct. 24 VDC powers the SRM drawer, including the trip relays and detector. A loss of instrument power results in the downscale indication of the SRM meters and period meters, both on Panel 5R and 4F. SRM 24 Period meter on Panel 5R will therefore indicate downscale from a loss of instrument power to the SRM 24 detector.

Explanati10n A is Incorrect. The SRM digital display indication on Panel 4F receives power from 120 VAC CIP Div 1, therefore the digital display has not lost power.

Band D are Incorrect. These distractors are plausible if the applicant does not recall the effect of a loss if instrument power to an SRM channel detector drawer on SRM indications.

References to be None provided durin~ exam:

Lesson Plan 2621.828.0.0029, Nuclear Instrumentation Learning NIS-10436, Using plant procedures and electrical Objective/ drawings, determine electrical power supply for system equipment and any associated/applicable logic, including power loss effects.

Question Source (New, Modified, Bank) I Bank If Bank or Modified:

VISION S),stem/Question ID 510841/ OC RO NRC 19 Question Source ILT 05-1 NRC Exam Memory or X Comprehension Fundamental 1 :1 Cognitive or Analysis Knowledge Level NUREG 1021 Appendix B: Interlocks, setpoints, or system (singular) response 55.41 I 7 I 55.43 I 10CRF55, Design, components, and functions of control and Content safety systems, including instrumentation, signals, interlocks, failure modes, and automatic and manual features.

Justification for LORT questions with N/A KIA value:s < 3.0 Time to Complete: 1-2 minutes I Point Value: 1 System I[) No.: I 215004 I PRA: I NO ILT 10-1 NRC & AUDIT EXAM Page: 34 of 218 26 May 2011

EXAMINATION ANSWER KEY ILT 10-1 NRC RO Exam Safety ~ Initial License Level 7

Function: o LORT ILT 10-1 NRC & AUDIT EXAM Page: 35 of 218 26 May 2011

EXAMINATION ANSWER KEY IlT 10-1 NRC RO Exam 13 ID: 10-1 NR013 Points: 1.00 The following plant conditions and sequence of events occur:

  • The plant is operating at 30% power
  • Master Feed Controller is set at 163"
  • Reactor water level is 163"
  • Reactor pressure is 1020 psig At T = 0 seconds, a manual scram is inserted and a hydraulic A TWS occurs At T =60 seconds the following plant conditions exist:
  • Reactor power is 20%
  • Reactor water level has lowered to 155"
  • Reactor pressure is 1010 psig With NO operator action, how will the feedwater control system respond to maintain level?

Reactor water level will be automatically controlled at a ...

A. 142" setpoint using the low flow regulating valves B. 163" setpoint using the low flow regulating valves

c. 142" setpoint using the main feed regulating valves D. 163" setpoint using the main feed regulating valves Answer: 0 Answer Explanation I QID: 10-1 NR013 Question # I 13 I Developer I Date: JJR I 7-11-11 Knowledge and Ability Reference Information Importance Rating K&A RO I SRO ILT 10-1 NRC & AUDIT EXAM Page: 36 of 218 26 May 2011

EXAMINATION ANSWER KEY ILT 10-1 NRC RO Exam 259002 Reactor Water Level Control A1.05* Ability to predict and/or monitor changes in parameters associated with operating the REACTOR WATER LEVEL 2.9 2.9 CONTROl. SYSTEM controls including:

FWRV/startup level control position: Plant*

Specific Level I RO I Tier 2 I Group I 1 Generall MDD*OC*625*B Referencl9s Div I o is Correct. A normal scram is processed through RPS to Feedwater Control and validated by a 10 "

drop in level. Though this scram was processed by RPS a 10" drop in level did not validate it due to the hydraulic lock. The post scram level control setpoint of 142" TAF will not be substituted. 'rhere is no automatic transfer to the low flow valves for level control nor will feed flow requirements at this power level allow a manual swap to the low flow valves as would be the case in a successful scram. Because the scram was not validated by a 10" drop in level, Explanation the FW control system will continue to attempt to maintain level at the existing setpoint of 163"TAF without post scram level control.

A and C are Incorrect. These distractors are plausible if the applicant believes the post scram level control setpoint will have been processed.

B is Incorrect. This distractor is plausible if the applicant believes the Low Flow Valves will be regulatin~ flow for these conditions.

References to be None provided durin ~ exam:

Lesson Plan 2621.828.0.0018, Feedwater Control System Learning FWC-10446, Identify and explain system operating Objecti"e/ controls / indications under all plant operating conditions.

Question Source (New, Modified, Bank) I Bank If Bank or Modified:

VISION S3,stem/Question 10 608199 Question Source ILT 07*1 RO Comp #2 ILT 10-1 NRC & AUDIT EXAM Page: 37 of 218 26 May 2011

EXAMINATION ANSWER KEY ILT 10-1 NRC RO Exam Memory or Comprehension X Fundamental Cognitive or Analysis 2:DR Knowledge Level NUREG 1021 Appendix B: Describing or Recognizing relationships 55.41 I 5 I 55.43 I Facility operating characteristics during steady state 10CRF55 and transient conditions, including coolant chemistry, Content causes and effects of temperature, pressure and reactivity changes, effects of load changes, and operating limitations and reasons for these operating characteristics.

Justification for LORT questions with N/A KIA values < 3.0 Time to Complete: 1-2 minutes I Point Value: 1 System ID No.: 259002 PRA: I NO Safety L?:?l Initial License Level 2

Function: o LORT ILT 10-1 NRC & AUDIT EXAM Page: 38 of 218 26 May 2011

EXAMINATION ANSWER KEY ILT 10*1 NRC RO Exam 14 10: 10..1 NR014 Points: 1.00 The plant is at rated power with the following plant conditions:

  • Drywell pressure rises to 3.6 psig due to a steam leak.

ONE MINUTE later, Drywell pressure indicates 2.6 psig.

If the RO depresses the DRYWELL ISOLATION RESET pushbutton on Panel 4F, how will the SGTS AND Reactor Building differential pressure (~P) indication on Panel 11 R respond?

SGTS Response Reactor Building AP Response A. Shutdown occurs Goes to zero B. Continues to run Remains the same C. Shutdown occurs Remains the same D. Continues to run Becomes more negative Answer: A Answer Explanation I QID: 10..1 NR014 Question # I 14 I Developer I Date: JJR /7-11-11 Knowledge and Ability Reference Information Importance Rating K&A RO SRO 261000 SGTS A 1.04 .. Ability to predict and/or monitor changes in parameters associated with operating the STANDBY GAS TREATMENT 3.0 3.3 SYSTEM controls Including: Secondary containment differential pressure Level I RO I Tier I 2 I Group I 1 ILT 10-1 NRC & AUDIT EXAM Page: 39 of 218 26 May 2011

EXAMINATION ANSWER KEY ILT 10-1 NRC RO Exam General 330 References A is Correct. A high drywell pressure is an initiating signal for the SGTS. After the condition clears and the drywell isolation reset is depressed, an automatic shutdown of the SGTS occurs. Since the running exhaust fan trips a negative pressure is no longer maintained, pressure is equalized and goes to zero.

B is Incorrect. This distractor is plausible if the applicant does not recall that SGTS Shutdown, not continue to run for these conditions.

Explanation C is Incorrect. This distractor is plausible if the applicant does not recall that RB AP becomes less negative and goes to zero. The applicant may believe there is no reason for RB AP to drift to zero.

o is Incorrect. This distractor is plausible if the applicant does not recall that SGTS Shutdown, not continue to run for these conditions. It is plausible with continued SGTS operation, RB AP might be more negative.

References to be None provided durin ~ exam:

Lesson Plan 2621.828.0.0042, Secondary Containment & SGTS Learning SGT-10439, Given the system logic/electrical Objective/ drawings, describe the system auto initiation Signals, setpoints and expected system response including power loss or failed components.

Question Source (New, Modified, Bank) I Modified If Bank or Modified:

VISION S3,stem/Question 10 506540/IRH-21-28-0042 Question Source ILT 07-1 Comp 1 Memory or X Comprehension Fundamental 3:PEO Cognitive or Analysis Knowledge Level NUREG 1021 Appendix B: Predict an Event or Outcome ILT 10-1 NRC & AUDIT EXAM Page: 40 of 218 26 May 2011

EXAMINATION ANSWER KEY ILT 10-1 NRC RO Exam 55.41 I 7 I 55.43 I 10CRF55 Design, components, and functions of control and Content safety systems, including instrumentation, signals, interlocks, failure modes, and automatic and manual features.

Justification for LORT questions with N/A KIA values < 3.0 Time to Complete: 1*2 minutes I Point Value: 1 System ID No.: 261000 PRA: I NO Safe~v rgj Initial License Level 9

Functicm: o LORT ILT 10-1 NRC & AUDIT EXAM Page: 41 of 218 26 May 2011

EXAMINATION ANSWER KEY ILT 10-1 NRC RO Exam 15 ID: 10-1 NR015 Points: 1.00 The operating crew is raising power with control rods after an outage. The plant is in 5 recirc loop operation.

Plant indications include the following:

  • Reactor power is 85%
  • TOTAL RECIRC FLOW (PaneI4F) indicates 150 x 103 GPM The following annunciator alarmed:
  • APRM FLO BIAS OFF NORMAL Investigation revealed that the flow transmitter in the "C" Recirc. Loop that feeds the TOTAL RECIRC FLOW indicator on 4F, failed to 0 (zero).

Which of the following states the impact of the above alarm/indications AND what action is required?

ACTION A. ONLY a rod block exists Place the affected APRMs in BYPASS and continue raising power B. ONLY a rod block exists Hold power until the "Cit flow transmitter can be returned to service C. A rod block AND a 1/2 scram Hold power until the "c" flow exists transmitter can be returned to service D. A rod block AND a 1/2 scram Place the affected APRMs in exists BYPASS and continue raising power Answer: B Answer Explanation I QID: 10-1 NR015 Question # I 15 I Developer I Date: J..IR 17-11-11 ILT 10-1 NRC & AUDIT EXAM Page: 42 of 218 26 May 2011

EXAMINATION ANSWER KEY ILT 10-1 NRC RO Exam Knowledge and Ability Reference Information Importance Rating K&A RO SRO 215005 APRM I LPRM A2.05 - Ability to (a) predict the Impacts of the following on the AVERAGE POWER RANGE MONITORJLOCAL POWER RANGE MONITOR 3.5 3.6 SYSTEM; and (b) based on those predictions, use procedures to correct, control, or mitigate the consequences of those abnormal conditions: Loss of recirculation flow signal Level I RO I Tier 2 I Group I 1 General 202.1 RAP-H7a RAP*G5f References B is Correct. Prior to the failure, each (Division 2) flow transmitter is sensing approximately 30.0 E3 gpm, which is summed to produce 150.0 E3 total recirc flow. One transmitter failing to zero results in a total indicated recirc flow of 120.0 E3 gpm. This produces in a 20% mismatch between the Division 1 and 2 recirc flow monitors, causing a flow comparator rod block (setpoint 10%). Also, from the Power Operations Curve, there is no Y2 scram from this power and flow. There is no bypass for this type of rodblock, and the only way to resume the power rise is to return the failed sensor to service.

Explanation A is Incorrect. This distractor is plausible if the applicant does not recall that bypassing APRMs for this condition will not clear the Rod Block.

C is Incorrect. This distractor is plausible if the applicant does not recall that there is no 1/2 scram from the conditions provided.

C is Incorrect. This distractor is plausible if the applicant does not recall that there is no 1/2 scram from the conditions provided. In addition, bypassing APRMs for this condition will not clear the Rod Block.

References to be Attachment provided during exam: 202.1-2 ILT 10*1 NRC & AUDIT EXAM Page: 430f218 26 May 2011

EXAMINATION ANSWER KEY ILT 10-1 NRC RO Exam Lesson Plan 2621.828.0.0029, Nuclear Instrumentation Learning NIS-10445, Given a set of system indications or Objecth,e/ data, evaluate and interpret them to determine limits, trends and system status.

Question Source (New, Modified, Bank) I Modified If Bank or Modified:

VISION System/Question ID 608584/IRH-21-28-0029 Question Source ILT 07-1 Comp 3 Memory or X Comprehension Fundamental 3:PEO Cognitive or Analysis Knowledge Level NUREG 1021 Appendix B: ~redict an Event or Outcome 55.41 I 7 I 55.43 I 10CRF55~

Design, components, and functions of control and Content safety systems, including instrumentation, signals, interlocks, failure modes, and automatic and manual features.

Justification for LORT questions with N/A KIA values < 3.0 Time to Complete: 1-2 minutes I Point Value: 1 System ID No.: 215005 PRA: I NO Safety 7

L8J Initial License Level Function: o lORT ILT 10-1 NRC & AUDIT EXAM Page: 44 of 218 26 May 2011

EXAMINATION ANSWER KEY ILT 10-1 NRC RO Exam 16 10: 10-1 NR016 Points: 1.00 The plant is at 20% power during an ascension to rated power. An event then occurs resulting in the crew executing Emergency Depressurization (ED). Plant conditions include the following:

  • All Control Rod indications on Panel 4F indicate a green backlight
  • All EMRV Control switches on Panel 1F/2F are in MAN
  • Reactor Pressure indicates 5 psig
  • RPV Water Level indicates 165 inches
  • Torus Pressure indicates 1.5 psig What is the correct status of all EMRV acoustic indications on Panel 1F/2F AND required action (lAW the ED procedure) associated with the EMRVs, if any?

All EMRVs Acoustics Indicate In The... Required ktion A. VALVE OPEN REGION Place All EMRVs in AUTO B. VALVE CLOSED REGION Leave All EMRVs in MAN C. VALVE OPEN REGION Leave All EMRVs in MAN D. VALVE CLOSED REGION Place All EMRVs in AUTO Answer: B Answer Explanation I QID: 10-1 NR016 Question # I 16 I Developer 1 Date: JJR 17-11-11 Knowledge and Ability Reference Information Importance Rating K&A RO SRO 239002 SRVs A2.05 - Ability to (a) predict the impacts of the following on the RELIEF/SAFETY VALVES; and (b) based on those predictions, use 3.2 3.4 procedures to correct, control, or mitigate the consequences of those abnormal conditions or operations: Low reactor pressure ILT 10-1 NRC & AUDIT EXAM Page: 450f218 26 May 2011

EXAMINATION ANSWER KEY ILT 10-1 NRC RO Exam Level I RO I Tier 2 I Group I 1 General EOP User's ED-noATWS References Guide EOP B is Correct. The question stem provides a condition where all EMRVs have been manually opened for ED. When RPV pressure lowers to where there is < 50 psid between the RPV and Torus, the EMRVs will close. The ED procedure has the operator leave the EMRVs in MAN until the ED procedure has been exited.

A is Incorrect. This distractor is plausible if the applicant does not recall that EMRVs solenoid indication will indicate closed when there is < 50 Explanation psid between RPV pressure and Torus pressure. In addition, the ED procedure has the crew leave all EMRVs in MAN.

C is Incorrect. This distractor is plausible if the applicant does not recall that EMRVs solenoid indication will indicate closed when there is < 50 psid between RPV pressure and Torus pressure.

D is Incorrect. "rhis distractor is plausible if the applicant does not recall that the ED procedure has the crew leave all EMRVs in MAN.

References to be None provided durin~ exam:

Lesson Plan 2621.845.0.0054, Emergency Depressurization Learning EED-9572, Given a copy of the ED EOP, describe Objectivel the technical basis for each step or conditional statement of the procedure.

Question Source (New, Modified, Bank) I New If Bank or Modified: N/A VISION S~rstem/Question ID Question Source Memory or Comprehension X

Fundamental 3:PEO Cognitive or Analysis Knowledge Level NUREG 1021 Appendix B: Predict an Event or Outcome 10CRF55 55.41 10 55.43 Content ILT 10-1 NRC & AUDIT EXAM Page: 46 of 218 26 May 2011

EXAMINATION ANSWER KEY ILT 10-1 NRC RO Exam Administrative, normal, abnormal, and emergency operating procedures for the facility.

Justification for LORT questions with N/A KIA values < 3.0 Time to C4)mplete: 1-2 minutes I Point Value: 1 System 10 No.: 239002 PRA: I NO Safety 3

I:8J Initial License Level Function: o LORT ILT 10-1 NRC & AUDIT EXAM Page: 47 of 218 26 May 2011

EXAMINATION ANSWER KEY ILT 10-1 NRC RO Exam 17 ID: 10..1 NR017 Points: 1.00 A plant startup is in progress with the REACTOR MODE SELECTOR switch in STARTUP. An event then occurs and IRM 15 fails INOP.

Which of the following conditions will occur as a result of this event?

A. A 1/2 scram ONLY B. A Rod Block ONLY C. A Rod Block AND a 1/2 scram D. NEITHER a Rod Block OR 1/2 scram Answer: C Answer Explanation I QID: 10-1 NR017 Question # I 17 I Developer 1 Date: JJR 17-11-11 Knowledge and Ability Reference Information Importance Rating K&A RO SRO 2150031RM A3.03 .. Ability to monitor automatic operations 3.7 3.6 of the INTERMEDIATE RANGE MONITOR (IRM)

SYSTEM including: RPS status Level I RO I Tier 2 I Group I 1 General RAP-G2e RAP-H7a References ILT 10-1 NRC & AUDIT EXAM Page: 480f218 26 May 2011

EXAMINATION ANSWER KEY ILT 10-1 NRC RO Exam C is Correct. An IRM that's failed INOP with the REACTOR MODE SELECTOR switch in STARTUP or REFUEL will result in both a rod block and 1/2 scram.

A is Incorrect. This distractor is plausible if the applicant does not recall that a rod block also results from this event.

B is Incorrect. This distractor is plausible if the applicant does not recall that a 1/2 scram also results from this event.

Explanation D is Incorrect. This distractor is plausible if the applicant does not recall that both a rod block and 1/2 scram results from this event.

NOTES:

1) This question left as Low Cognitive due to exam having the maximum limit of 45 High Cognitive questions on the RO exam already.
2) The question stem must state that the Mode Switch is in Startup since the Mode Switch in Run would change the answer. The Mode Switch is placed in Run during a startup when all APRM downscale alarms are clear.

References to be None provided during exam:

Lesson Plan 2621.828.0.0029, Nuclear Instrumentation Learning NIS-10441 , Given the system logic/electrical Objective/ drawings, describe the system trip signals, setpoints and expected system response including power loss or failed components.

Question Source (New, Modified, Bank) I Modified If Bank or Modified:

VISION System/Question ID 608227 /IRL..21*28..0029 Question Source ILT 07..1 Comp 2 Memory or X Comprehension Fundamental 1:1 Cognitive or Analysis Knowledge Level NUREG 1021 Appendix B: Interlocks, setpoints, or system (singular) response ILT 10-1 NRC & AUDIT EXAM Page: 490f218 26 May 2011

EXAMINATION ANSWER KEY ILT 10-1 NRC RO Exam 55.41 I 7 I 55.43 I 10CRF55 Design, components, and functions of control and Content safety systems, including instrumentation, signals, interlocks, failure modes, and automatic and manual features.

Justification for LORT queistions with N/A KIA values < 3.0 Time to Complete: 1-2 minutes I Point Value: 1 System ID No.: 215003 PRA: I NO Safety ~ Initial License Level 7

Function: D LORT ILT 10-1 NRC & AUDIT EXAM Page: 50 of 218 26 May 2011

EXAMINATION ANSWER KEY ILT 10-1 NRC RO Exam 18 10: 10-1 NR018 Points: 1.00 The plant was at rated power when an event occurred resulting in an ATWS.

The RO has just placed the Standby Liquid Control (SLC) System 1 keylock to FIRE SYS 1.

ONE MINUTE later, which of the following shows the correct Reactor Water Cleanup (RWCU) valve indications?

NO'rE:

V-16-1: RWCU CLEANUP SYSTEM isolation (Panel 11 F)

V-16-2: RWCU AUX PUMP SUCTION (Panel 3F)

V-16-14: RWCU SYSTEM INLET (PaneI3F)

V-16-61: RWCU SYSTEM OUTLET (Panel 3F)

Y-16-1 Y-16-2 V-16-14 Y-16-61 A. CLOSED CLOSED CLOSED OPEN B. CLOSED CLOSED OPEN OPEN C. OPEN OPEN CLOSED CLOSED D. CLOSED CLOSED CLOSED CLOSED Answer: A Answer Explanation I QID: 10-1 NR018 Question # I 18 I Developer / Date: JJR / 7-11-11 Knowledge and Ability Reference Information Importance Rating K&A RO SRO 211000 SLC A4.06 - Ability to manually operate and/or 3.9 3.9 monitor in the control room: RWCU system isolation: Plant-Specific Level I RO I Tier 2 I Group I 1 General RAP-G1b References ILT 10-1 NRC & AUDIT EXAM Page: 51 of 218 26 May 2011

EXAMINATION ANSWER KEY ILT 10*1 NRC RO Exam A is Correct. lAW RAp*G1 b, when SLC is initiated, RWCU valves V*16*1, V*16*2, and V*16*14 all isolate.

RWCU valve V-16-61 remains open.

Explanati40n B, C, and D are Incorrect. These distractors are plausible if the applicant does not recall which RWCU valves isolate when SLC is initiated.

Reference~s to be None provided during exam:

Lesson Plan 2621.828.0.0046, Standby Liquid Control Learning SLC-10453, Explain or describe how this system is Objecti~rel interrelated with other plant systems.

Question Source (New, Modified, Bank) I Modified If Bank or Modified:

VISION S)rstem/Question ID 506610 Question Source SLC-4 Memory or X Comprehension Fundamental Cognitive 1 :1 or Analysis Knowledge Level NUREG 1021 Appendix B: Interlocks, setpoints, or system (singular) response 55.41 I 7 I 55.43 I 10CRF55 Design, components, and functions of control and Content safety systems, including instrumentation, signals, interlocks, failure modes, and automatic and manual features.

Justification for LORT que!stions with N/A KIA values < 3.0 Time to C1omplete: 1-2 minutes I Point Value: 1 System ID No.: 211000 PRA: I NO Safety ~ Initial License Level 1

Functi()n: D LORT ILT 10-1 NRC & AUDIT EXAM Page: 52 of 218 26 May 2011

EXAMINATION ANSWER KEY ILT 10-1 NRC RO Exam 19 10: 10-1 NR019 Points: 1.00 The plant is at rated power. Which of the following indication(s) below PROCEOURALLY REQUIRE entry into ABN-1, Reactor Scram?

A. REACTOR LEVEL NARROW RANGE YARWAY GEMAC B

~180

,IIO!

...... 100 l

170 110 fleo fleo I ~,oo I ~11'50 N i- N '

C 1-140 C ~NO H  !- H r-E s ~130 T -no A

~120 F ,..... 120 T ~

f; A

F ~110 fl00 GO ILT 10-1 NRC & AUDIT EXAM Page: 53 of 218 26 May 2011

EXAMINATION ANSWER KEY ILT 10-1 NRC RO Exam B.

CONDENSER VACUUM ILT 10-1 NRC & AUDIT EXAM Page: 54 of 218 26 May 2011

EXAMINATION ANSWER KEY ILT 10-1 NRC RO Exam C.

D.

Answer: C IAnswer Explanation ILT 10-1 NRC & AUDIT EXAM Page: 55 of 218 26 May 2011

EXAMINATION ANSWER KEY ILT 10-1 NRC RO Exam QID: 10-1 NR019 Question # I 19 I Developer / Date: JJR /7-11-11 Knowledge and Ability Reference Information Importance Rating K&A RO SRO 300000 Instrument Air A4.01 - Ability to manually operate and/or 2.6 2.7 monitor in the control room: Pressure gauges Level I RO I Tier 2 I Group I 1 General ABN-35 References C is Correct. lAW ABN-35, Loss of Instrument Air, when Instrument Air Supply pressure on PT-3 (Panel 7F) indicates < 55 psig, enter ABN-1 and manually scram the reactor.

A is Incorrect. This distractor is plausible if the applicant believes the Main Turbine will trip at 170 inches. ABN-1 will have to be entered after the turbine trip. Actual turbine trip setpoint is 175 inches TAF.

Explanation B is Incorrect. This distractor is plausible if the applicant believes the reactor will trip on a low vacuum pressure of 25 in Hg. ABN*1 will require to be entered then. Actual low vacuum turbine/reactor trip setpoint is 22 in Hg.

D is Incorrect. This distractor is plausible if the applicant believes that a manual scram and entry into ABN*1 is required at 90F Torus temperature. A plant shutdown is required at 95F.

References to be None provided durin ~ exam:

Lesson Plan 2621.828.0.0043, Service, Instrument, and Breathing Air.

Learning Objective/ CAS-10445, Given a set of system indications or data, evaluate and interpret them to determine limits, trends and system status.

I Question Source (New, Modified, Bank) New ILT 10-1 NRC & AUDIT EXAM Page: 56 of 218 26 May 2011

EXAMINATION ANSWER KEY ILT 10-1 NRC RO Exam If Bank or Modified: N/A VISION Sl,stem/Question 10 Question Source Memory or X Comprehension Fundamental 3:SPK Cognitive or Analysis Knowledge Level NUREG 1021 Appendix B: Solve a Problem using Knowledge and its meaning 10CRF55 55.41 I 10 I 55.43 I Content Administrative, normal, abnormal, and emergency operating procedures for the facility.

Justification for LORT questions with N/A KIA valuels < 3.0 Time to C,omplete: 1-2 minutes I Point Value: 1 System I[) No.: 300000 PRA: I NO Safety ~ Initial License Level 8

Functic)n: D LORT ILT 10-1 NRC & AUDIT EXAM Page: 57 of 218 26 May 2011

EXAMINATION ANSWER KEY ILT 10-1 NRC RO Exam 20 10: 10-1 NR020 Points: 1.00 The plant is at power when the following alarms and indications were noted:

  • BOTH CHARGER C1 AND CHARGER C2 indicate 0 AMPS
  • BATT C indicates 0 AMPS Which of thl3 following components can still be operated from the Control Room?

A. CRD Pump A B. Feedwater Pump B C. Core Spray Main Pump NZ01 D D. Containment Spray Pump 51 B Answer: B Answer Explanation I QID: 10-1 NR020 Question # I 20 I Developer 1 Date: JJR 17-11-11 Knowledge and Ability Reference Information Importance Rating K&A RO SRO 263000 DC Electrical Distribution A4.01 - Ability to manually operate and/or 3.3 3.5 monitor ira the control room: Major breakers and contrlol power fuses Level I RO I Tier 2 I Group I 1 General 3001B 3033 3002 References 3001C ILT 10-1 NRC & AUDIT EXAM Page: 58 of 218 26 May 2011

EXAMINATION ANSWER KEY ILT 10-1 NRC RO Exam B is Correct. The question stem shows a low voltage condition on 125 VDC Bus C. It also shows that both chargers to Bus C show no current and that the C Battery also shows no current. These conditions depict a fault on 125 VDC Bus C and that both the chargers and the battery are disconnected from Bus C. DC Bus C provides the DC control power for remote breaker operation for 4160 VAC Bus 1A, 4160 VAC Bus 1C, 460 VAC Busses 1A1, 1A2, and 1A3. With breaker control power gone, there is no longer remote control of the breakers on the AC busses from the control room. Feedwater Pump B is powered from Bus 1B, and thus has DC power.

Explanation A is Incorrect. This distractor is plausible if the applicant does not recall that CRD Pump A is powered from USS Bus 1A2, and thus has no DC power.

C is Incorrect. This distractor is plausible if the applicant does not recall that Core spray main pump NZ01 D is powered from Bus 1C, and thus has no DC power.

D is Incorrect. This distractor is plausible if the applicant does not recall that Containment Spray Pump 51 B is powered from USS 1A2, and thus has no DC power.

References to be None provided durin~ exam:

Lesson Plan 2621.828.0.0017, Feed and Condensate System Learning CFW-10453, Explain or describe how this system is Objective/ interrelated with other plant systems.

Question Source (New, Modified, Bank) I Bank If Bank or Modified:

VISION System/Question ID 663320 / CFW-IRH-001 Question Source ILT 08-1 NRC Exam Memory or X Comprehension Fundamental 3:SPK Cognitive or Analysis Knowledge Level NUREG 1021 Appendix B: S.olve a ~roblem using Knowledge and its meaning ILT 10-1 NRC & AUDIT EXAM Page: 59 of 218 26 May 2011

EXAMINATION ANSWER KEY ILT 10-1 NRC RO Exam 10CRF55 55.41 I 7 I 55.43 I Content Administrative, normal, abnormal, and emergency operating procedures for the facility.

Justification for LORT questions with N/A KIA values < 3.0 Time to Complete: 1-2 minutes I Point Value: 1 System ID No.: 263000 PRA: I NO Safety [gJ Initial License Level 6

Function: D LORT ILT 10-1 NRC & AUDIT EXAM Page: 60 of 218 26 May 2011

EXAMINATION ANSWER KEY ILT 10-1 NRC RO Exam 21 ID: 10-1 NR021 Points: 1.00 The plant is at rated power when a MAJOR FIRE in the Control Room erupted. All personnel were evacuated. NO actions required by ABN-30, Control Room Evacuation, have been completed.

lAW ABN-30, which of the following local 'Backup Method' actions must be completed to scram the reactor and close the MSIVs? In addition. what location are these actions taking place?

Actions Required Location A. -Trip the RPS MG supply breakers Old Cable

-Trip the supply breakers to PS-1 Spreading Room B. -Trip the RPS MG output circuit breakers 480V Room

-Tum off SW-733-169

-Tum off SW-733-170 C. -Trip the RPS MG output circuit breakers Old Cable

-Tum off SW-733-169 Spreading Room

-Tum off SW-733-170 D. -Trip the RPS MG supply breakers 480V Room

-Trip the supply breakers to PS-1 Answer: D Answer Explanation I QID: 10-1 NR021 Question # I 21 I Developer / Date: JJR / 7*11*11 Knowledge and Ability Reference Information Importance Rating K&A RO SRO 223002 PelS/Nuclear Steam Supply Shutoff 2.1.30 - Conduct of Operations: Ability to 4.4 4.0 locate and operate components, Including local controls.

Level I RO I Tier I 2 I Group I 1 ILT 10-1 NRC & AUDIT EXAM Page: 61 of 218 26 May 2011

EXAMINATION ANSWER KEY ILT 10-1 NRC RO Exam Generall ABN-30 References Att. ABN-30-1 D is Correct. lAW ABN-30, Control Room Evacuation, the Backup Method to scram the reactor and close the MSIVs when a fire in the Control Room exists is Method 1. Locally scram the reactor and close the MSIVs by 1) Tripping the RPS MG supply breakers, and 2) Tripping the supply breaker to the PS-1 MTS from the 480V Room.

ExplanaUon A is Incorrect. This distractor is plausible if the applicant does not recall the location of these components..

B & C are Incorrect. These distractors are plausible since these are alternate actions to scram and close MSIVs, however they are only performed if there is NOT a fire in the Control Room. Since there is, these actions are not an option lAW ABN-30.

References to be None provided during exam:

Lesson Plan 2621.828.0.0030, Nuclear Steam Supply System Learning NSS-3957, List the automatic actions which occur Objectivel when the MSIVs close (automatic and manual closure).

Question Source (New, Modified, Bank) I New If Bank or Modified: N/A VISION S3(stem/Question ID Question Source Memory or X Comprehension Cognitive Fundamental 1:F or Analysis Level Knowledge NUREG 1021 Appendix B: Facts 10CRF55i 55.41 I 10 I 55.43 I Content Administrative, normal, abnormal, and emergency operating procedures for the facility.

Justification for LORT questions with N/A KIA values < 3.0 Time to Complete: 1-2 minutes I Point Value: 1 System ID No.: I 223002 I PRA: I NO ILT10-1 NRC & AUDIT EXAM Page: 62 of 218 26 May 2011

EXAMINATION ANSWER KEY ILT 10-1 NRC RO Exam Safety [gl Initial License Level 5

Function: o LORT ILT 10-1 NRC & AUDIT EXAM Page: 63 of 218 26 May 2011

EXAMINATION ANSWER KEY ILT 10-1 NRC RO Exam 22 10: 10-1 NR022 Points: 1.00 The plant was at rated power. An event then occurred and plant conditions now include the followin~~:

H&V OW RXBLDG TURB BLDG RECIRC FANS SGTS ILT 10-1 NRC & AUDIT EXAM Page: 64 of 218 26 May 2011

EXAMINATION ANSWER KEY ILT 10-1 NRC RO Exam Based SOLELY on the indications provided, which Standby Gas Treatment System (SGTS) is running AND state whether a Secondary Containment Control (SCC) EOP entry condition has been exceeded?

SGTS System Running sec EOP Entry Exceeded A. SGTS 1 No B. SGTS 1 Yes C. SGTS2 Yes D. SGTS2 No Answer: B Answer Explanation I QID: 10-1 NR022 Question # I 22 I Developer J Date: JJR J 7-11-11 Knowledge and Ability Reference Information Importance Rating K&A RO SRO 261000 SGTS 2.4.31 - El1nergency Procedures I Plan:

4.2 4.1 Knowledge of annunciator alarms, indications, or response procedures.

Level I RO I Tier 2 I Group I 1 General EOP User's RAP-L1b SeeEOP References Guide RAP-L5b ILT 1()"1 NRC & AUDIT EXAM Page: 65 of 218 26 May 2011

EXAMINATION ANSWER KEY ILT 10-1 NRC RO Exam B is Correct. The question stem provides the operator indications that SGTS 1 is running, SGTS 2 has shutdown, and RB dP is > 0 in which requires entry into the SCC EOP.

A is Incorrect. This distractor is plausible if the applicant does not recognize that a SCC EOP entry condition has been exceeded on RB dP being> 0 in.

C is Incorrect. This distractor is plausible if the Explanation applicant does not recognize from the annunciators in alarm that SGTS 1 is running and SGTS 2 has shutdown.

o is Incorrect. This distractor is plausible if the applicant does not recognize from the annunciators in alarm that SGTS 1 is running and SGTS 2 has shutdown. In addition, this distractor is plausible if the applicant does not recognize that a SCC EOP entry condition has been exceeded on RB AP being

> 0 in.

References to be None provided durin~ exam:

Lesson Plan 2621.845.0.0057, Secondary Containment Control Learning SCC-1667, Based upon specific plant parameters Objective/ and conditions, determine if entry conditions for EOPs have been met and which EOPs are applicable to the conditions provided.

Question Source (New, Modified, Bank) I New If Bank or Modified: N/A VISION System/Question 10 Question Source Memory or X Comprehension Fundamental 2:DR Cognitive or Analysis Knowledge Level NUREG 1021 Appendix B: Describing or recognizing Relationships 55.41 I 7 I 55.43 I 10CRF55 Design, components, and functions of control and Content safety systems, including instrumentation, signals, interlocks, failure modes, and automatic and manual features.

ILT 10-1 NRC & AUDIT EXAM Page: 66 of 218 26 May 2011

EXAMINATION ANSWER KEY ILT 10-1 NRC RO Exam Justification for LORT questions with N/A KIA values < 3.0 Time to Complete: 1-2 minutes I Point Value: 1 System 10 No.: 261000 PRA: I NO Safety ~ Initial License Level 9

Function: D LORT ILT 10-1 NRC & AUDIT EXAM Page: 67 of 218 26 May 2011

EXAMINATION ANSWER KEY ILT 10-1 NRC RO Exam 23 ID: 10*1 NR023 Points: 1.00 The plant was at rated power when an A TWS occurred. The Operator placed the STANDBY LIQUID CONTROL keylock switch to the FIRE SYS 2 position.

If ONLY ONE of the Standby Liquid Control System 2 squibs actuated, which of the following states the impact on the SLC System's ability to inject into the RPV?

SLC System 2 .....

A. will inject at the normal rate.

B. will NOT inject into the RPV.

C. will inject at % the normal rate.

D. will inject less than the normal rate, but> % the rate.

Answer: A Answer Explanation I QID: 10-1 NR023 Question # I 23 I Developer I Date: JJR 17-11-11 Knowledge and Ability Reference Information Importance Rating K&A RO SRO 211000 SLC K5.04 - Krlowledge of the operational implications of the following concepts as they 3.1 3.2 apply to STANDBY LIQUID CONTROL SYSTEM: Explosive valve operation Level I RO I Tier 2 I Group I 1 General 15786350 304 References Sh.188 ILT 1IJ..1 NRC & AUDIT EXAM Page: 68 of 218 26 May 2011

EXAMINATION ANSWER KEY ILT 10-1 NRC RO Exam A is Correct. Placing the keylock switch in SYS 2 will fire both System 2 squibs and start only the system SLC pump (Pump B). Only actuation of a single squib in System 2 needs to function to allow full SLC System flow.

Explanation B, C, and 0 are Incorrect. These distractors are plausible if the applicant does not recall that only one squib valve needs to fire for SLC to inject and develop full flow.

References to be None provided during exam:

Lesson Plan 2621.828.0.004, Standby Liquid Control System Learning SLC*10446, Identify and explain system operating Objective/ controls under all plant operating conditions.

Question Source (New, Modified, Bank) I Bank If Bank or Modified:

VISION System/Question 10 609049/IRL*21*28-0046/ PRT*10 Question Source ILT07-1 Audit Memory or X Comprehension Fundamental 1 :1 Cognitive or Analysis Knowledge Level NUREG 1021 Appendix B: Interlocks, setpoints, or system (singular) response 55.41 I 7 I 55.43 I 10CRF55 Design, components, and functions of control and Content safety systems, including instrumentation, signals, interlocks, failure modes, and automatic and manual features.

Justification for LORT questions with N/A KIA values < 3.0 Time to Complete: 1*2 minutes I Point Value: 1 System 10 No.: 211000 PRA: I NO Safety ~ Initial License Level 1

Function: D LORT ILT 10-1 NRC & AUDIT EXAM Page: 69 of 218 26 May 2011

EXAMINATION ANSWER KEY ILT 10-1 NRC RO Exam 24 10: 10-1 NR024 Points: 1.00 The plant was at rated power. An electrical transient resulted in the following annunciators on Pane19XF:

DC XFERS Which of the following describes the status of CIP-3?

CIP-3 is ...

A. DE-ENERGIZED.

B. ENERGIZED via VMCC-1A2.

C. ENERGIZED from the Rotary Inverter via DC-B.

D. ENERGIZED from the Rotary Inverter via VMCC-1 82.

Answer: 8 ILT 1()"1 NRC & AUDIT EXAM Page: 70 of 218 26 May 2011

EXAMINATION ANSWER KEY ILT 10-1 NRC RO Exam Answer Explanation I QID: 10-1 NR024 Question # I 24 I Developer/ Date: JJR/7-11-11 Knowledge and Ability Reference Information Importance Rating K&A RO SRO 262002 UPS (AC/DC)

A3.01 - Ability to monitor automatic operations of the UNINTERRUPTABLE POWER SUPPLY 2.8 3.1 (A.C.lD.C.) including: Transfer from preferred to alternate source Level I RO I Tier 2 I Group I 1 General BR 3013 RAP-9XF4c RAP-9XF6c References B is Correct. The question stem provides annunciators that indicate CIP-3 has transferred to its alternate power source ofVMCC-1A2. CIP-3 is normally powered from the Rotary Inverter which has a normal (AC from VMCC-1 B2) and backup (DC from DC-B) power supply. Both annunciators 9XF-4 c and 9XF-6-c indicate the Rotarty Inverter has lost all power. In this instance, Automatic Transfer Switch (ATS) IT-3 will automatically transfer, re powering CIP-3 from VMCC-1A2, its alternate source of power.

Explanation A is Incorrect. This distractor is plausible if the applicant misinterprets the annunciator indications and believes that CIP-3 is now de-energized.

C is Incorrect. This distractor is plausible if the applicant misinterprets the annunciator indications and believes that the Rotary Inverter just lost its AC power source (VMCC-1 B2).

D is Incorrect. This distractor is plausible if the applicant misinterprets the annunciator indications and believes that the Rotary Inverter just lost its DC power source (DC-B).

References to be None provided during exam:

ILT 10-1 NRC & AUDIT EXAM Page: 71 of 218 26 May 2011

EXAMINATION ANSWER KEY ILT 10-1 NRC RO Exam Lesson Plan 2621.828.0.0056, Vital AC Distribution System Learning VAC-10445, Given a set of system indications or Objectivel data, evaluate and interpret them to determine limits, trends and system status.

Question Source (New, Modified, Bank) I New If Bank or Modified: N/A VISION S~'stem/Question ID Question Source Memory or Comprehension X Fundamenta Cognitive or Analysis 3:SPK I Knowledge Level NUREG 1021 Appendix B: Solve a E.roblem using Knowledge and its meanin;J 55.41 I 7 55.43 I 10CRF55 Design, components, and functions of control and Content safety systems, including instrumentation, signals, interlocks, failure modes, and automatic and manual features.

Justification for LORT questions with N/A KIA values < 3.0 Time to Complete: 1-2 minutes I Point Value: 1 System ID No.: 262002 PRA: I NO Safety rgj Initial License Level 6

Function: D LORT lLT 10-1 NRC & AUDIT EXAM Page: 72 of 218 26 May 2011

EXAMINATION ANSWER KEY ILT 10-1 NRC RO Exam 25 10: 10-1 NR025 Points: 1.00 The plant was at rated power. The following indications for EDG-1 were observed for several minutes on Panel 8F/9F:

  • BREAKER OPEN light is LIT
  • BREAKER CLOSED light is OFF
  • UNIT IDLING light is LIT
  • UNIT START light is OFF Which of the following plant parameters would result in the above EDG-1 indications?

A. Drywell Pressure indicates 2.7 psig B. RPV Water Level indicates 95 inches C. 4160 VAC Bus 1C indicates 2500 Volts D. EDG-1 Lube Oil Temperature is 80° F Answer: D Answer Explanation I QID: 10-1 NR025 Question # I 25 I Developer / Date: JJR / 7-11-11 Knowledge and Ability Reference Information Importance Rating K&A RO SRO 264000 E[)Gs A1.01 - Ability to predict and/or monitor changes in parameters associated with 3.0 3.0 operating the EMERGENCY GENERATORS (DIESEUJIET) controls including: Lube oil temperatLire Level I RO I Tier 2 I Group I 1 Generall 341 3E-861-21-1002 Referencl9s ILT 10-1 NRC & AUDIT EXAM Page: 73 of 218 26 May 2011

EXAMINATION ANSWER KEY ILT 10-1 NRC RO Exam D is Correct. The question stem provides indications of an EDG-1 Idle Start. Conditions which will Idle Start and EDG are High Drywell Pressure

(>3psig, 2.9psig setpoint), RPV Lo-Lo Water Level

<<86",90" setpoint), and EDG Lo Lube Oil Temperature (~85F).

A is Incorrect. This distractor is plausible if the applicant does not recall the correct EDG Idle Start Explanation logic.

B is Incorrect. This distractor is plausible if the applicant does not recall the correct EDG Idle Start logic.

C is Incorrect. This distractor is plausible if the applicant does not recall the correct EDG Idle Start logic. It is true that the EDG will Fast Start, however during a Fast Start, the UNIT START light will be LIT, not OFF, as indicated in the question stem.

References to be None provided durin~ exam:

Lesson Plan 2621.828.0.0013, Emergency Diesel Generators Learning EDG-10444, Describe the interlock signals and Objective/ setpoints for the affected system components and expected system response including power loss or failed components.

Question Source (New, Modified, Bank) I New If Bank or Modified: N/A VISION System/Question ID Question Source Memory or X Comprehension Fundamental 3:SPK Cognitive or Analysis Knowledge Level NUREG 1021 Appendix B: S.olve a eroblem using Knowledge and its meaning ILT 10-1 NRC & AUDIT EXAM Page: 74 of 218 26 May 2011

EXAMINATION ANSWER KEY ILT 10-1 NRC RO Exam 55.41 I 5 I 55.43 I Facility operating characteristics during steady state 10CRF55 and transient conditions, including coolant chemistry, Content causes and effects of temperature, pressure and reactivity changes, effects of load changes, and operating limitations and reasons for these operating characteristics.

Justification for LORT questions with N/A KIA values < 3.0 Time to Complete: 1*2 minutes I Point Value: 1 System ID No.: 264000 PRA: I NO Safety [8J Initial License Level 6

Function: o LORT ILT 10-1 NRC & AUDIT EXAM Page: 75 of 218 26 May 2011

EXAMINATION ANSWER KEY ILT 10-1 NRC RO Exam 26 10: 10-1 NR026 Points: 1.00 At Time =0 seconds, the plant was at rated power when a LOCA occurred.

At Time = 30 seconds, the following conditions exist:

  • RPV Pressure indicates 500 psig and lowering
  • All EMRVs indicate GREEN light ON
  • Core Spray Booster Pumps C AND D indicate GREEN light ON At Time = 105 seconds, an operator placed BOTH ADS Timers in BYPASS.

At Time =107 seconds, how many EMRVs will indicate RED light ON?

A. 0 B. 2 C. 3 D. 5 Answer: A Answer Explanation I QID: 10-1 NR026 Question:# I 26 I Developer 1 Date: JJR 17-11-11 Knowledge and Ability Reference Information Importance Rating K&A RO SRO 218000 ADS 2.4.47 - Emergency Procedures I Plan: Ability to diagnose and recognize trends in an 4.2 4.2 accurate and timely manner utilizing the appropriat~1t control room reference material.

Level I RO I Tier I 2 I Group I 1 ILT 10-1 NRC & AUDIT EXAM Page: 76 of 218 26 May 2011

EXAMINATION ANSWER KEY ILT 10-1 NRC RO Exam General RAP-Big ADS Lesson 729E182 References RAP*B4g Plan A is Correct. The plant was at power when a LOCA occurred. With all ADS TIMER START annunciators alarming, then the logic to open the EMRVs has been satisfied and a 105 second timer is counting down.

When the times makes it to 0, then 2 EMRVs open immediately, 1 EMRV will open at 1.5 seconds later, and 2 EMRVs will open at the 3.25 second later mark.

=

At Time 107 seconds, the ADS timers have only counted down for 77 seconds, therefore NO EMRVs will be open. This question tests the KIA and examines the applicant's ability to use procedures in a timely manner in order to inhibit ADS lAW EOPs and examines their knowledge of trending by testing their knowledge on how long they have before EMRVs open when ADS logic is satisfied.

Explanaticm B is Incorrect. This distractor is plausible if the applicant does not add 30 seconds to the 105 second time delay and believes only 2 EMRVs will be open.

C is Incorrect. This distractor is plausible if the applicant does not add 30 seconds to the 105 second time delay since 3 EMRVs will be open 107 seconds after ADS Timers have started counting down.

D is Incorrect. 'rhis distractor is plausible if the applicant does not recall that the EMRVs open on a staggered time delay and believes all EMRVs will be open.

References to be None provided durin~ exam:

Lesson Plan 2621.828.0.0005, Automatic Depressurization System Learnin!g Objectiv1e/ ADS-379, Describe the operation of the ADS controls including: 1) Resetting ADS timers; 2)

Bypassing ADS timers; 3) Disabling ADS; 4)

Clearing and resetting ADS auto initiation signals;

5) Removal of ADS control logic fuses to close EMRVs; 6) Obtaining readings from EMRV temperature indicators ILT 10-1 NRC & AUDIT EXAM Page: 77 of 218 26 May 2011

EXAMINATION ANSWER KEY ILT 10-1 NRC RO Exam Question Source (New, Modified, Bank) I Modified If Bank or Modified:

VISION System/Question 10 663297/ AOS..IRH-001 Question Source ILT 08-1 NRC RO#9 Memory or . X Fundamental Comprehen~lon 3:PEO Cognitive Knowledge or AnalysIs Level ~ 1n?1 4-!"pendix B: ~redict an Event or 10CRF55 --------P-------~--------~------~

55.41 I 10 I 55.43 I Content Administrative, normal, abnormal, and emergency operating procedures for the facility.

Justification for LORT qU8!stions with N/A KIA valuels < 3.0 Time to Clomplete: 1-2 minutes I Point Value: 1 System ID No.: 218000 PRA: I NO Safety ~ Initial License Level 3

Functicm: D LORT ILT 10*1 NRC & AUDIT EXAM Page: 78 of 218 26 May 2011

EXAMINATION ANSWER KEY ILT 10-1 NRC RO Exam 27 ID: 10-1 NR027 Points: 1.00 Which of the following would REQUIRE entry into the Secondary Containment Control EQ.f.?

A. RB ~P LO annunciator is at the alarm setpoint B. B-7, TIP VALVE AREA, on Panel2R above MAX NORMAL C. Off-Site Radioactivity Release Rate above the ALERT level D. IB-13-A, TRUNNION ROOM EAST END RB ELEV 23 FT, indicates 140°F Answer: B Answer Explanation I QID: 10-1 NR027 Question # I 27 I Developer 1 Date: JJR 17-11-11 Knowledge and Ability Reference Information Importance Rating K&A RO SRO 215001 Traversing In-core Probe K1.10 - Krlowledge of the physical connectio,ns andlor cause- effect relationships 2.6 2.8 between 1rRAVERSING IN-CORE PROBE and the following: Area radiation monitoring system: (Not-BWR1)

Level I RO I Tier 2 I Group I 2 Generall EOP User's Referenc~es see EOP Guide RAP-L6c ILT 10-1 NRC & AUDIT EXAM Page: 79 of 218 26 May 2011

EXAMINATION ANSWER KEY ILT 10-1 NRC RO Exam B is Correct. lAW the Secondary Containment Control EOP, ARM B*7 above the Max Normal requires entry into Secondary Containment Control.

"rhis ARM is installed specifically to determine radiation abnormalities associated with the TIP system. The indication for this alarm is on Panel 2R in the Control Room.

A is Incorrect. The RB 4P LO annunciator alarms at

- 0.14" H20. The EOP entry into SCC is ~ 0" H20.

This distractor is plausible if the applicant does not recall the alarm setpoint is slightly less than the EOP Explanation setpoint in order to give the operator time to restore RB 4P before entry into SCC is required.

C is Incorrect. This is an entry into Radioactivity Release Control (which is contained on the same flowchart printout as SCC). This distractor is plausible if the applicant is confused by which EOP this condition requires entry into.

D is Incorrect. The Trunnion Room East temperature EOP entry is at 160°F. This distractor is plausible if the applicant does not recall the Max Normal setpoint for this temperature indication.

References to be None provided durin", exam:

Lesson Plan 2621.828.0.033A, Plant Radiation Monitoring System Learning Objective/ RAD*3025, Given key plant parameters, determine if entry conditions for the EOPs have been met and which, if any, EOP should be entered frist for these conditions.

Question Source (New, Modified, Bank) I New If Bank or Modified: N/A VISION System/Question ID Question Source Memory or X Comprehension Fundamental Cognitive 1:P or Analysis Knowledge Level NUREG 1021 Appendix B: erocedure steps and cautions ILT 1()"1 NRC & AUDIT EXAM Page: 80 of 218 26 May 2011

EXAMINATION ANSWER KEY ILT 10-1 NRC RO Exam 10CRF55 55.41 I 11 I 55.43 I Content Purpose and operation of radiation monitoring systems, including alarms and survey equipment.

Justification for LORT questions with N/A KIA values < 3.0 Time to Complete: 1-2 minutes I Point Value: 1 System 1[' No.: 215001 PRA: I NO Safety 7

121 Initial License Level Functic)O: o LORT ILT 10-1 NRC & AUDIT EXAM Page: 81 of 218 26 May 2011

EXAMINATION ANSWER KEY ILT 10-1 NRC RO Exam 28 10: 10-1 NR028 Points: 1.00 A loss of which of the following power supplies will render the Alternate Rod Injection (ARI) System INOPERABLE?

A. DC-E B. DC-2 C. VACP-1 D. PAIPP-1 Answer: A Answer Explanation I QID: 10..1 NR028 Question # I 28 I Developer 1 Date: JJR 17-11-11 Knowledge and Ability Reference Information Importance Rating K&A RO SRO 201001 CRD Hydraulic K2.05 .. Knowledge of electrical power 4.5 4.5 supplies to the following: Alternate rod insertion valve solenoids: Plant-Specific level I RO I Tier 2 I Group I 2 General BR E0578 ABN-53 References A is Correct. The power supply to the ARI system valve solenoids is DC-E. ARI initiation logic is energize to activate, therefore it requires power from DC-E or it will not operate.

All distractors are Incorrect but plausible sources of Explanation vital power.

NOTE: This question was changed to low Cog from High Cog due to the maximum number of High Cog questions on the RO exam (45 max allowed by NUREG .. 1021).

References to be None provided during exam:

ILT 10-1 NRC & AUDIT EXAM Page: 82 of 218 26 May 2011

EXAMINATION ANSWER KEY ILT 10-1 NRC RO Exam Lesson Plan 2621.828.0.0011, Control Rod Drive Hydraulic I

System Learning Objectivel CRD-2010, Describe the initiation logic for the Alternate Rod Injection (ARI) System including signals and setpoints.

Question Source (New, Modified, Bank) I New If Bank or Modified: N/A VISION System/Question 10 Question Source Memory or Fundamental X Comprehension Cognitive 1:F or Analysis Level Knowledge NUREG 1021 Appendix B: facts 55.41 I 7 I 55.43 I 10CRF55 Design, components, and functions of control and Content safety systems, including instrumentation, signals, interlocks, failure modes, and automatic and manual features.

Justification for LORT questions with N/A KIA values < 3.0 Time to Complete: 1..2 minutes I Point Value: 1 System 10 No.: 201001 PRA: I NO Safety 1

IZl Initial License Level Functicln: o LORT ILT 10-1 NRC & AUDIT EXAM Page: 83 of 218 26 May 2011

EXAMINATION ANSWER KEY ILT 10-1 NRC RO Exam 29 ID: 10-1 NR029 Points: 1.00 The plant was at rated power when an event then occurred. 5 seconds later, plant conditions were observed to include the following:

  • Panel 4F RPS 1 SCRAM SOLENOIDS lights are lit
  • Panel 4F RPS 2 SCRAM SOLENOIDS lights are lit
  • APRM power indicates 92% and lowering
  • Torus temperature indicates 90°F and rising
  • Drywell Pressure indicates 14 psig and rising
  • RPV water level indicates 178 inches and rising Based on these conditions, which of the following are in service and controlling RPV pressure? (Assume NO operator actions had been taken following the event)
1. EMRVs
2. Safety Valves
3. Isolation Condensers A. 1 ONLY B. 1 and 2 ONLY C. 1 and 3 ONLY D. 1,2, AND 3 Answer: D Answer Explanation I QID: 10-1 NR029 Question # I 29 I Developer 1 Date: JJR 17-11-11 Knowledge and Ability Reference Information Importance Rating K&A RO I SRO ILT 10-1 NRC & AUDIT EXAM Page: 84of218 26 May 2011

EXAMINATION ANSWER KEY ILT 10-1 NRC RO Exam 239001 Main and Reheat Steam K3.16 - Knowledge of the effect that a loss or malfunction of the MAIN AND REHEAT STEAM 3.6 3.6 SYSTEM will have on following: Relief/safety valves Level I RO I Tier 2 I Group I 2 General 420 BR2002 References D is Correct. The question stem provides indications of an MSIV isolation (loss of the Main Steam System) combined with an Electrical ATWS.

With an RPV Isolation and reactor power still at 92%

power, RPV Pressure is being controlled by the ICs, EMRVs, and Safety Valves. ICs can remove a combined 6% steam demand, all 5 EMRVs remove 40% steam flow (to the Torus), and the remaining 46% steam demand is being discharged to the Drywell air space (Drywell pressure rising). This question matches the KIA by testing the response of Explanation the Relief/Safety valves during a loss of the Main Steam System in an ATWS condition.

A, B, and C are Incorrect. These distractors are plausible if the applicant does recognize from the stem indications that both ICs and Safety valves are also in operation. ICs are not supposed to be manually initiated> 160". In this instance they would have automatically initiated. EMRVs are in every answer choice since it is low difficulty that the applicant would recognize that at least EMRVs would be in operation.

References to be None provided durin ~ exam:

Lesson Plan 2621.828.0.0026, Main Steam System Learning MSS-10453, Explain or describe how this system is Objective/ interrelated with other plant systems.

Question Source (New, Modified, Bank) I New If Bank or Modified: N/A VISION System/Question ID Question Source ILT 10-1 NRC & AUDIT EXAM Page: 85 of 218 26 May 2011

EXAMINATION ANSWER KEY ILT 10-1 NRC RO Exam Memory or X Comprehension Fundamental 2:RI or Analysis Cognitive Knowledge Level NUREG 1021 Appendix B: Recognizing Interaction between systems (plural), including consequences and implications 55.41 I 7 I 55.43 I 10CRF55 Design, components, and functions of control and Content safety systems, including instrumentation, signals, interlocks, failure modes, and automatic and manual features.

Justification for LORT questions with N/A KIA values < 3.0 Time to Complete: 1-2 minutes I Point Value: 1 System 10 No.: 239001 PRA: I NO Safe~v ~ Initial License Level 3

Functicm: o LORT ILT 10-1 NRC & AUDIT EXAM Page: 86 of 218 26 May 2011

EXAMINATION ANSWER KEY IlT 10-1 NRC RO Exam 30 ID: 10..1 NR030 Points: 1.00 A reactor startup is in progress. Control Rod 34-51 is being withdrawn to position 48.

Upon reaching position 48 the following annunciator came into alarm:

1 2 3 4 YB A. 1 B. 2 C. 3 D. 4 ILT 10-1 NRC & AUDIT EXAM Page: 87 of 218 26 May 2011

EXAMINATION ANSWER KEY ILT 10*1 NRC RO Exam Answer: B Answer Explanation I QID: 10-1 NR030 Question # I 30 I Developer / Date: JJR /7-11-11 Knowledge and Ability Reference Information Importance Rating K&A RO SRO 201003 Control Rod and Drive Mechanism K4.02 - Knowledge of CONTROL ROD AND DRIVE MECHANISM design feature(s) and/or 3.8 3.9 interlocks which provide for the following:

Detection of an uncoupled rod Level I RO I Tier 2 I Group I 2 General 302.2 References B is Correct. lAW 302.2, Control Rod Drive Manual Control System, if a control rod became uncoupled, the rod position display (on Panel 4F) will go dark (black with no position indication) and the ROD OVERTRAVEL alarm (H5a) will annunciate. These Explanation design features are what the applicant will use to detect if an uncoupled control rod condition exists.

All distractors are incorrect but plausible since they are control rod display indications under conditions other than an uncoupled rod.

References to be None provided during exam:

Lesson Plan 2621.828.0.0011, Control Rod Drive and Hydraulics Learning CRD..10460, Discribe the CRDM design features Objective/ and/or interlocks which provide for the detection of an uncoupled control rod.

Question Source (New, Modified, Bank) I New If Bank or Modified: N/A VISION System/Question ID Question Source ILT 10-1 NRC & AUDIT EXAM Page: 88 of 218 26 May 2011

EXAMINATION ANSWER KEY ILT 10-1 NRC RO Exam Memory or X Comprehension Fundamental 3:SPK Cognitive or Analysis Knowledge Level NUREG 1021 Appendix B: Solve a Problem using Knowledge and its meaning 55.41 I 7 I 55.43 I 10CRF55 Design, components, and functions of control and Content safety systems, including instrumentation, signals, interlocks, failure modes, and automatic and manual features.

Justification for LORT questions with N/A KIA values < 3.0 Time to Complete: 1-2 minutes I Point Value: 1 System ID No.: 201003 PRA: I NO Safety I2?J Initial License Level 1

Function: D LORT ILT 10-1 NRC & AUDIT EXAM Page: 89 of 218 26 May 2011

EXAMINATION ANSWER KEY ILT 10-1 NRC RO Exam 31 10: 10-1 NR031 Points: 1.00 The plant was at power when Reactor Recirc Pump 'A' began to coast down and trip.

Which of th~~ following would cause this event? (Assume all choices below reference the 'A' Recirc Pump MG Set)

A. A loss of speed control signal to the Bailey Scoop Tube Positioner.

B. A loss of the instrument air signal to the Bailey Scoop Tube Positioner.

C. A complete speed feedback signal failure to the MG Set voltage regulator.

D. A loss of power to the Recirc Pump MG Set Moore Controller on Panel 3F.

Answer: C Answer Explanation I QID: 10-1 NR031 Question # I 31 I Developer 1 Date: JJR 17-11-11 Knowledge and Ability Reference Information Importance Rating K&A RO SRO 202002 Recirculation Flow Control K5.02

  • Knowledge of the operational implications of the following concepts as they 2.6 2.6 apply to RECIRCULA'"ION FLOW CONTROL SYSTEM : Feedback signals Level I RO I Tier 2 I Group I 2 General RFC Lesson GE 148F961 References Plan ILT 10-1 NRC & AUDIT EXAM Page: 90 of 218 26 May 2011

EXAMINATION ANSWER KEY ILT 10-1 NRC RO Exam C is Correct. The MG Set Tachometer provides a feedback signal to the MG Set Voltage Regulator. A complete loss of the Speed Feedback signal to the MG Set Voltage Regulator will result in the MG Set Main Exciter amps lowering to the point where the MG Field Breaker will trip on undervoltage.

A is Incorrect. This will result in a Scoop Tube lockup. This distractor is plausible if the applicant does not recognize what happens to the Recirc Pump on a loss of speed control signal from DFRCS to the Bailey Scoop Tube Positioner.

Explanation B is Incorrect. This will result in a Scoop Tube lockup. This distractor is plausible if the applicant does not recognize what happens to the Recirc Pump on a loss of air signal from DFRCS to the Bailey Scoop Tube Positioner.

D is Incorrect. This will result in the automatic transfer to Local..Manual control of the MG Set. This distractor is plausible if the applicant believes the Recirc Pump will trip on a loss of power to its Moore controller.

References to be None provided durin", exam:

Lesson Plan 2621.828.0.0040, Recirc Flow Control System Learning RFC..158, Describe the following components Objectivel associated with the Recirc Flow Control System, including location, purpose, construction, operation, and power supply: 1) Tachometer, 2)

Fluid Coupler, 3) Bailer Positioner, 4) Air Failure Brake, 5) Individual Flow Controller, 6) Master Flow Controller, 7) Transfer of Control Logic, 8) Digital Control Computers, 9) MG Set Drive Motor, 10) MG Set Variable Speed Generator, 11) DC Exciter Question Source (New, Modified, Bank) I New If Bank or Modified: N/A VISION System/Question ID Question Source Memory or X Cognitive Comprehension Fundamental 2:DR Level or Analysis Knowledge ILT10-1 NRC & AUDIT EXAM Page: 91 of 218 26 May 2011

EXAMINATION ANSWER KEY ILT 10-1 NRC RO Exam NUREG 1021 Appendix B: Describing or recognizing Relationships 55.41 I 5 I 55.43 I Facility operating characteristics during steady state 10CRF55 and transient conditions, including coolant chemistry, Content causes and effects of temperature, pressure and reactivity changes, effects of load changes, and operating limitations and reasons for these operating characteristics.

Justification for LORT questions with N/A KIA values < 3.0 Time to Cc,mplete: 1-2 minutes I Point Value: 1 System ID No.: 202002 PRA: I NO Safety [gI Initial License Level 1

Function: o LORT ILT 10-1 NRC & AUDIT EXAM Page: 92 of 218 26 May 2011

EXAMINATION ANSWER KEY ILT 10-1 NRC RO Exam 32 ID: 10-1 NR032 Points: 1.00 The plant is at rated power. Panel 7F indications include the following:

MPR R:ELAY EPR RELAY The steam sensing line to the EPR then breaks. What is the effect on Turbine Control Valves (TCVs) AND RPV Pressure from the break?

TCYs will ... RPV Pressure wilL.

A. open LOWER to 890 psig B. close RISE until the MPR takes control C. open LOWER until the MPR takes control D. close RISE to the RPS high pressure scram setpoint Answer: B ILT10-1 NRC & AUDIT EXAM Page: 93 of 218 26 May 2011

EXAMINATION ANSWER KEY ILT 10-1 NRC RO Exam Answer Explanation I QID: 10-1 NR032 Question # I 32 I Developer 1 Date: JJR 17-11-11 Knowledge and Ability Reference Information Importance Rating K&A RO SRO 241000 ReactorlTurbine Pressure Regulating System K6.07 - Knowledge of the effect that a loss or 3.4 3.4 malfunction of the following will have on the REACTORITURBINE PRESSURE REGULATING SYS'rEM: Turbine inlet pressure Level I RO I Tier 2 I Group I 2 General ABN-9 ReferencEls B is Correct. The question stem provides indications of a loss of steam flow input to the EPR which is the same as turbine inlet pressure. This will tell the EPR that RPV pressure is lowering (which it is trying to remain constant). The EPR will then close TCVs which will raise RPV Pressure.

When the EPR Relay Setpoint exceeds the MPR relay setpoint, the MPR will take control of the Turbine Control System.

A is Incorrect. This distractor is plausible if the applicant is confused on the operation of the Turbine Explanaticm Control System. The value of 890 psig is plausible since this is where steam header pressure will stabalize if the TCVs failed open (and it's the bottom EPR Relay setpoint indication on Panel 7F).

C is Incorrect. This distractor is plausible if the applicant does not recongize this malfunction will result in TCVs closing to raise RPV pressure.

D is Incorrect. This distractor is plausible if the applicant does not believe the MPR will take control of RPV pressure before reaching the high RPV pressure scram setpoint.

References to be None provided during exam:

ILT 10-1 NRC & AUDIT EXAM Page: 94of218 26 May 2011

EXAMINATION ANSWER KEY ILT 10-1 NRC RO Exam Lesson Plan 2621.828.0.0051, Turbine Controls Learning TCS-10441 , Given the system logic/electrical Objective/ drawings, describe the system trip signals, setpoints and expected system response including power loss or failed components.

Question Source (New, Modified, Bank) I Modified If Bank or Modified:

VISION System/Question 10 507200 Question Source 448 (old OC LORT Bank)

Memory or X Comprehension Fundamental 3:PEO Cognitive or Analysis Knowledge Level NUREG 1021 Appendix B: eredict an Event or

.outcome 55.41 I 7 I 55.43 I 10CRF55 Design, components, and functions of control and Content safety systems, including instrumentation, signals, interlocks, failure modes, and automatic and manual features.

Justificati.)n for LORT quelstions with N/A KIA value!i < 3.0 Time to Complete: 1-2 minutes I Point Value: 1 System ID No.: 241000 PRA: I NO Safety ~ Initial License Level 3

Function: o LORT ILT 10-1 NRC & AUDIT EXAM Page: 95 of 218 26 May 2011

EXAMINATION ANSWER KEY ILT 10-1 NRC RO Exam 33 10: 10-1 NR033 Points: 1.00 The plant was at rated power when an event resulted in a high Drywell pressure condition. Present plant conditions are as follows:

  • LKOUT RELAY 86/S 1B TRI P Which of thj3 following states the impact on the operating Containment Spray/ESW Pumps?

Containment SprayLESW B Containment SprayLESW 0 A.

  • Both pumps trip
  • Both pumps remain
  • Both pumps can be running immediately restarted B.
  • Both pumps remain
  • Both pumps trip nJnning
  • ESW Pump can NOT be restarted C.
  • Both pumps trip
  • Both pumps trip
  • Both pumps can be Pump can be restarted immediately restarted after 200 seconds
  • ESW Pump can NOT be resta rted D.
  • Both pumps remain
  • Both pumps trip running
  • Both pumps can be restarted after 200 seconds Answer: D ILT 10-1 NRC & AUDIT EXAM Page: 96 of 218 26 May 2011

EXAMINATION ANSWER KEY ILT 10-1 NRC RO Exam Answer Explanation I QID: 10-1 NR033 Question # I 33 I Developer / Date: JJR / 7-11-11 Knowledge and Ability Reference Information Importance Rating K&A RO SRO 219000 RHRlLPCI: Torus/Suppression Pool Cooling Mode A1.07* Ability to predict and/or monitor changes in parameters associated with 3.2 3.3 operating 'the RHRlLPCI:

TORUS/SUPPRESSION POOL COOLING MODE conltrols including: Emergency generator loading Level I RO I Tier 2 I Group I 2 General 341 RAP-Sic 237E901 References ILT 1()"1 NRC & AUDIT EXAM Page: 97 of 218 26 May 2011

EXAMINATION ANSWER KEY ILT 10-1 NRC RO Exam D is Correct. The plant was at rated power when an event occurred resulting in a high Drywell pressure condition. B Containment Spray Loop is in the Drywell Spray mode, and D Containment Spray Loop is in the Torus cooling mode. The annunciator provided shows that startup transformer S1 B has tripped. It had been supplying 4160 Bus 1B, and Bus 1D (and Bus 1B2). When the transformer trips, Bus 1B becomes de-energized and EDG 2 starts and loads onto Bus 1D. Thus, Bus 1B is de-energized and EDG 2 is supplying Bus 1D (and Bus 1B2).

Loop B pumps are powered from 4160 Bus 1C (ESW pump) and Bus 1A2 (from Bus 1C for the Containment Spray pump). Both these busses are still powered from startup transformer S1A and are unaffected by the loss of the other startup transformer. Therefore, the Loop B pumps remain running.

Explanation Loop D pumps are powered from 4160 Bus 1D (ESW pump) and Bus 1B2 (from Bus 1D for the Containment Spray pump). Both these busses were initially powered from startup transformer S1 B, which has been lost. Since Bus 1D has been re powered by the EDG, ESW Pump D has power. EDG 2 immediately started and loaded onto Bus 1D. But, to prevent EDG loading concerns, the manual start of any containment spray pump and ESW pump on Bus 1D is prevented for 200 seconds after the EDG picks up the bus.

Therefore: Loop B pumps remain running, Loop D ESW Pump trips and can be re-started in 200 seconds, and containment spray pump D trips and can be manually restarted after 200 seconds.

A, B, & C are Incorrect but plausible if the candidate does not know the meaning of the provided alarm, electrical distribution or the associated logic.

References to be None provided during exam:

ILT 10-1 NRC & AUDIT EXAM Page: 98 of 218 26 May 2011

EXAMINATION ANSWER KEY ILT 10-1 NRC RO Exam Lesson Plan 2621.828.0.0009, Containment Spray/ESW System Learning CNS-10446, Identify and explain system operating Objective/ controls/indications under all plant operating conditions.

Question Source (New, Modified, Bank) I Bank If Bank or Modified:

VISION System/Question 10 663341 / CNS-IRH-001 Question Source ILT 08*1 NRC RO EXAM Memory or X Comprehension Fundamental 2:RI or Analysis Cognitive Knowledge Level NUREG 1021 Appendix B: Recognizing Interaction between systems (plural), including consequences and implications 55.41 I 5 I 55.43 I Facility operating characteristics during steady state 10CRF55 and transient conditions, including coolant chemistry, Content causes and effects of temperature, pressure and reactivity changes, effects of load changes, and operating limitations and reasons for these operating characteristics.

Justification for LORT questions with N/A KIA values, < 3.0 Time to Complete: 1*2 minutes J Point Value: 1 System 10 No.: 219000 PRA: J NO Safety 5

L8l Initial License Level Function: D LORT ILT 10-1 NRC & AUDIT EXAM Page: 99 of 218 26 May 2011

EXAMINATION ANSWER KEY ILT 10-1 NRC RO Exam 34 10: 10-1 NR034 Points: 1.00 The plant was at rated power when a common mode failure causes all Reactor Recirc flow controllers to drop to minimum frequency. Current plant conditions include the following:

  • All APRM indications are cycling between 38 - 51 %
  • TOTAL RECIRC FLOW indicates 6.9 x 104 gpm Which of thE~ following actions is required NEXT and for what reason?

Action is to ... Reason is for...

A. manually scram the reactor. power oscillations.

B. insert rods or raise recirc flow. entering the Exclustion Zone.

C. manually scram the reactor. multiple Recirc Pump trip.

D. maintain a hightened entering the Buffer Zone.

awareness of Plant Parameters.

Answer: A Answer Explanation I QID: 10-1 NR034 Question # I 34 I Developer 1 Date: JJR 17-11-11 Knowledge and Ability Reference Information Importance Rating K&A RO SRO 202001 Recirculation A2.06 - Ability to (a) predict the impacts of the following 40n the RECIRCULA1'ION SYS'rEM ;

and (b) ba:sed on those predictions, use 3.6 3.8 procedures to correct, control, or mitigate the consequences of those abnormal conditions or operatic)ns: Inadvertent recirculation flow decrease Level I RO I Tier I 2 I Group I 2 ILT 10-1 NRC & AUDIT EXAM Page: 1000f218 26 May 2011

EXAMINATION ANSWER KEY ILT 10-1 NRC RO Exam General 202.1 References A is Correct. The question stem provides a condition where a Recirculation System failure resulted in Recirculation Flow lowering rapidly to minimum frequency (which is 11 Hz). With Reactor Power cycling between 38..51% (and at 45% avg) and Recirc Flow at 6.9x10E4, the plant is in the Buffer Zone however there are also power oscillations that resulted which are greater than +/- 5% on ~ 2 APRMs.

Procedure 202.1 then requires a reactor scram lAW ABN-1. Power oscillations are a major concern in high power low flow conditions and is the reason this is the first item an operator analyzes when recirculation flow lowers less than 8.5x10E4 gpm.

Explanatic~n B is Incorrect. This distractor is plausible if the applicant does recognize that power ocillations exist and believes the plant has entered the Exclusion Zone.

C is Incorrect. This distractor is plausible if the applicant believes conditions have been met for a loss of multiple Recirc Pumps and a scram is required lAW ABN-2, Recirc Flow Abnormalities.

o is Incorrect. This distractor is plausible if the applicant does not recognize power oscillations exist greater than that required to scram by procedure. It is true the Buffer Zone has been entered which also makes this distractor plausible.

References to be Attachment NOTE: This provided during exam: 202.1*2 reference is also provided for RO question #15 Lesson Plan 2621.828.0.0038, Reactor Recirculation System Learning RRS*10445, Given a set of system indications or Objective/ data, evaluate and interpret them to determine limits, trends and system status.

Question Source (New, Modified, Bank) I Modified If Bank or Modified:

VISION System/Question 10 506449 Question :Source RFC*13 ILT 10-1 NRC & AUDIT EXAM Page: 101 of 218 26 May 2011

EXAMINATION ANSWER KEY ILT 10-1 NRC RO Exam Memory or Comprehension X Fundamental Cognitive or Analysis 3:SPR Knowledge Level NUREG 1021 Appendix B: Solve a eroblem using References 55.41 I 5 I 55.43 I Facility operating characteristics during steady state 10CRF55 and transient conditions, including coolant chemistry, Content causes and effects of temperature, pressure and reactivity changes, effects of load changes, and operating limitations and reasons for these operating characteristics.

Justification for LORT questions with N/A KIA values < 3.0 Time to Complete: 1-2 minutes I Point Value: 1 System 10 No.: 202001 PRA: I NO Safety ~ Initial License Level 1

Function: o LORT ILT 10-1 NRC & AUDIT EXAM Page: 102 of 218 26 May 2011

EXAMINATION ANSWER KEY ILT 10-1 NRC RO Exam 35 10: 10-1 NR035 Points: 1.00 The plant is shutdown and fuel shuffling is taking place. The following annunciator is then received in the Control Room:

  • ROD CNTRL - ROD BLOCK Which of the following states the cause of this alarm?

A. The Main Fuel Hoist was just loaded with a fuel bundle over the core.

B. The Monorail Auxiliary Hoist was just loaded with a control rod blade over the core.

C. The Main Fuel Hoist was positioned on a fuel bundle when the grapple ENGAGED light went ON.

D. The Main Fuel Hoist was loaded with fuel in the Spent Fuel Pool when a control rod was withdrawn to position 02.

Answer: A Answer Explanation I QID: 10-1 NR035 Question # I 35 I Developer I Date: JJR 17-11-11 Knowledge and Ability Reference Information Importance Rating K&A RO SRO 234000 Fuel Handling Equipment A3.02 - Ability to monitor automatic operations 3.1 3.7 of the FUEL HANDLING EQUIPMENT Including: tlnterlock operation Level I RO I Tier 2 I Group I 2 General UFSAR Table ReferencG!s 7.7-1 ILT 10-1 NRC & AUDIT EXAM Page: 103 of 218 26 May 2011

EXAMINATION ANSWER KEY ILT 10-1 NRC RO Exam A is Correct. The plant is shutdown and fuel shuffling is underway. When the hoist loaded light comes on, this means that the hoist is loaded with fuel (as sensed by the load cell). When the hoist is loaded with fuel over the core, a control rod block is installed.

B is Incorrect but plausible. Even with the bridge over the core, a loaded Monorail Auxiliary Hoist does not install a control rod block.

Explanaticln C is Incorrect but plausible. There is not yet any load on the fuel hoist, even though it is positioned over the core. The grapple engaged light verifies that the grapple is closed. It does not input into the rod block circuit.

o is Incorrect but plausible. A loaded hoist in the Spent Fuel Pool does not create a control rod block nor does a single control rod withdrawn to position 02.

References to be None provided durin~ exam:

lesson Plan 2621.812.0.0003, Refueling learning RFl-2391, Demonstrate understanding of the Objective/ interlocks and rod blocks associated with the following refueling platform components, including their purpose and applicable technical specifications: bridge and trolley, main hoist, aux.

hoist.

Question Source (New, Modified, Bank) I Bank If Bank or Modified:

VISION System/Question 10 609313 Question Source IlT 07-1 NRC RO EXAM Memory or X Comprehension Fundamental Cognitive 1:1 or Analysis Knowledge level NUREG 1021 Appendix B: Interlocks, setpoints, or system (singular) response ILT 10-1 NRC & AUDIT EXAM Page: 104of218 26 May 2011

EXAMINATION ANSWER KEY ILT 10-1 NRC RO Exam 55.41 I 7 I 55.43 I 10CRF55 Design, components, and functions of control and Content safety systems, including instrumentation, Signals, interlocks, failure modes, and automatic and manual features.

Justification for LORT questions with N/A KIA values < 3.0 Time to Complete: 1-2 minutes I Point Value: 1 System ID No.: 234000 PRA: I NO Safety I:8;j Initial License Level 8

Function: D LORT ILT 10-1 NRC & AUDIT EXAM Page: 1050f218 26 May 2011

EXAMINATION ANSWER KEY ILT 10-1 NRC RO Exam 36 10: 10-1 NR036 Points: 1.00 The plant was at rated power when an ATWS occurred. Plant conditions include the following:

  • The 'A' Reactor Feed Pump (RFP) is being placed in operation lAW SP-19, Feedwater/Condensate And CRD System Operation
  • The BOP momentarily places the FEED PUMP 1A control switch to the START position
  • The 'A' Reactor Feed Pump fails to start and annunciator FEED PUMP TRIP A comes into alarm Which of thE~ following would cause this condition?

A. 'A' RFP shaft shear has occurred.

B. 'A' RFP Aux Lube Oil Pump did NOT start.

C. Reactor vessel water level is 165 inches above TAF.

D. 120 VAC Control Power to starting circuitry is NOT available.

Answer: B Answer Explanation J QID: 10-1 NR036 Question # I 36 I Developer 1 Date: JJR 17-11-11 Knowledge and Ability Reference Information Importance K&A Rating RO SRO 259001 Reactor Feedwater A4.02 - Ability to manually operate andlor 3.9 3.7 monitor in the control room: Manually start/control a RFPITDRFP Level I RO I Tier 2 I Group I 2 General 223R0173 15786350 RAP-J1d References Sh.7 Sh.184a ILT 10-1 NRC & AUDIT EXAM Page: 106 of 218 26 May 2011

EXAMINATION ANSWER KEY ILT 10-1 NRC RO Exam B is Correct. Since the lube oil permissive is required to start a feed pump taking the control switch to normal after start will close 7/7T and 9T/9.

With the breaker open 52 3/3C is closed causing 30T to annunciate. The pump will not start because PSX1 contact did not close in the starting circuit.

A is Incorrect but plausible. There is nothing in the trip circuitry which will trip the pump on a pump shaft shear. A seized shaft on the other hand would Explanation cause an overcurrent. The operator must have this knowledge to rule out this distractor.

C is Incorrect but plausible. Although the ROPS would be a plausible answer it does not come into play until 181 inches. The operator must understand this is high end of the normal water level band.

o is Incorrect but plausible since there are other large breakers in the plant that utilize 120 VAC for breaker control power.

References to be None provided durin~ exam:

Lesson Plan 2621.828.0.0017, Feed and Condensate System LeaminS3 CFW-10449, State the function and interpretation of Objectivel system alarms, alone and in combination, as applicable in accordance with the system RAPS.

Question Source (New, Modified, Bank) I Bank If Bank or Modified:

VISION SYlstem/Question 10 505913 Question Source FEED & COND-36 Memory or X Comprehension Fundamental Cognitive 1 :1 or Analysis Knowledge Level NUREG 1021 Appendix B: Interlocks, setpoints, or system (singular) response 55.41 I 7 I 55.43 I 10CRF55 Design, components, and functions of control and Content safety systems, including instrumentation, signals, interlocks, failure modes, and automatic and manual features.

ILT 10-1 NRC & AUDIT EXAM Page: 107 of 218 26 May 2011

EXAMINATION ANSWER KEY ILT 10-1 NRC RO Exam Justificati()n for LORT questions with N/A KIA values, < 3.0 Time to Complete: 1*2 minutes I Point Value: 1 System ID No.: 259001 PRA: I NO Safety 2

IZl Initial License Level Function: o LORT ILT 10-1 NRC & AUDIT EXAM Page: 108 of 218 26 May 2011

EXAMINATION ANSWER KEY ILT 10-1 NRC RO Exam 37 10: 10-1 NR037 Points: 1.00 The plant had reached the point of adding heat during a startup and has established a stable heatup rate. LETDOWN FLOW CONTROLLER FCV-ND22 was open to 20%.

If the airline to ND22 broke off, which of the following states the impact on RPV water level control and the corrective action to mitigate this impact?

Impact on RPV Water Level Corrective Action A. Lowers Increase makeup to the RPV B. Rises Open V-16-S7, LETDOWN TO RADWASTE C. Rises Limit makeup to the RPV D. Lowers Close V-16-60, LETDOWN TO CONDENSER Answer: C Answer Explanation I QID: 10-1 NR037 Question # I 37 I Developer I Date: JJR/7-11-11 Knowledge and Ability Reference Information Importance Rating K&A RO SRO 204000 RWCU 2.1.28 - Conduct of Operations: Knowledge of 4.1 4.1 the purpose and function of major system components and controls.

Level I RO I Tier 2 I Group I 2 General ABN-35 References ILT 10-1 NRC & AUDIT EXAM Page: 109 of 218 26 May 2011

EXAMINATION ANSWER KEY ILT 10~1 NRC RO Exam C is Correct. During a startup with a heatup underway, RPV water level will rise due to thermal expansion. RPV water level control is through ND22 to the condenser. When air is lost to this valve, it fails closed. As the heatup continues, RPV water level will rise. The only course of action until letdown can be re-established is to limit makeup into the RPV. The applicant must have knowledge of the purpose and function of ND-22 (a major system Explanation component in the RWCU system) in order to answer this question.

A & 0 are Incorrect but plausible since water level rises, not lower.

B is Incorrect but plausible. Letdown to radwaste is downstream of ND22, which means that opening the radwaste letdown path will have no flow, and thus no impact on water level.

References to be None provided during exam:

Lesson Plan 2621.828.0.0039, Reactor Water Cleanup Learning SDC-10435, Given plant operating conditions, Objective/ describe or explain the purpose(s)/function(s) of the system and its components.

Question Source (New, Modified, Bank) I Bank If Bank or Modified:

VISION System/Question 10 609072 Question Source ILT 07-1 RO Audit Exam Memory or X Comprehension Fundamental 1:F Cognitive or Analysis Knowledge 1:P Level NUREG 1021 Appendix B: facts; f.rocedure steps and cautions 55.41 I 7 I 55.43 I 10CRF55 Design, components, and functions of control and Content safety systems, including instrumentation, signals, interlocks, failure modes, and automatic and manual features.

Justification for LORT questions with N/A KIA values < 3.0 ILT 10-1 NRC & AUDIT EXAM Page: 110 of 218 26 May 2011

EXAMINATION ANSWER KEY ILT 10-1 NRC RO Exam Time to Cc,mplete: 1-2 minutes I Point Value: 1 System 10 No.: 204000 PRA: I NO Safetlf ~ Initial License Level 2

Functio'n: D LORT ILT 10-1 NRC & AUDIT EXAM Page: 111 of 218 26 May 2011

EXAMINATION ANSWER KEY ILT 10-1 NRC RO Exam 38 10: 10..1 NR038 Points: 1.00 The plant was at rated power. I&C then informed the Control Room that ALL Lo-Lo-Lo Level Barton (RE-18A. 18B, 18C, and 180) level instruments are frozen at 160 inches TAF and will NOT provide the correct level indication.

A primary coolant leak then developed outside of the Drywell.

If RPV water level lowered to 50 inches TAF, which of the following statments is correct?

A. MSIVs are OPEN.

B. EDGs are in STANDBY.

C. RBCCW to the Drywell is in service.

D. RWCU system isolation valves are OPEN.

Answer: C IAnswer Explanation QID: 10-1 NR038 Question # I 38 I Developer 1 Date: JJR 17-11-11 Knowledge and Ability Reference Information Importance Rating K&A RO SRO 216000 Nuclear Boiler Instrumentation K3.02 - Knowledge of the effect that a loss or malfunction of the NUCLEAR BOILER 4.0 4.3 Instrumentation will have on following:

PCIS/NSSSS Level I RO I Tier 2 I Group I 2 General UFSAR RVI Lesson 148F712 References Pg.9.2-18 Plan ILT 10-1 NRC & AUDIT EXAM Page: 112 of 218 26 May 2011

EXAMINATION ANSWER KEY ILT 10-1 NRC RO Exam C is Correct. The Lo-Lo-Lo Bartons provide an isolation signal for RBCCW to the Drywell and also have an input into ADS logic. Both Lo-Lo-Lo Bartons being isolated will prevent them from actuating.

Note that a High Drywell Pressure input AND Lo-Lo Level input will isolate RBCCW to the Drywell also, however the question stem states that the leak is Explanation outside of the Drywell so it can be assumed that Drywell pressure is unaffected by the leak.

A, B, and 0 are Incorrect. The Lo-Lo-Lo Bartons do not provide input for EDG start, MSIV closure, or RWCU system isolation. The logic for this comes off other vessel level instrumentation. These distractors are plausible if the applicant does not recall this information.

References to be None provided durin tI exam:

Lesson Plan 2621.828.0.0055, Reactor Vessel Instrumentation Learning RVI-10453, Explain or describe how this system is Objective/ interrelated with other plant systems.

Question Source (New, Modified, Bank) I New If Bank or Modified: N/A VISION System/Question 10 Question Source Memory or X Comprehension Fundamental 2:RI or Analysis Cognitive Knowledge Level NUREG 1021 Appendix B: Recognizing Interaction between systems (plural), including consequences and implications 55.41 I 7 I 55.43 I 10CRF55 Design, components, and functions of control and Content safety systems, including instrumentation, signals, interlocks, failure modes, and automatic and manual features.

Justification for LORT questions with N/A KIA values < 3.0 Time to Complete: 1-2 minutes I Point Value: 1 System 10 No.: 216000 PRA: I NO Safety ~ Initial License Level 7

Function: o LORT ILT 10-1 NRC & AUDIT EXAM Page: 113 of 218 26 May 2011

EXAMINATION ANSWER KEY ILT 10-1 NRC RO Exam 39 ID: 10-1 NR039 Points: 1.00 The plant is at rated power with the following conditions:

  • Main Generator volts: 24KV
  • Main Generator MW: 650 MW
  • Main GeneratorVARS: 100 MVARS
  • Hydrogen Pressure: 45 psig A grid distrubance results in steadily LOWERING grid voltage. The Main Generator voltage regulator responds as designed by attempting to raise Main Generator terminal voltage.

Panel8F/9F Generator Capability Curve is provided below.

GENERATOR EXCITATION CURVE 700 600

.4. ** ..: I I I~ 1","' "t<!!

l

!"'~

1 soo -

-..- '0

--~

--.-I""-"lK.:--'"

WutY'f. ..

i 1

-- - -~!"-. 'l:: i'-... ~l,,~' i

/ tT" r-.... "") N)c. ;,;', 1 ,,<;<1II I

" ---l"- v 1$

r- ~ V V ~ ~ i

  • l r-H- V V V i ' ~ ./"

.", ~ ~

~,)("

\

_\ _ i"/!.o. j

'\ v . . ~-:l ,

/'

tOO

..... II,' fI!!O / V V ~ ......... ~ ~ ~\ 1I,.J.u.-t v,. !5 ~ .....

-...- ... , ~ ~~I

'/

.. ......... I "r!

100 il!-

I l I

  • *o. ** * ,,"

t.J, Me

~

~.""'UIIIIJICI.""HM_

~\'Me-...,...~.o-_~_

,,*i

. , , f With NO operator action, this transient could result in _ _ _ _ _ __

A. overheating the Main Generator rotor windings B. overheating the Main Generator stator windings C. exceeding the Generator Under Excitation Limit D. Generator Lockout due to reverse power relay trip ILT 1()"1 NRC & AUDIT EXAM Page: 114 of 218 26 May 2011

EXAMINATION ANSWER KEY ILT 10-1 NRC RO Exam Answer: B swer Explanation QID: 10-1 NR039 Question # I 39 I Developer 1 Date: JJR 17-11-11 Knowledge and Ability Reference Information Importance Rating K&A RO SRO 700000 Generator Voltage and Electric Grid Disturbances AK1.01 - Knowledge of the operational implications of the following concepts as they 3.3 3.4 apply to GENERATOR VOLTAGE AND ELECTRIC GRID DISTURBANCES and the following: Over-excitation Level I RO I Tier 1 I Group I 1 General 336.1 References ILT 10-1 NRC & AUDIT EXAM Page: 115 of 218 26 May 2011

EXAMINATION ANSWER KEY ILT 10-1 NRC RO Exam B is Correct. The given conditions (lowering grid voltage) will cause the generator automatic voltage regulator to attempt to raise grid voltage, cuasing the generator to pick up additional VARS (i.e., move up on the Generator Capability Curve). Without operator action, this would result in exceeding the Generator Capability Curve (B-C) for 45 psig hydrogen pressure. Per procedure 336.1 Attachment 336.1-1, curve B-C is limited by armature (stator) heating.

A is Incorrect but plausible since this would be true if curve A-B was the limiting factor. Also plausible if the applicant was confused between field, armature, Explanation rotor and stator.

C is Incorrect but plausible since this would be true if grid voltage was rising, resulting in lowering VARS on the main generator (i.e., move down on the Generator Capability Curve).

D is Incorrect. A reverse power trip occurs when real load (MW) is reduced to the point where the grid supplies the generator. The given conditions would not result in lowering MW, especially to the point of reverse power. Plausible if the applican was confused on real vs. reactive load sharing between generators.

References to be None provided durin ~ exam:

lesson Plan 2621.828.0.0025, Main Generator learning GEN-10445, Given a set of system indications or Objective/ data, evaluate and interpret them to determine limits, trends and system status.

Question Source (New, Modified, Bank) I Bank If Bank or Modified:

VISION System/Question ID N/A Question Source Peach Bottom 2011 RO NRC Exam Memory or X Comprehension Fundamental 3:SPK Cognitive or Analysis Knowledge level NUREG 1021 Appendix B: Solve a ~roblem using Knowledge and its meaning ILT 10-1 NRC & AUDIT EXAM Page: 1160f218 26 May 2011

EXAMINATION ANSWER KEY ILT 10-1 NRC RO Exam 55.41 I 7 I 55.43 I 10CRF55 Design, components, and functions of control and Content safety systems, including instrumentation, Signals, interlocks, failure modes, and automatic and manual features.

Justification for LORT questions with N/A KIA values < 3.0 Time to Complete: 1-2 minutes I Point Value: 1 System 10 No.: 700000 PRA: I NO Safety 6

I2J Initial License Level Function: o LORT ILT 10-1 NRC & AUDIT EXAM Page: 117 of 218 26 May 2011

EXAMINATION ANSWER KEY ILT 10-1 NRC RO Exam 40 ID: 10..1 NR040 Points: 1.00 The plant is operating at 75% power when a loss of 125 VDC control power to 4160 VAC Bus 1A occurs.

Which of thE~ following describes the effect this event has on the Reactor Recirculation System?

The (1 ) Recirc Pump DRIVE MOTOR breakers have lost indication in the Control Room.

Placing their DRIVE MOTOR breaker switch in STOP (2) open the breaker.

ill A. A, B, & E will B. A,C,&E will C. A,B,&E will NOT D. A,C,&E will NOT Answer: D Answer Ex.planation I QID: 10-1 NR040 Question # I 40 I Developer I Date: JJR 17..11-11 Knowledge and Ability Reference Information Importance Rating K&A RO SRO 295004 Partial or Total Loss of DC Pwr 16 AK1.05* Knowledge of the operational implications of the following concepts as they 3.3 3.4 apply to Pl"RTIAL OR COMPLETE LOSS OF D.C. POWER: Loss of breaker protection Level I RO I Tier 1 I Group I 1 General EB D..3033 BR3001A References ILT 10*1 NRC & AUDIT EXAM Page: 118 of 218 26 May 2011

EXAMINATION ANSWER KEY ILT 10-1 NRC RO Exam D is Correct. The question stem provides a condition where 125 VDC (from DC-C) Control Power is lost to 4160 VAC Bus 1A. Reactor Recirc Pump A, C, & E are powered from 4160 VAC Bus 1A and a loss of breaker control power to these pumps will result in a loss of breaker indication and remote operation of the breaker will become unavailable.

A is Incorrect. 'rhis distractor is plausible since there is other logic (such as Recirc Pump ATWS logic) that affects the A, B, & E pumps together. "rhe applicant may confuse pump power supplies with the ATWS logic and is plausible if the applicant does Explanation not recall that a loss of breaker control power disables remote operation of the breaker.

B is Incorrect. This distractor is plausible if the applicant does not recall that a loss of breaker control power disables remote operation of the breaker.

C is Incorrect. This distractor is plausible since there is other logic (such as Recirc Pump ATWS logic) that affects the A, B, & E pumps together. The applicant may confuse pump power supplies with the ATWS logic.

References to be None provided durin ~ exam:

Lesson Plan 2621.828.0.0012, DC Distribution Learning DCD-1121, State potential consequences on plant Objective/ operation, plant equipment, and environment due to failure of DC Electrical systems.

Question Source (New, Modified, Bank) I Modified If Bank or Modified:

VISION System/Question ID 507192 Question Source 440 Memory or X Comprehension Fundamental 2:RI or Analysis Cognitive Knowledge Level NUREG 1021 Appendix B: Recognizing Interaction between systems (plural), including consequences and implications ILT 10-1 NRC & AUDIT EXAM Page: 119 of 218 26 May 2011

EXAMINATION ANSWER KEY ILT 10-1 NRC RO Exam 55.41 I 7 I 55.43 I 10CRF55 Design, components, and functions of control and Content safety systems, including instrumentation, signals, interlocks, failure modes, and automatic and manual features.

Justification for LORT questions with N/A KIA values < 3.0 Time to Complete: 1-2 minutes I Point Value: 1 System 10 No.: 295004 PRA: I NO Safety 6

IZI Initial License Level Function: o LORT ILT 10-1 NRC & AUDIT EXAM Page: 120 of 218 26 May 2011

EXAMINATION ANSWER KEY ILT 10-1 NRC RO Exam 41 10: 10-1 NR041 Points: 1.00 Given the following conditions:

  • Reactor power is 60% and steady
  • Main Generator output indicates 330 MWe and steady An event th4~n occurs resulting in the sequential opening of ALL Turbine Bypass Valves over a 3 minute period. Plant conditions now include the following:
  • Reactor power is 60% and steady
  • Main Generator output indicates 110 MWe and steady Based on current plant conditions, which one of the following states the plant impact if the Main Turbine were to trip?

The reactor would scram 'from ....

A. MSIV position.

B. turbine acceleration relay.

C. turbine stop valve position.

D. reactor pressure or neutron monitoring.

Answer: D Answer ElC:planation I QID: 10-1 NR041 Question # I 41 I Developer 1 Date: JJR 17-11-11 Knowledge and Ability Reference Information Importance Rating K&A RO SRO 295005 MCllin Turbine Generator Trip 1 3 AK1.01 - ~~nowledge of the operational implications of the following concepts as they 4.0 4.1 apply to MAIN TURBINE GENERATOR TRIP:

Pressure effects on reactor power Level I RO I Tier I 1 I Group I 1 ILT 10-1 NRC & AUDIT EXAM Page: 121 of 218 26 May 2011

EXAMINATION ANSWER KEY ILT 10-1 NRC RO Exam General UFSAR BR 2002 Sh. 4 LER 95-005 References 7.7.1.5 ABN-10 D is Correct. The turbine trip scram is bypassed (ie.,

no reactor scram if the turbine trips) when reactor power is less than 30%, but the mechanism to sense this amount of power is the HP turbine 3rd stage steam extraction pressure. When everything is normal, this pressure will be directly proportional to reactor power. In the case above, when the turbine bypass valves (TBVs) are open, the main turbine only sees about 20% reactor power, with the other 40% going through the TBVs. Now, it would seem as though the turbine trip-reactor scram is bypassed since the turbine only senses 20% power. Therefore, when the turbine does trip with reactor power at 60%

Explanation and TBV passing only 40% steam flow, reactor will not scram from TSV closure and reactor pressure will rise. Void suppression will act to raise reactor power and the reactor will scram on either high pressure or nuclear instrumentation.

A, B, & C are Incorrect but plausible if the applicant does not recall the turbine anticipatory scram logic.

The action to not exceed 40% power until all TBVs are closed and to scram if the Turbine Trips at > 30%

power are commitments in LER 95-005. All choices also provide their own scram signals, but not under the scenario for this question.

References to be None provided durin~ exam:

Lesson Plan 2621.828.0.0038, Reactor Protection System Learning RPS-1157, Describe all RPS scram logic trip Objective/ signals, including the following: 1. Purpose /

Design Basis; 2. Setpoints; 3. Conditions that allow bypassing scram signals; 4. How bypassing scram signals is accomplished.

Question Source (New, Modified, Bank) I Bank If Bank or Modified:

VISION System/Question 10 608552 Question Source ILT 07-1 Comp 3 Memory or X Cognitive Comprehension Fundamental 3:PEO Level or Analysis Knowledge ILT 10*1 NRC & AUDIT EXAM Page: 122 of 218 26 May 2011

EXAMINATION ANSWER KEY ILT 10-1 NRC RO Exam NUREG 1021 Appendix B: ~redict an Event or Outcome 55.41 I 7 I 55.43 I 10CRF55 Design, components, and functions of control and Content safety systems, including instrumentation, Signals, interlocks, failure modes, and automatic and manual features.

Justification for LORT questions with N/A KIA values < 3.0 Time to Complete: 1..2 minutes I Point Value: 1 System ID No.: 295005 PRA: I NO Safety 3

IZI Initial License Level Function: o LORT ILT 10-1 NRC & AUDIT EXAM Page: 123 of 218 26 May 2011

EXAMINATION ANSWER KEY ILT 10-1 NRC RO Exam 42 ID: 10-1 NR042 Points: 1.00 lAW procedure 205.0, Reactor Refueling, which of the following choices would be the MAXIMUM fuel pool temperature where fuel transfers into the fuel pool would still be allowed?

Answer: C Answer Explanation I QID: 10-1 NR042 Question # I 42 I Developer I Date: JJR/7-11-11 Knowledge and Ability Reference Information Importance Rating K&A RO SRO 295023 REtfueling Acc Cooling Mode I 8 AK2.02 * ~~nowledge of the interrelations between REFUELING ACCIDENTS and the 2.9 3.2 following: Fuel pool cooling and cleanup system Level I RO I Tier 1 I Group I 1 General 205.0 References D is Correct. lAW 205.0, Reactor Refueling, fuel transfers into the fuel pool are not permitted if fuel pool temperature exceeds 115°F. This requirement applies in case there is an event which results in a Explanation loss of fuel pool cooling during refuel operations.

All distractors are Incorrect but plausible if the applicant does not recall the correct temperature value.

References to be None provided during exam:

ILT 10-1 NRC & AUDIT EXAM Page: 124 of 218 26 May 2011

EXAMINATION ANSWER KEY ILT 10*1 NRC RO Exam Lesson Plan 2621.812.0.0003, Refueling Learning RFL-7442, Describe, in general, refueling and fuel Objectiv.~/ handling procedures to include precautions and limitations per Procedure 205 series.

Question Source (New, Modified, Bank) I New If Bank or Modified: N/A VISION System/Question ID Question Source Memory or X Comprehension Fundamental Cognitive 1:P or Analysis Knowledge Level NUREG 1021 Appendix B: f!rocedure steps and cautions 55.41 I 7 I 55.43 I 10CRF55 Design, components, and functions of control and Content safety systems, including instrumentation, signals, interlocks, failure modes, and automatic and manual features.

Justification for LORT questions with N/A KIA values < 3.0 Time to Complete: 1-2 minutes I Point Value: 1 System ID No.: 295023 PRA: I NO Safety I25j Initial License Level 8

Function: o LORT ILT 10-1 NRC & AUDIT EXAM Page: 125 of 218 26 May 2011

EXAMINATION ANSWER KEY ILT 10-1 NRC RO Exam 43 ID: 10-1 NR043 Points: 1.00 The plant was at rated power when an event occurred resulting in an airborne radiological release outside of the plant structures. Plant conditions include the following:

  • A radiological release is in-progress Which of th '9 following states how and why the control room HVAC system should be aligned?

A. System A must be run in the PART RECIRC Mode to maintain a positive pressure in the Control Room.

B. System B must be run in the FULL RECIRC Mode to minimize the use of outside air into the Control Room.

C. System A must be run in the PURGE Mode, to remove contaminated air from the Control Room, utilizing the fan only.

D. System B must be run in the PURGE Mode, to remove contaminated air from the Control Room, utilizing the fan only.

Answer: A Answer Explanation I QID: 10-1 NR043 Question # I 43 I Developer 1 Date: JJR 17-11-11 Knowledge and Ability Reference Information Importance Rating K&A RO SRO 295038 High Off-site Release Rate 19 EK2.03

  • Knowledge of the interrelations 3.6 3.8 between HIGH OFF-5ITE RELEASE RATE and the following: Plant ventilation systems Level I RO I Tier 1 I Group I 1 General 331.1 References ILT 10-1 NRC & AUDIT EXAM Page: 126 of 218 26 May 2011

EXAMINATION ANSWER KEY ILT 10-1 NRC RO Exam A is Correct. "rhere are no automatic actions of the control room ventilation system from any high radiation signal.

Procedure 331.1, Control Room and Old Cable Spreading Room Heating, Ventilation and Air Conditioning System, describes the partial recirculation mode: this mode of operation is provided to minimize contamination infiltration into the control room by maintaining a positive pressure in the control room using partial outside air.

Section 8.1.1 of 331.1, provides guidance for a radiological release with offsite power available.

With offsite power available, System B or System A should be run in PART RECIRC mode. Only when there is a loss of offsite power, shall the System be run with the fan only (to limit EDG loading).

Explanation B is Incorrect but plausible. Running System A in the FULL RECIRC Mode is incorrect. Full Recirc mode is used to minimize the intrusion of toxic gases into the control room.

C is Incorrect but plausible. Running System A in the PURGE mode is incorrect. Purge mode is used to remove smoke, fumes, or other undesirable odors from the control room. Also, running the systems with fans only is required only when combined with a loss of off-site power to reduce EDG loading.

D is Incorrect but plausible. Running System B in the PURGE mode is incorrect. Purge mode is used to remove smoke, fumes, or other undesirable odors from the control room.

References to be None provided durin ~ exam:

Lesson Plan 2621.828.0.0054, Turbine Building & MISC HVAC Learning SDC-2324, Explain the basis, with use of Objectivel procedure, the four different modes of control room ventilation damper alignment and the effects of the damper alignment modes on control room habitability.

ILT 10-1 NRC & AUDIT EXAM Page: 127 of 218 26 May 2011

EXAMINATION ANSWER KEY ILT 10-1 NRC RO Exam Question Source (New, Modified, Bank) I Bank If Bank or Modified:

VISION System/Question 10 510832 Question Source ILT 05-1 NRC RO EXAM Memory or X Comprehension Fundamental 3:SPK Cognitive or Analysis Knowledge Level NUREG 1021 Appendix B: Solve a Problem using Knowledge and its meaning 55.41 I 7 I 55.43 I 10CRF55 Design, components, and functions of control and Content safety systems, including instrumentation, signals, interlocks, failure modes, and automatic and manual features.

Justification for LORT quefstions with N/A KIA values < 3.0 Time to C1omplete: 1-2 minutes I Point Value: 1 System I[) No.: 295038 PRA: I NO Safety [8J Initial License Level 9

Functicm: D LORT ILT 10-1 NRC & AUDIT EXAM Page: 128 of 218 26 May 2011

EXAMINATION ANSWER KEY ILT 10-1 NRC RO Exam 44 ID: 10-1 NR044 Points: 1.00 Under which of the following conditions CAN Containment Sprays be initiated in the OW SPRAY mode during high drywell temperature conditions?

CONTAINMENT SPRAY INITIATION LIMIT 600

- 8 550 I (7,550) c 500 450

- J 400

- II DRYWELL r TEMPERATURE 350 (OF) 300 I

250

- I 200

- I 150

- I

- /A 100 o 1,6 5 10 15 20 25 30 35 40 DRYWELL PRESSURE (PSIG)

Drywell Pressure (psig) Drywell Temperature (OF)

A. 2 200 B. 3 250 C. 5 300 O. 6 470 Answer: C Answer Explanation I QID: 10*1 NR044 Question # I 44 I Developer 1 Date: JJR 17-11-11 Knowledge and Ability Reference Information ILT 10-1 NRC & AUDIT EXAM Page: 129 of 218 26 May 2011

EXAMINATION ANSWER KEY ILT 10-1 NRC RO Exam Importance Rating K&A RO SRO 295028 High Drywell Temperature / 5 EK2.01 - Knowledge of the interrelations 3.7 4.1 between tUGH DRYWELL TEMPERATURE and the following: Drywell spray: Mark-I&II Level I RO I Tier 1 I Group I 1 General EOP User's PCC EOP References Guide C is Correct. Starting containment spray due to high DW temperature can be performed only when DW temp/DW pressure point is below the containment spray initiation limit (CSIL) curve. Only answer C is Explanati40n below the curve and others are above.

All distractors are Incorrect but plausible if the applicant does not correctly interpret the CSIL graph or understand the basis for the CSIL curve.

References to be None provided during exam:

Lesson Plan 2621.845.0.0056, Primary Containment Control Learning PCC-3000, Using the EOP User's Guide, evaluate Objective/ the technical bases for each step in the procedure and apply this evaluation to determine correct courses of action under emergency conditions.

Question Source (New, Modified, Bank) I Bank If Bank or Modified:

VISION System/Question ID 609031 Question Source ILT 07-1 AUDIT RO EXAM Memory or X Comprehension Fundamental 3:SPR Cognitive or Analysis Knowledge Level NUREG 1021 Appendix B: Solve a Problem using References 10CRF55 55.41 I 10 I 55.43 I Content Administrative, normal, abnormal, and emergency operating procedures for the facility.

Justification for LORT que!stions with N/A KIA values < 3.0 Time to Complete: 1-2 minutes I Point Value: 1 ILT 10-1 NRC & AUDIT EXAM Page: 130 of 218 26 May 2011

EXAMINATION ANSWER KEY ILT 10-1 NRC RO Exam System 10 No.: 295028 PRA: I NO Safety [8J Initial License Level 5

Function: D LORT ILT 10-1 NRC & AUDIT EXAM Page: 131 of218 26 May 2011

EXAMINATION ANSWER KEY ILT 10-1 NRC RO Exam 45 ID: 16-1 NR045 Points: 1.00 The plant was at rated power when an event resulted in the following conditions:

  • RPV water level indicates 0" and lowering slowly
  • NO RPV injection systems are available The Steam Cooling EOP has been entered. Which of the following is correct?

lAW the EOP Users Guide, an RPV water level of _ _(1l._ _, would still provide enough steam flow through the core to prevent exceeding _ _W- clad temperature.

ill (2)

A. -17" 1500 OF B. -23" 1500 OF C. -33" D. -38" Answer: C Answer Explanation I QID: 10-1 NR045 Question # I 45 I Developer 1 Date: JJR 17-11-11 Knowledge and Ability Reference Information Importance Ratina K&A RO SRO 295031 Reactor Low Water Level EK3.04 - Knowledge of the reasons for the following responses as they apply to 4.0 4.3 REACTOR LOW WATER LEVEL: Steam cooling Level I RO I Tier 1 I Group I 1 General EOP User's References Guide ILT 10-1 NRC & AUDIT EXAM Page: 132 of 218 26 May 2011

EXAMINATION ANSWER KEY ILT 10-1 NRC RO Exam C is Correct. The Steam Cooling EOP has been entered. Core cooling is maintained from the steam passing the uncovered portions of the fuel by one of two mechanisms: injection into the RPV is available or injection is not available. H injection is available, as long as RPV water level is ~ -20", then cladding temperature will remain ~ 1500 of. If no injection is available, as long as RPV water level is ~ -35", then Explanation cladding temperature will remain ~1800 OF. With no RPV injection, an RPV water level of -33" ensures clad temperature ~1800 OF.

A & B are Incorrect but plausible due to level above 35", but the temperature limit is incorrect.

o is Incorrect but plausible due to level being less than -35".

References to be None provided during exam:

Lesson Plan 2621.845.0.0055, Steam Cooling Learning ESC-3004, Describe in detail each step or Objective/ conditional statement including the technical basis and how to verify or perform each step as required.

Question Source (New, Modified, Bank) I Bank If Bank or Modified:

VISION System/Question 10 718322 Question Source ILT 09-1 NRC RO EXAM Memory or X Comprehension Cognitive Fundamental 1:B or Analysis Level Knowledge NUREG 1021 Appendix B: B.ases or purpose 10CRF55 55.41 I 10 I 55.43 I Content Administrative, normal, abnormal, and emergency operating procedures for the facility.

Justification for LORT questions with N/A KIA values < 3.0 Time to Complete: 1-2 minutes I Point Value: 1 System 10 No.: 295031 PRA: I NO Safety 2

IZJ Initial License Level Function: o LORT ILT 10-1 NRC & AUDIT EXAM Page: 133 of 218 26 May 2011

EXAMINATION ANSWER KEY ILT 10-1 NRC RO Exam 46 ID: 10-1 NR046 Points: 1.00 The plant was at rated power when an event resulted in a scram. The plant is currently cooling down with the Shutdown Cooling System (SOC). Current conditions are as follows:

  • RPV water level is 181 in TAF and steady
  • Recirculation Pump suction temperature is 265°F
  • SOC Pump C is operating, with the other SOC Pumps unavailable
  • Main Condenser vacuum indicates 8 in Hg An electrical fault in the breaker cubicle for SOC C discharge valve V-17-57 causes the valve to close. RPV temperature starts to rise.

Under these conditions, which of the following methods (and reason for using that method) can be used to cooldown the RPV?

A. Isolation Condensers since using this method will preserve RPV water inventory.

B. The Turbine Bypass Valves since this is the preferred method for rejecting decay heat from the reactor.

C. Feed with CRO and Bleed with Reactor Water Cleanup System letdown since the hotwell can still be considered to be available.

O. Alternate shutdown cooling with Safety Valves and Core Spray since this is the method recommended by ABN-3, Loss of Shutdown Cooling.

Answer: C Answer Explanation I QID: 10-1 NR046 Question # I 46 I Developer 1 Date: JJR 17-11-11 Knowledge and Ability Reference Information Importance Rating K&A RO SRO 295021 Loss of Shutdown Cooling 14 AK3.02 - Knowledge of the reasons for the following responses as they apply to LOSS OF 3.3 3.4 SHUTDOWN COOLING: Feeding and bleeding reactor vessel Level I RO I Tier I 1 I Group I 1 ILT 10-1 NRC & AUDIT EXAM Page: 134 of 218 26 May 2011

EXAMINATION ANSWER KEY ILT 10-1 NRC RO Exam Generall ABN-3 303 References C is Correct. The question stem describes a loss of main condenser vacuum followed by a total loss of Shutdown Cooling (SOC). ABN-3, Loss of SOC, describes several methods of alternate cooling.

Feed (with CRO/Cond Pump) and Bleed (with RWCU letdown) are the only choices available due to the conditions in the question stem. The reason the RWCU letdown can be used is even with no condenser vacuum, the condenser is still considered intact and available. RWCU to the hotwell might reach 120-130F, however this is not steam conditions.

Explanation A is Incorrect. This distractor is plausible if the applicant does not recall that Isolation Condensers cannot be used when RPV water level is > 160 in TAF.

B is Incorrect. This distractor is plausible if the applicant does not recall that the Main Condenser is not capable of accepting steam with no vacuum since the Bypass Valves will be closed.

o is Incorrect. This distractor is plausible since this method is available if EMRVs were used instead of SRVs, which the distractor states. SRVs do not have the capability to be manually operated.

References to be None provided during exam:

Lesson Plan 2621.828.0.0045, Shutdown Cooling System Learning SOC-10453, Explain or describe how this system is Objecti"el interrelated with other plant systems.

Question Source (New, Modified, Bank) I Modified If Bank or Modified:

VISION S~,stem/Question 10 510757 Question Source ILT 05-1 RO AUDIT EXAM Memory or X Comprehension Fundamental 3:SPK Cognitive or Analysis Knowledge Level NUREG 1021 Appendix B: Solve a eroblem using Knowledge and its meaning ILT 10-1 NRC & AUDIT EXAM Page: 135 of 218 26 May 2011

EXAMINATION ANSWER KEY ILT 10-1 NRC RO Exam 55.41 I 5 I 55.43 I Facility operating characteristics during steady state 10CRF55 and transient conditions, including coolant chemistry, Content causes and effects of temperature, pressure and reactivity changes, effects of load changes, and operating limitations and reasons for these operating characteristics.

Justification for LORT questions with N/A KIA values < 3.0 Time to Cc.mplete: 1-2 minutes I Point Value: 1 System 10 No.: 295021 PRA: I NO Safety ~ Initial License Level 4

Function: o LORT ILT 10-1 NRC & AUDIT EXAM Page: 136 of 218 26 May 2011

EXAMINATION ANSWER KEY ILT 10-1 NRC RO Exam 47 ID: 10*1 NR047 Points: 1.00 The plant was at rated power when an event resulted in a large break LOCA. Plant conditions include the following:

  • Reactor Power indicates 1% on all APRMs
  • Reactor Pressure indicates 600 psig and lowering
  • Torus temperature indicates 95°F and rising
  • Torus water level indicates 150 inches and rising
  • Drywell Pressure indicates 30 psig and rising
  • Torus Pressure indicates 28 psig and rising Based on the above plant parameters, the US enters and directs Emergency Depressurization (ED) due to exceeding an EOP Figure limit.

Refer to the EOP Figures below.

TORUS LOAD LIMIT 180 TORUS WATER LEVEL (IN.)

110 100

--r B o 100 200 300 400 500 000 700 800 900 1000 1100 1200 RPV PRESSURE (PSIG)

ILT 10-1 NRC & AUDIT EXAM Page: 137 of 218 26 May 2011

EXAMINATION ANSWER KEY ILT 10-1 NRC RO Exam PRESSURE SUPPRESSION PRESSURE TORUS PRESSURE 15+-~+---f---+---~--~--~--~--~~~~

(PSIG) 10~~~--,---+---+---~---r---r---+---H~~

5 -t---II-------I ---t----t-----t--r---t--+--H---t D

100 110 120 130 140 150 160 170 180 190 200 IQJ3Y£jlA!.~R..~JWJ ILT 10-1 NRC & AUDIT EXAM Page: 138 of 218 26 May 2011

EXAMINATION ANSWER KEY ILT 10-1 NRC RO Exam PRIMARY CONTAINMENT PRESSURE LIMIT 60 60 -

rA.l - ~

,~) ,r:::-.,

I&!

TORUS PRESSURE 31'}

(PSIG) 40

'" \.0

.tE~

'-' (I'F 21'}

10 I'}

I I I I I I I I I I I I I I I I I I I I I I I I I I I I I o 60 12.0 180 240 3CI') S6C 42.0 490 PRIMARY CONT.>\INI\.tENT WATER LEVEL UN; ILT 10-1 NRC & AUDIT EXAM Page: 139 of 218 26 May 2011

EXAMINATION ANSWER KEY ILT 10-1 NRC RO Exam TORUS HIGH LEVEL 230 220 210 200 +--T"f!ot:o~

TORUS TEMPERATURE 190 r"'~~~~"""'F'"

(~

TORUS LEVEL


144

-154

-164

-174 150+---+---"~--+---+---+---+---+-~~~~~~~ --180

-188 140~~~~-~~~~~~~~~~~~~~~~~~~

o 100 200 300 400 500 600 100 800 900 1000 1100 RPV PRESSURE (pSIG)

TORUS LOW LEVEL 240 230 220 210 200 TORUS TEMPERATURE 180

(~

180 170 TORUS 1eo .

LEVEL

-143 150 120

--- 110 140 0 100 200 300 400 500 eoo 700 800 800 1000 1100 RPV PRESSURE (PSIG)

ILT 1()"1 NRC & AUDIT EXAM Page: 140 of 218 26 May 2011

EXAMINATION ANSWER KEY ILT 10-1 NRC RO Exam What EOP Figure limit has been exceed?

A. Torus Load Limit (TLL)

B. Pressure Supression Pressure (PSP)

C. Heat Capacity Temperature Limit (HCTL)

D. Primary Containment Pressure Limit (PCPL)

Answer: B IAnswer Explanation QID: 10-1 NR047 Question # I 47 I Developer/Date: JJR/7-11-11 Knowledge and Ability Reference Information Importance Rating K&A RO SRO 295024 High Drywell Pressure / 5 EK3.04 - Knowledge of the reasons for the following responses as they apply to HIGH 3.7 4.1 DRYWELL PRESSURE: Emergency depressulrization Level I RO I Tier 1 I Group I 1 General EOP User's PCC EOP Referencles Guide B is Correct. The question stem provides a condition plant parameters where an EOP Figure Limit has been exceeded. The applicant must analyze the conditions and determine that the reason an ED was directed was the PSP had been exceeded (Torus Pressure at 28 psig and Torus Level at 150 in exceeds the limit). Drywell pressure and Torus Explanation pressure are relatively equal in a Mark-I containment during LOCA conditions.

All distractors are plausible if the applican does not interpret the plant parameters correctly. All distractors are EOP Figures that if they were exceeded would require an ED.

ILT 10-1 NRC & AUDIT EXAM Page: 141 of 218 26 May 2011

EXAMINATION ANSWER KEY ILT 10-1 NRC RO Exam References to be None provided durin~ exam:

Lesson Plan 2621.845.0.0056, Primary Containment Control Learning PCC-3000, Using the EOP User's Guide, evaluate Objective/ the technical bases for each step in the procedure and apply this evaluation to determine correct courses of action under emergency conditions.

Question Source (New, Modified, Bank) I New If Bank or Modified: N/A VISION System/Question 10 Question Source Memory or X Comprehension Fundamental 3:SPK Cognitive or Analysis Knowledge Level NUREG 1021 Appendix B: Solve a ~roblem using Knowledge and its meaning 10CRF55 55.41 I 10 I 55.43 I Content Administrative, normal, abnormal, and emergency operating procedures for the facility.

Justification for LORT questions with N/A KIA values < 3.0 Time to Complete: 1-2 minutes I Point Value: 1 System ID No.: 295024 PRA: I NO Safety 5

L8l Initial License Level Function: o LORT ILT 10-1 NRC & AUDIT EXAM Page: 142 of 218 26 May 2011

EXAMINATION ANSWER KEY ILT 10-1 NRC RO Exam 48 ID: 10-1 NR048 Points: 1.00 Given the following conditions:

  • RPV pressure is 1050 PSIG and rising
  • RPV water level is 100 inches T AF and lowering
  • Isolation Condenser Transfer Switches for Train "A" and "B" are in ALTERNATE
  • ALL actions required to be performed prior to exiting the Control Room have been completed Concerning the Isolation Condensers, which of the following will occur if an initiation signal is received?

A. ONLY the "A" Isolation Condenser will automatically initiate B. ONLY the "B" Isolation Condenser will automatically initiate C. Initiation signals are bypassed; the operator must open the DC Condensate Return Valve to place "A" Isolation Condenser in service D. Initiation signals are bypassed; the operator must open the DC Condensate Return Valve to place "B" Isolation Condenser in service Answer: D IAnswer Explanation QID: 10-1 NR048 Question # I 48 I Developer 1 Date: JJR 17-11-11 Knowledge and Ability Reference Information Importance Rating K&A RO SRO 295016 Control Room Abandonment 1 7 AA1.09 - Ability to operate and/or monitor the following as they apply to CONTROL ROOM 4.0 4.0 ABANDONMENT: Isolation/emergency condenser(s): Plant-Specific Level I RO I Tier 1 I Group I 1 General ABN-30 346 References ILT 10-1 NRC & AUDIT EXAM Page: 143 of 218 26 May 2011

EXAMINATION ANSWER KEY ILT 10*1 NRC RO Exam o is Correct. Only the 'B' Isolation Condenser (IC) can be operated from the Remote Shutdown Panel (RSP). When 'A' and 'B' IC Train switches are in ALTERNATE, all automatic initiations are bypassed.

  • rhe 'B' IC Condensate Return Valve, V-14-35, must Explanati~on be manually opened on the RSP.

All distractors are Incorrect but plausible if the applicant does not recall the IC initiation logic when their Train controls are in Alternate.

References to be None provided durin~ exam:

Lesson F*lan 2621.828.0.0023, Isolation Condensers Learning ICS-10456, Describe the Isolation Condenser Objective/ System design feature which provides for the following: a. System control outside of the control room (including automatic actions bypassed) b.

Removal of non-condensable gases.

Question Source (New, Modified, Bank) I Bank If Bank or Modified:

VISION System/Question 10 507169 Question Source 417 Memory or X Comprehension Fundamental 3:PEO Cognitive or Analysis Knowledge Level NUREG 1021 Appendix B: eredict an Event or Outcome 55.41 I 7 I 55.43 I 10CRF55 Design, components, and functions of control and Content safety systems, including instrumentation, signals, interlocks, failure modes, and automatic and manual features.

Justification for LORT qUE!stions with N/A KIA value~s < 3.0 Time to C,omplete: 1-2 minutes I Point Value: 1 System I[) No.: 295016 PRA: I NO Safety 7

IZI Initial License Level Functic)n: D LORT ILT 10-1 NRC & AUDIT EXAM Page: 144 of 218 26 May 2011

EXAMINATION ANSWER KEY ILT 10-1 NRC RO Exam 49 ID: 10..1 NR049 Points: 1.00 The plant was at rated power. An event then occurs and plant conditions include the following:

At Time =0 seconds:

  • Annunciator 4160V STATION POWER - BUS 1C VOLTS LO comes into alarm
  • Panel8F/9F 4160V BUS 1C voltage indicates 3750 AC VOLTS Based on these conditions, at what time will the following Panel 8F/9F indications be observed?
  • At Time = (1) seconds, EDG 1 white UNIT START light will be lit.
  • At Time = (2) seconds, MAIN BREAKER IC green OPEN light will be LIT, and red CLOSED light will be OFF.

ill A. o 3 B. 3 3 C. 3 10 D. 10 10 Answer: 0 IAnswer Explanation QID: 10*1 NR049 Question # I 49 I Developer 1 Date: JJR 17*11*11 Knowledge and Ability Reference Information Importance Rating K&A RO I SRO ILT 10-1 NRC & AUDIT EXAM Page: 1450f218 26 May 2011

EXAMINATION ANSWER KEY ILT 10-1 NRC RO Exam 295003 Partial or Complete Loss of AC / 6 AA1.03 - Ability to operate and/or monitor the following as they apply to PARTIAL OR 4.4 4.4 COMPLETE LOSS OF A.C. POWER: Systems necessary to assure safe plant shutdown Level I RO I Tier 1 I Group I 1 General References RAP-T3a UFSAR 7.4 D is Correct. The question stem provides a condition where 4160 VAC Bus 1C is experiencing a low Voltage condition <<3830 volts). lAW RAP T-3-a, after 10 seconds, 4160 V Breaker 1C will trip and EDG-1 will Fast Start. lAW the UFSAR, the EDGs are part of Systems Required for Safe Shutdown. This question examines the applicants ability to monitor this system upon a loss of 4160 VAC to the 1C Explanation emergency bus.

All distractors are Incorrect. On a complete loss of voltage, or voltage < 2704 V for 3 seconds, 4160 V Breaker 1C will trip and EDG-1 will Fast Start at the 3 second mark. These distractors are plausible if the applicant does not recall all automatic breaker trip logic or EDG fast start logic.

References to be None provided during exam:

Lesson Plan 2621.828.0.0016, AC Electrical Distribution Learning EDS-10449, State the function and interpretation of Objective/ system alarms, alone and in combination, as applicable in accordance with the system RAPS.

Question Source (New, Modified, Bank) I Modified If Bank or Modified:

VISION System/Question ID 510791 Question Source ILT 05-1 RO AUDIT EXAM Memory or X Comprehension Fundamental Cognitive 1:1 or Analysis Knowledge Level NUREG 1021 Appendix B: Interlocks, setpoints, or system (singular) response ILT 10-1 NRC & AUDIT EXAM Page: 146of218 26 May 2011

EXAMINATION ANSWER KEY ILT 10-1 NRC RO Exam 55.41 I 7 I 55.43 I 10CRF55 Design, components, and functions of control and Content safety systems, including instrumentation, signals, interlocks, failure modes, and automatic and manual features.

Justificath)n for LORT questions with N/A KIA values < 3.0 Time to Complete: 1*2 minutes I Point Value: 1 System 10 No.: 295003 PRA: I NO Safety [gI Initial License Level 6

Function: D LORT ILT 10-1 NRC & AUDIT EXAM Page: 147 of 218 26 May 2011

EXAMINATION ANSWER KEY ILT 10-1 NRC RO Exam 50 10: 10-1 NR050 Points: 1.00 The reactor is at rated power. An event then occurred and plant conditions include the following:

  • RPV Pressure indicated 1070 psig for several seconds, then started lowering How did the plant respond?

A. Isolation Condenser 'A' ONLY is in service and its vent valves are CLOSED B. Isolation Condenser 'B' ONLY is in service and its vent valves are OPEN C. NEITHER Isolation Condenser is in service and their vent valves are OPEN D. BOTH Isolation Condensers are in service and their vent valves are CLOSED Answer: D IAnswer Explanation QID: 10..1 NR050 Question # I 50 I Developer 1 Date: JJR 17*11-11 Knowledge and Ability Reference Information Importance Rating K&A RO SRO 295025 High Reactor Pressure 1 3 EA1.06 - Ability to operate and/or monitor the following as they apply to HIGH REACTOR 4.5 4.5 PRESSURE: Isolation condenser: Plant-Specific Level I RO I Tier 2 I Group I 1 General RAP-C1a BR3029 IC Lesson Plan References ILT 10-1 NRC & AUDIT EXAM Page: 148 of 218 26 May 2011

EXAMINATION ANSWER KEY ILT 10-1 NRC RO Exam D is Correct. The question stem provides a condition where RPV pressure rose above the EMRV lift setpoint. When RPV pressure is > 1051 psig for 1.5 seconds, both ICs will initiate and their vent valves will close. The question stem indicates that the EMRV rose for several seconds.

A is Incorrect. The first EMRV to lift would be the 'A' EMRV due to its setpoint and location in the main steam header. This distractor is plausible if the Explanation applicant believes that only 'A' EMRV lifted, only 'A' IC will initiate.

B is Incorrect. This distractor is plausible if the applicant does not recall that both ICs initiate when a high RPV pressure condition exists.

C is Incorrect. This distractor is plausible if the applicant does not recall that ICs should have initiated on high RPV pressure.

References to be None provided during exam:

Lesson Plan 2621.828.0.0023, Isolation Condensers Learning ICS-2030, Describe the Isolation Condenser design Objectivel feature(s) and/or interlocks (including signals and setpoints) which provide for the following: a)

Automatic system initiation; b) Automatic system isolation Question Source (New, Modified, Bank) I Bank If Bank or Modified:

VISION System/Question ID 510768 Question Source ILT 05-1 RO AUDIT EXAM Memory or Fundamental X Comprehension Cognitive Knowledge 1 :1 or Analysis Level NUREG 1021 Appendix B: Interlocks, setpoints, or system (singular) response 55.41 I 7 I 55.43 I 10CRF55 Design, components, and functions of control and Content safety systems, including instrumentation, signals, interlocks, failure modes, and automatic and manual features.

ILT 10-1 NRC & AUDIT EXAM Page: 149 of 218 26 May 2011

EXAMINATION ANSWER KEY ILT 10-1 NRC RO Exam Justification for LORT que~stions with N/A KIA values < 3.0 Time to Complete: 1*2 minutes I Point Value: 1 System ID No.: 295025 PRA: I NO Safety I2Sl Initial License Level 3

Function: D LORT ILT 10-1 NRC & AUDIT EXAM Page: 150 of 218 26 May 2011

EXAMINATION ANSWER KEY ILT 10-1 NRC RO Exam 51 10: 10-1 NR051 Points: 1.00 The plant was at rated power when an event occurred requiring entry into the Primary Containment Control EOP.

Panel 1 F/2F Torus indications are as follows (assume these parameters remain constant):

ILT 10-1 NRC & AUDIT EXAM Page: 151 of218 26 May 2011

EXAMINATION ANSWER KEY ILT 10-1 NRC RO Exam Which of the following choices below indicates the MAXIMUM RPV Pressure allowed before the Heat Capacity Temperature Limit (HCTL) has been exceeded?

TORUS HIGH LEVEL 230 220 210 +--~

200+-~~ __

TORUS TEMPERATURE 190 +--II'!I~~""'~-':

(1)

TORUS LEVEL 110 +----+

-. 144

-154

-164

---114 150 +----~--~-4---+--_+--_r--~~~--+_~~~ --1_

-188 140~~,-~~~~~~~~~~~~~~~~~~~~

o 100 200 300 400 500 600 700 _0 900 1000 1100 RPV PRESSURE (PSIG)

A. 450 psig B. 650 psig C. 750 psig D. 800 psig Answer: B IAnswer Explanation QID: 10-1 NR051 Question # I 51 I Developer I Date: JJR 17-11-11 Knowledge and Ability Reference Information K&A I Importance Rating ILT 10-1 NRC & AUDIT EXAM Page: 152 of 218 26 May 2011

EXAMINATION ANSWER KEY ILT 10-1 NRC RO Exam RO SRO 295026 Suppression Pool High Water Temp. /

5 EA2.03 .. Ability to determine and/or interpret 3.9 4.0 the following as they apply to SUPPRESSION POOL HIGH WATER TEMPERA'rURE: Reactor pressure level I RO I Tier 1 I Group I 1 General EOP User's PCC EOP References Guide B is Correct. At a Torus level of 177", the green 180" Torus level line applies. RPV Pressure must be maintained below the green line. The maximum RPV pressure listed where Torus temperature and level intersect without exceeding the HCTl is 650 psig.

A is Incorrect. This distractor is plausible if the applicant swaps Torus Temperature and Torus level indications when interpreting the EOP Figure limit.

In this instance the orange 164" level line would apply and from the choices listed, only 450 psig would not violate the HCTL.

Explanath)n C is Incorrect. This distractor is plausible if the applicant tries to extrapolate the Torus level of 177" in between the green and orange level lines. In this case, 175 psig is the maximum pressure listed which would not violate the HCTL.

D is Incorrect. This distractor is plausible if the applicant chooses the orange 174" level line to maintain pressure below. In this case the applicant will choose 800 psig as the maximum RPV pressure listed which will not violate the HCTl.

References to be None provided durin ~ exam:

Lesson Plan 2621.845.0.0056, Primary Containment Control learning PCC..10445, Given a set of system indications or Objective/ data, evaluate and interpret them to determine limits, trends and system status.

I Question Source (New, Modified, Bank) New ILT 10-1 NRC & AUDIT EXAM Page: 153 of 218 26 May 2011

EXAMINATION ANSWER KEY ILT 10-1 NRC RO Exam If Bank or Modified: N/A VISION System/Question ID Question Source Memory or X Comprehension Fundamental 3:SPK Cognitive or Analysis Knowledge Level NUREG 1021 Appendix B: Solve a ~roblem using References 10CRF55 55.41 I 10 I 55.43 I Content Administrative, normal, abnormal, and emergency operating procedures for the facility.

Justification for LORT questions with N/A KIA values < 3.0 Time to Complete: 1-2 minutes I Point Value: 1 System ID No.: 295026 PRA: I NO Safety 5

IZI Initial License Level Function: D LORT ILT 10-1 NRC & AUDIT EXAM Page: 154 of 218 26 May 2011

EXAMINATION ANSWER KEY ILT 10-1 NRC RO Exam 52 ID: 10-1 NR052 Points: 1.00 The plant was at rated power when a fire was confirmed in the 'A' 4BOV Switchgear Room. Sev~eral minutes later the following indications were observed in the Control Room:

  • EMRV NR-10BB and NR-10BE indicate spuriously opening and closing
  • ALL other EMRVs indicate closed and have NOT opened lAW ABN-29, Plant Fires, which of the following actions is required due to the condition stated above?

Enter ABN-'I, Reactor Scram, and place ...

A. ALL EMRV keylocks in DISABLE.

B. ALL EMRV control switches in OFF.

C. ONLY EMRV NR-10BB and NR-10BE keylocks in DISABLE.

D. ONLY EMRV NR-10BB and NR-10BE control switches in OFF.

Answer: C IAnswer Explanation QID: 10-1 NR052 Question # I 52 I Developer I Date: JJR I 7-11-11 Knowledge and Ability Reference Information Importance Rating K&A RO SRO 600000 Plant Fire On-site I 8 AA2.16 - Ability to determine and interpret the following ,IS they apply to PLANT FIRE ON 3.0 3.5 SITE: Vita~ equipment and control systems to be maintamned and operated during a fire Level I RO I Tier 1 I Group I 1 General FSP-OB6A ABN-29 References ILT 10-1 NRC & AUDIT EXAM Page: 1550f218 26 May 2011

EXAMINATION ANSWER KEY ILT 10-1 NRC RO Exam A is Correct. The question stem provides a condition where there is a fire in the 'A' 480V Switchgear Room. Fire Support Procedure (FSP)

OB6A directs manually scramming the reactor and disabling the spuriously opening EMRVs if spurrious operation is observed. This is due to the fire causing hot shorts in the EMRV circuitry.

FSP-OB6A also states that all EMRV can be un-Explanation DISABLED if required by EOPs. EMRVs are Vital Equipment required for ADS and overpressure protection. This is how their operation is managed during a plant fire in the 'A' 480V Switchgear Room testing their knowledge per the KIA.

All distractors are Incorrect but plausible if the applicant does not recall the correct action to take in this situation. Too difficult (level 5) without reference.

References to be FSP-OB6A provided durin ~ exam:

Lesson Plan 2621.828.0.0019, Fire Protection System Learning FPS-10450, Describe and interpret procedure Objective/ sections and steps for plant emergency or off normal conditions that involve this system including personnel allocation and equipment operation lAW applicable ABN, SDRP, EOP & EOP su~port procedures and EP Procedures.

Question Source (New, Modified, Bank) I New If Bank or Modified: N/A VISION System/Question ID Question Source Memory or X Comprehension Fundamental Cognitive 1:P or Analysis Knowledge level NUREG 1021 Appendix B: frocedure steps and cautions 10CRF55 55.41 I 10 I 55.43 I Content Administrative, normal, abnormal, and emergency operating procedures for the facility.

ILT 10-1 NRC & AUDIT EXAM Page: 156 of 218 26 May 2011

EXAMINATION ANSWER KEY ILT 10-1 NRC RO Exam Justification for LORT questions with N/A KIA values < 3.0 Time to Complete: 1-2 minutes I Point Value: 1 System ID No.: 600000 PRA: I NO Safet),

8 181 Initial License Level Function: o LORT ILT 10-1 NRC & AUDIT EXAM Page: 157 of 218 26 May 2011

EXAMINATION ANSWER KEY ILT 10-1 NRC RO Exam 53 10: 10-1 NR053 Points: 1.00 The plant is in hot shutdown and has just commenced cooling down for an outage utilizing the Shutdown Cooling System, when the following Panel 1 F/2F annunciator came into alarm:

@O~R8CCW ISOL Which of th.~ following annunciators would be the NEXT expected annunciator to come into alarm?

A. TORUS/ORYWELL - OW TEMP HI B. MAIN STEAM - TRUNNION RM TEMP HI

c. RBCCW - CCWISO CLG/FUEL POOL TEMP HI O. CLEANUP SYSTEM - AUX PUMP CCW TEMP HI Answer: A IAnswer ElC:planation QID: 10-1 NR053 Question # I 53 I Developer I Date: JJR 17-11-11 Knowledge and Ability Reference Information Importance Rating K&A RO SRO 295018 Partial or Total Loss of CCW 18 AA2.01 - Ability to determine andlor interpret the following as they apply to PARTIAL OR 3.3 3.4 COMPLETE LOSS OF COMPONENT COOLING WATER: Component temperatures Level I RO I Tier I 1 I Group I 1 ILT 10-1 NRC & AUDIT EXAM Page: 158 of 218 26 May 2011

EXAMINATION ANSWER KEY ILT 10-1 NRC RO Exam General RAP-C2c RAP-C8h References A is Correct. The given alarm in the question stem shows that one of the RBCCW to primary containment isolation valves is not full open (RAP C2c). [RBCCW OW Isolation Valve V-5-147, V-5-166, or V-5-167 not full open]. This would prevent/hinder RBCCW flow to all Primary Containment loads.

Answer A is the only alarm listed whose components (loss of RBCCW cooling to the DW air coolers would result in DW temperature high alarm) is cooled by RBCCW and are located within the primary containment (RAP-C8h).

Explanation B is Incorrect but plausible. This is indicative of a high temperature in the trunion room (also cooled by RBCCW) but it also is outside the primary containment and is unaffected by the loss of cooling to the primary containment.

C & D are Incorrect but plausible. These distractors could result from a total loss of RBCCW, but these components are located outside the primary containment and are not effected by the loss of RBCCW to the primary containment components.

References to be None provided durin ~ exam:

Lesson Plan 2621.828.0.0035, RBCCW System Learning RBC-0048, List possible causes, system response, Objective/ and affected RBCCW system components for an isolation signal.

Question Source (New, Modified, Bank) I Bank If Bank or Modified:

VISION System/Question ID 510676 Question Source ILT 05-1 RO AUDIT EXAM Memory or X Comprehension Fundamental 2:RI or Analysis Cognitive Knowledge Level NUREG 1021 Appendix B: Recognizing Interaction between systems (plural), including consequences and implications 10CRF55 55.41 10 55.43 Content ILT 10-1 NRC & AUDIT EXAM Page: 159 of 218 26 May 2011

EXAMINATION ANSWER KEY ILT 10-1 NRC RO Exam Administrative, normal, abnormal, and emergency operating procedures for the facility.

Justification for LORT questions with N/A KIA values < 3.0 Time to Complete: 1-2 minutes I Point Value: 1 System ID No.: 295018 PRA: I NO Safety 8

I:8J Initial License Level Function: o LORT ILT 10-1 NRC & AUDIT EXAM Page: 160 of 218 26 May 2011

EXAMINATION ANSWER KEY ILT 10-1 NRC RO Exam 54 10: 10-1 NR054 Points: 1.00 The plant is at 70% power. The Panel 7F Air Compressor lineup is as follows:

COMPRESSOR 1

  • LEA[) compressor
  • Indicates RED light ON and GREEN light OFF
  • RED breaker flag is showing COMPRESSOR 2
  • LAG compressor
  • Indicates GREEN light ON and RED light OFF
  • GREEN breaker flag is showing COMPRESSOR 3
  • STANDBY compressor
  • Indicates GREEN light ON and RED light OFF
  • GREEN breaker flag is showing A seizmic event results in a leak in the Instrument Air Header and an electrical fault on Unit Substation 1A 1. Plant indications now include the following:
  • Panel 7F meter INSTR AIR SUPPLY PRESS indicates 60 psig and slowly lowering Which of the following describes the current Air Compressor indications?

(assume NO operator action has been taken)

A.

  • COMPRESSOR 1 indicates GREEN light ON
  • COMPRESSOR 2 indicates RED light ON
  • COMPRESSOR 3 indicates RED light ON B.
  • COMPRESSOR 1 indicates GREEN light ON
  • COMPRESSOR 2 indicates RED light ON
  • COMPRESSOR 3 indicates GREEN light ON C.
  • COMPRESSOR 1 indicates RED light ON
  • COMPRESSOR 2 indicates GREEN light ON
  • COMPRESSOR 3 indicates RED light ON D.
  • COMPRESSOR 1 indicates RED light ON
  • COMPRESSOR 2 indicates RED light ON
  • COMPRESSOR 3 indicates GREEN light ON ILT 10-1 NRC & AUDIT EXAM Page: 161 of 218 26 May 2011

EXAMINATION ANSWER KEY ILT 10-1 NRC RO Exam Answer: A Answer Explanation I QID: 10-1 NR054 Question # I 54 I Developer I Date: JJR 17-11-11 Knowledge and Ability Reference Information Importance Rating K&A RO SRO 295019 Partial or Total Loss of Inst. Air 18 2.1.31 - Ability to locate control room switches, controls, and indications, and to 4.6 4.3 determine that they correctly reflect the desired plant lineup.

Level I RO I Tier 1 I Group I 1 General 334 References ILT 10-1 NRC & AUDIT EXAM Page: 162 of 218 26 May 2011

EXAMINATION ANSWER KEY ILT 10-1 NRC RO Exam A is Correct. The question stem provides a condition Air Compressor 1 is in LEAD, Compressor 2 is in LAG, and Compressor 3 is in STANDBY. A seizmic event results in a loss of USS-1A1 which is the power supply to Compressor 1. The event also results in an Instrument Air leak. As Instrument Air pressure lowers, the LAG Compressor will start at 95 psig and STANDBY Compressor will start at 85 psig.

The applicant just be able to recognize what Compressor indications are expected to be for the current situation in the question stem.

B is Incorrect. This distractor is plausible if the applicant does not recall that the STANDBY Explanation Compressor #3 will also start. The applicant might believe it must be manually started (and the stem states no operator action has been taken).

C is Incorrect. "rhis distractor is plausible if the applicant does not recognize that Compressor 1 lost power from the loss of USS-1A1, not Compressor 2.

These would be the expected indications if Compressor 2 had lost power.

D is Incorrect. This distractor is plausible if the applicant believes that neither Compressor has lost power and that Compressor 3 must be manually started.

References to be None provided durin g exam:

Lesson Plan 2621.828.0.0043, Service, Instrument, and Breathing Air Learning Objective/ CAS-10440, Given the system logic/electrical drawings, describe the system auto isolation signals, setpoints and expected system response including power loss or failed components.

Question Source (New, Modified, Bank) I New If Bank or Modified: N/A VISION System/Question ID Question Source Memory or X Cognitive Comprehension Fundamental 2:DR Level or Analysis Knowledge ILT 10-1 NRC & AUDIT EXAM Page: 163 of 218 26 May 2011

EXAMINATION ANSWER KEY ILT 10-1 NRC RO Exam NUREG 1021 Appendix B: D.escribing or recognizing Relationships 55.41 I 7 I 55.43 I 10CRF55 Design, components, and functions of control and Content safety systems, including instrumentation, signals, interlocks, failure modes, and automatic and manual features.

Justification for LORT questions with N/A KIA values < 3.0 Time to Complete: 1-2 minutes I Point Value: 1 System 10 No.: 295019 PRA: I NO Safety I:gj Initial License Level Function:

8 o LORT ILT 10-1 NRC & AUDIT EXAM Page: 164 of 218 26 May 2011

EXAMINATION ANSWER KEY ILT 10-1 NRC RO Exam 55 ID: 10-1 NR055 Points: 1.00 The plant w.as at rated power when a turbine trip/reactor scram occurred. Plant conditions include the following:

  • Both Isolation Condensers have auto initiated.
  • Two (2) EMRV's are OPEN.
  • Isolation Condenser B level is 7.7 feet and rising
  • Attempts to isolate the affected isolation condenser have failed
  • Torus bulk temperature is 91 degrees F and steady
  • 51' Cleanup Pump area radiation monitor C-1 is reading 5 mr/hr (annunciator AREA MON HI is not in alarm)
  • NO other annunciators are in alarm IN ADDITION TO RPV CONTROL -NO ATWS EOP, which EOP{s) has(have) met entry conditions and require implementation?

A. Primary Containment Control EOP AND Radioactivity Release Control EOP B. Primary Containment Control EOP ONLY C. Radioactivity Release Control EOP ONLY D. Secondary Containment Control EOP ONLY Answer: C Answer Explanation I QID: 10-1 NR055 Question # I 55 I Developer I Date: JJR 17-11-11 Knowledge and Ability Reference Information Importance Rating K&A RO SRO 295006 SCRAM 11 2.4.4 - AbUity to recognize abnormal indications for system operating parameters 4.5 4.7 that are entry-level conditions for emergency and abnormal operating procedures.

Level I RO I Tier I 1 I Group I 1 ILT 10-1 NRC & AUDIT EXAM Page: 165 of 218 26 May 2011

EXAMINATION ANSWER KEY ILT 10-1 NRC RO Exam General EOP User's RREOP ReferencE!S Guide C is Correct. The question stem provides a conditions of an Isolation Condenser Tube Leak.

lAW the Radioactivity Release EOP, an confirmed IC tube leak requires entry into the RR EOP.

A & B are Incorrect. These distractors are plausible if the applicant does not recognize Torus Explanation Temperature is below that required for entry into the Primary Containment Control EOP.

D is Incorrect. This distractor is plausible if the applicant does not recognize that the 51' Cleanup Pump area radiation monitor C-1 is below that which requires entry into Secondary Containment Control.

References to be None provided during exam:

Lesson Plan 2621.845.0.0058, Radioactivity Release Control Learning RRC-01667, Based upon specific plant parameters Objectivel and conditions, determine if entry conditions for EOPs have been met and which EOPs are applicable to the conditions provided.

Question Source (New, Modified, Bank) I Bank If Bank or Modified:

VISION Sy'stem/Question ID 608565 Question Source ILT 07-1 Comp # 3 Memory or X Comprehension Fundamental 3:SPK Cognitive or Analysis Knowledge Level NUREG 1021 Appendix B: Solve a Problem using Knowledge and its meaning 10CRF55 55.41 I 10 I 55.43 I Content Administrative, normal, abnormal, and emergency operating procedures for the facility.

Justification for LORT questions with N/A KIA values < 3.0 Time to Complete: 1-2 minutes I Point Value: 1 System 101 No.: 295006 PRA: I NO Safet~f 1

IZI Initial License Level Functicln: D LORT ILT 10-1 NRC & AUDIT EXAM Page: 166 of218 26 May 2011

EXAMINATION ANSWER KEY ILT 10-1 NRC RO Exam 56 10: 10-1 NR056 Points: 1.00 The plant was at rated power when a combined RPV Isolation and ATWS occurred.

Several EMRVs are cycling open and closed.

In the RPV Control - with ATWS EOP Pressure Leg, the following caution resides:

(~_C_A_U_T_IO_N_ _~}-------'

YARWAV L.EVEL INSTRUMENTS SHALl.. ~OT BE USED -:-0 DETERM!NE RPV WATER LEVEL DURING RAPID DEF'~ESSURIZATIO'"

BELa';\' 500 PSIG According to the EOP Users Guide, which one of the following states the basis for this caution?

A. Cold reference legs can provide higher RPV water level indication.

B. Heated reference legs can provide higher RPV water level indication.

C. Flashing variable legs can provide lower RPV water level indication.

D. Flashing variable legs can provide erratic RPV water level indication.

Answer: B IAnswer Explanation QID: 10-1 NR056 Question # I 56 I Developer I Date: JJR 17-11-11 Knowledge and Ability Reference Information Importance Rating K&A RO SRO 295025 High Reactor Pressure I 3 2.4.20 - Emergency Procedures I Plan:

3.8 4.3 Knowledge of operational implications of EOP warnings, cautions, and notes Level I RO I Tier 1 I Group I 1 General RPV Control- EOP User's References with ATWS EOP Guide ILT 10-1 NRC & AUDIT EXAM Page: 167 of 218 26 May 2011

EXAMINATION ANSWER KEY ILT 10-1 NRC RO Exam B is Correct. EOP Users Guide provides the following (page 1A-51): The caution warns the operator that rapid depressurization of the RPV can cause flashing and possible loss of liquid inventory from the water level instrument reference legs resulting in erratic RPV water level indications substantially higher than actual. This effect applies only to RPV water level instruments with heated reference legs (YARWAY level instruments). Since heated reference leg temperatures seldom exceed Explanatic)n 450F (saturation temperature for 500 psig), this phenomenon occurs only during rapid depressurization below 500 psig.

A is Incorrect but plausible since it refers to a cold reference leg - not a heated reference leg. The applicant may confuse this difference.

C & D are Incorrect but plausible since they refer to flashing in the variable legs - not the reference leg.

The applicant may confuse this difference.

References to be None provided during exam:

Lesson Plan 2621.845.0.0052, RPV Control* no ATWS Learning ENA-3056, Given a copy of RPV Control-no ATWS, Objective/ describe in detail each Caution or Note, including the technical basis and how to verify conformance at any time.

Question Source (New, Modified, Bank) I Bank If Bank or Modified:

VISION System/Question ID 510709 Question Source ILT 05*1 RO Audit Exam Memory or X Comprehension Cognitive Fundamental 1:B or Analysis Level Knowledge NUREG 1021 Appendix B: 8ases or purpose 10CRF55 55.41 I 10 I 55.43 I Content Administrative, normal, abnormal, and emergency operating procedures for the facility.

Justification for LORT questions with N/A KIA values < 3.0 ILT 10-1 NRC & AUDIT EXAM Page: 168of218 26 May 2011

EXAMINATION ANSWER KEY ILT 10-1 NRC RO Exam Time to Complete: 1-2 minutes I Point Value: 1 System ID No.: 295025 PRA: I NO Safe~, ~ Initial License Level Function:

3 o LORT ILT 10-1 NRC & AUDIT EXAM Page: 169 of 218 26 May 2011

EXAMINATION ANSWER KEY ILT 10-1 NRC RO Exam 57 ID: 10-1 NR057 Points: 1.00 The plant was at rated power when an event resulted in the following annunciator:

  • RX RECIRC PUMPS/DRIVES RECIRC PUMP A MG SET - DRV MOT BRKR TRIP A After the plant becomes stable, the operator placed the recirculation loop in an IDLE condition in accordance with ABN-2, Recirculation System Failures.

Which of the following states the response of ACTUAL total core flow (flow through the core):

(1) from when the annunciator came into alarm until the plant was stable, and (2) as a result of the operator action?

ACTUAL total core flow will.. ..

A. (1) drop ONLY (2) rise slightly B. (1) drop, then rise slightly (2) rise slightly C. (1) drop ONLY (2) remain the same D. (1) drop, then rise slightly (2) remain the same Answer: A IAnswer Explanation QID: 10-1 NR057 Question # I 57 I Developer 1 Date: JJR 17-11-11 Knowledge and Ability Reference Information Importance Rating K&A RO I SRO ILT 10-1 NRC & AUDIT EXAM Page: 170 of 218 26 May 2011

EXAMINATION ANSWER KEY ILT 10-1 NRC RO Exam 295001 Partial or Complete Loss of Forced Core Flow Circulation /1 & 4 AA2.03

  • Ability to determine and/or interpret 3.3 3.3 the following as they apply to PARTIAL OR COMPLETE LOSS OF FORCED CORE FLOW CIRCULA"nON: Actual core flow Level I RO I Tier 1 I Group I 1 General ABN*2 RAP-E1c References ILT 10-1 NRC & AUDIT EXAM Page: 171 of 218 26 May 2011

EXAMINATION ANSWER KEY ILT 10-1 NRC RO Exam A is Correct. A Recirc pump trip results in a drop in ACTUAL total core flow (less flow through the core).

When the operator place the recirc loop in an IDLE condion, more flow will be directed through the core and actual total core flow will rise slightly. Indicated total core flow however is different. Total indicated core flow will drop as the pump slows down.

Forward flow through the loop stops (which represents the lowest core flow value) and then there will be reverse flow through the idle recirc loop driven by the remaining operating pumps. This reverse flow is sensed by the A Recirculation loop flow transmitters as flow through the loop. Total core flow is a summation of the recirculation loop flows. Total core flow rises from its minimum flow (when forward flow stopped) then raises slightly to account for the reverse flow in recirculation loop A.

ABN-2 provides the definition of an idle recirculation Explanation loop as the discharge valve closed, and suction/discharge bypass valves as open. As the discharge valve is closed, indicated core flow will decrease.

B is Incorrect. This distractor is plausible if the applicant confuses Actual core flow with Indicated core flow.

C is Incorrect. This distractor is plausible if the applicant does not recall that placing a loop in an IDLE condition forces more actual flow through the core.

o is Incorrect. This distractor is plausible if the applicant confuses Actual core flow with Indicated core flow and does not recall that placing a loop in an IDLE condition forces more actual flow through the core.

References to be None provided durintl exam:

Lesson Plan 2621.828.0.0038, Reactor Recirculation System Learning RRS-10445, Given a set of system indications or Objective/ data, evaluate and interpret them to determine limits, trends and system status.

ILT 10-1 NRC & AUDIT EXAM Page: 172 of 218 26 May 2011

EXAMINATION ANSWER KEY ILT 10-1 NRC RO Exam Question Source (New, Modified, Bank) I Modified If Bank or Modified:

VISION System/Question 10 510673 Question Source ILT 05-1 RO Audit Exam Memory or X Comprehension Fundamental 3:PEO Cognitive or Analysis Knowledge Level NUREG 1021 Appendix B: Predict an Event or

.outcome 55.41 I 5 I 55.43 I Facility operating characteristics during steady state 10CRF55 and transient conditions, including coolant chemistry, Content causes and effects of temperature, pressure and reactivity changes, effects of load changes, and operating limitations and reasons for these operating characteristics.

Justification for LORT questions with N/A KIA values < 3.0 Time to Complete: 1-2 minutes I Point Value: 1 S-'Ystem 10 No.: 295001 PRA: I NO Safel3, [8J Initial License Level 1&4 Functioln: D LORT ILT 10-1 NRC & AUDIT EXAM Page: 173 of 218 26 May 2011

EXAMINATION ANSWER KEY ILT 10-1 NRC RO Exam 58 ID: 10-1 NR058 Points: 1.00 The plant was at rated power when an event required the crew to manually scram the reactor. The following crew actions have been completed:

  • BOTli MANUAL SCRAM BUS 1 and BUS 2 pushbuttons have been depressed
  • The REACTOR MODE SELECTOR switch has been placed in SHUTDOWN
  • NO additional operator actions have occurred
  • Panel 4F indications include the following:

lAW EOPs. which ONE of the following can be used to determine if the reactor is SHUTDOWN under all conditions without boron?

SRM - Source Range Monitor LPRM - Local Power Range Monitor APRM - Average Power Range Monitor RPIS - Rod Position Indication System A. SRM readings B. LPRM readings C. RPIS indications D. LPRM downscale lights ILT 10-1 NRC & AUDIT EXAM Page: 174 of 218 26 May 2011

EXAMINATION ANSWER KEY ILT 10-1 NRC RO Exam Answer: C IAnswer Explanation QID: 10-1 NR058 Question # I 58 I Developer / Date: JJR / 7-11-11 Knowledge and Ability Reference Information Importance Rating K&A RO SRO 295037 SCRAM Conditions Present and Reactor Power Above APRM Downscale or Unknown /1 EK2.14 - Knowledge of the interrelations 3.6 3.9 between SCRAM CONDrnON PRESENT AND REACTOR~ POWER ABOVE APRM DOWNSCALE OR UNKNOWN and the following: RPIS: Plant-Specific Level I RO I Tier 1 I Group I 1 General RPV Control EOP User's References EOP Guide C is Correct. lAW the EOP User's Guide, the reactor can be considered shutdown under all conditions without boron if all rods are at or beyond position 04 (RPIS indication).

Explanath)n All distractors are Incorrect but plausible since they will all indicate reactor power, however in an ATWS condition, the RPV Control- with ATWS EOP only allows for all rods at or beyond position 04 to be used when transitioning to RPV Control - no A TWS.

References to be None provided durin~ exam:

Lesson Plan 2621.845.0.0053, RPV Control- with ATWS Learning EWA-3053, Explain the basis for each of the RPV Objecthl'e/ Control - with A TWS entry conditions.

Question Source (New, Modified, Bank) I Modified If Bank or Modified:

VISION System/Question 10 560406 Question :Source Limerick ILT ILT 10-1 NRC & AUDIT EXAM Page: 175 of 218 26 May 2011

EXAMINATION ANSWER KEY ILT 10-1 NRC RO Exam Memory or X Comprehension Fundamental 2:DR Cognitive or Analysis Knowledge Level NUREG 1021 Appendix B: Describing or recognizing Relationships 55.41 I 7 I 55.43 I 10CRF55 Design, components, and functions of control and Content safety systems, including instrumentation, signals, interlocks, failure modes, and automatic and manual features.

Justification for LORT questions with N/A KIA values < 3.0 Time to Complete: 1-2 minutes I Point Value: 1 System ID No.: 295037 PRA: I NO Safety ~ Initial License Level 1

Function: o LORT ILT 10-1 NRC & AUDIT EXAM Page: 176 of 218 26 May 2011

EXAMINATION ANSWER KEY ILT 10-1 NRC RO Exam 59 10: 10-1 NR059 Points: 1.00 The plant w.as at rated power when the Secondary Containment Control EOP, EMG 3200.11, was entered due to high area temperatures (not due to a fire).

Which of the following area leak detection system annunciators will indicate an automatic isolation of the affected system?

A. Cleanup System area leak detection: CLEANUP SYSTEM - RWCU HELB annunciators B. Shutdown Cooling System area leak detection: SO HX CLG - SO HX PUMP RM TEMP HI annunciators C. Isolation Condenser System area leak detection: ISOL COND - CONO AREA TEMP HI annunciators O. Trunion Room area leak detection: MAIN STEAM - TRUNION RM TEMP HI annunciators Answer: A IAnswer Explanation QID: 10-1 NR059 Question # I 59 I Developer I Date: JJR 17-11-11 Knowledge and Ability Reference Information Importance Rating K&A RO SRO 295032 High Secondary Containment Area Temperature I 5 EK1.03 - Knowledge of the operational implications of the following concepts as they 3.5 3.9 apply to HIGH SECONDARY CONTAINMENT AREA TEMPERATURE: Secondary containment leakage detection: Plant-Specific Level I RO I Tier 1 I Group I 2 Generall RAP-D1d RAP..D2d References ILT 10-1 NRC & AUDIT EXAM Page: 177of218 26 May 2011

EXAMINATION ANSWER KEY ILT 10-1 NRC RO Exam A is Correct. Cleanup system leaks will be annunciated by D-1-d and D-2-d (RWCU HELB at 160F) and by D-8-d (CU ROOM TEMP HI). The HELB annunciators, when alarmed simultaneously, will isolate the cleanup system at 160F area temperature.

B is Incorrect but plausible. Shutdown cooling system leaks will be annunciated by C-8-d (SD HX PUMP RM TEMP HI) but provide no automatic Explanation actions.

C is Incorrect but plausible. Isolation condenser leaks will be annunciated by C-8-b (COND AREA TEMP HI) but provide no automatic actions.

D is Incorrect but plausible. Trunion room leaks will be annunciated by J-8-a (TRUNION RM TEMP HI) but provide no automatic actions.

References to be None provided durin~ exam:

Lesson Plan 2621.828.0.0039, Reactor Water Cleanup Learning RCU-10450, Describe and interpret procedure Objective/ sections and steps for plant emergency or off normal conditions that involve this system including personnel allocation and equipment operation lAW applicable ABN, SDRP, EOP & EOP support procedures and EP Procedures.

Question Source (New, Modified, Bank) I Bank If Bank or Modified:

VISION System/Question ID 510835 Question Source ILT 05-1 RO NRC Exam Memory or X Comprehension Fundamental Cognitive 1 :1 or Analysis Knowledge Level NUREG 1021 Appendix B: Interlocks, setpoints, or system (singular) response 55.41 I 7 I 55.43 I 10CRF55 Design, components, and functions of control and Content safety systems, including instrumentation, signals, interlocks, failure modes, and automatic and manual features.

ILT 10-1 NRC & AUDIT EXAM Page: 178 of 218 26 May 2011

EXAMINATION ANSWER KEY ILT 10-1 NRC RO Exam Justification for LORT questions with N/A KIA values < 3.0 Time to Complete: 1*2 minutes I Point Value: 1 System ID No.: 295032 PRA: I NO Safety [8J Initial License Level 5

Function: o LORT ILT 10-1 NRC & AUDIT EXAM Page: 179 of 218 26 May 2011

EXAMINATION ANSWER KEY ILT 10-1 NRC RO Exam 60 ID: 10*1 NR060 Points: 1.00 Given the following conditions:

  • RPV water level is 85 in and slowly lowering
  • Drywell pressure is 8.6 psig and slowly rising
  • Torus temperature is 96° F and rising
  • EOG-1 is out of service
  • 4160VAC Bus 1C MAIN BREAKER 1C is open What operator actions are required by procedure and can be executed with the above conditions present?

A. Start Containment Spray pump 51 Band ESW pump 52B in Torus Cooling mode.

B. Start Containment Spray pump 51C and ESW pump 52C in Torus Cooling mode.

C. Start Containment Spray pump 51 Band ESW pump 52B in Drywell Spray mode.

D. Start Containment Spray pump 51C and ESW pump 52C in Drywell Spray mode.

Answer: B IAnswer Explanation QID: 10-1 NR060 Question # I 60 I Developer I Date: JJR 17-11-11 Knowledge and Ability Reference Information Importance Rating K&A RO SRO 295013 High Suppression Pool Temperature I 5

AK2.01

  • Knowledge of the interrelations 3.6 3.1 between HIGH SUPPRESSION POOL "rEMPERATURE and the following:

Suppression pool cooling Level I RO I Tier I 1 I Group I 2 ILT 10*1 NRC & AUDIT EXAM Page: 180 of 218 26 May 2011

EXAMINATION ANSWER KEY ILT 10-1 NRC RO Exam General EMG*SP25 BR3001C BR 3002 Sh. 2 References B is Correct. The question stem provides a condition where Torus Temperature is above the Primary Containment Control EOP Entry of 95F.

Torus Cooling is then directed to be put in service.

Due to 4160VAC Bus 1C not having power, only System 2 Containment Spray/ESW Pumps have power (CS Pump 51C and ESW Pump 52C are in System 2).

A is Incorrect but plausible if the applicant does not Explanation recognize that System 1 Containment Spray/ESW Pumps do not have any power.

C is Incorrect. This distractor is plausible if the applicant does not recognize that System 1 Containment Spray/ESW Pumps do not have any power and that Drywell Sprays are not required (required at 12 psig Torus/Drywell pressure).

D is Incorrect. This distractor is plausible if the applicant does not recognize that Drywell Sprays are not required.

References to be None provided durin ~ exam:

Lesson Plan 2621.828.0.0009, Containment Spray/ESW Learning CNS-10445, Given a set of system indications or Objective/ data, evaluate and interpret them to determine limits, trends and system status.

Question Source (New, Modified, Bank) I Bank If Bank or Modified:

VISION System/Question ID 607938 Question Source ILT 07*1 RO Comp #1 Memory or Comprehension X

Fundamental 2:RI or Analysis Cognitive Knowledge Level NUREG 1021 Appendix B: Recognizing Interaction between systems (plural), including consequences and implications 10CRF55 55.41 I 10 I 55.43 I Content Administrative, normal, abnormal, and emergency operating procedures for the facility.

ILT 10-1 NRC & AUDIT EXAM Page: 181 of 218 26 May 2011

EXAMINATION ANSWER KEY ILT 10-1 NRC RO Exam Justification for LORT questions with N/A KIA values < 3.0 Time to Complete: 1-2 minutes I Point Value: 1 System 10 No.: 295013 PRA: I NO Safe~f [8J Initial License Level 5

Function: D LORT ILT 10-1 NRC & AUDIT EXAM Page: 182 of 218 26 May 2011

EXAMINATION ANSWER KEY ILT 1()"1 NRC RO Exam 61 ID: 10*1 NR061 Points: 1.00 The plant was at rated power. The following plant condition existed at the start of shift:

  • DRYVVELL PRESS indicates 1.21 psig Plant conditions at the end of shift include the following:
  • DRYVVELL PRESS indicates 1.45 psig Consider the following two conditions below for Drywell Temperature from the start of shift to the end of shift:
  • CONDITION (1): PPC indicates Drywell Temperature is UNCHANGED
  • CONDITION (2): PPC indicates Drywell Temperature has RISEN Which of the following could cause the indications for CONDITION (1) and CONDITION (2)?

A. (1) The barometric pressure has fallen.

(2) A Drywell Recirc Fan has tripped.

B. (1) Nitrogen has been added to the Drywell.

(2) A TBCCW pump has tripped.

C. (1) Reactor Recirc Pump 'D' has tripped.

(2) Varway reference leg has a small leak.

D. (1) All Fuel Pool Cooling Pumps have tripped.

(2) Intake temperature has RISEN.

Answer: A IAnswer Explanation QID: 10..1 NR061 Question # I 61 I Developer 1 Date: JJR 17*11*11 Knowledge and Ability Reference Information Importance Rating K&A RO I SRO ILT10-1 NRC & AUDIT EXAM Page: 183 of 218 26 May 2011

EXAMINATION ANSWER KEY ILT 10-1 NRC RO Exam 29S010 High Drywell Pressure /S AK3.0S

  • Knowledge of the reasons for the following responses as they apply to HIGH 3.S 3.4 DRYWELL PRESSURE: Temperature monitoring Level I RO I Tier 1 I Group I 2 General 'rhermo - Units RAP-C3f References & Properties A is Correct. The question stem provides a condition where Drywell Pressure has reached the DW PRESS HI/LO alarm. lAW RAP-C3f, Drywell Temperature must be checked due to determine cause of the DW pressure rise. To demonstrate the reason for and importance of checking (monitoring)

DW Temperature, the question asks the applicant to recognize a condition where DW Pressure would rise without a change in DW Temp, and a condition where DW Pressure would rise with a concurrent rise in DW Temp. This is important to diagnose the cause of the DW pressure rise. For Condition 1, barometric (atmospheric) pressure falling would result in indicated DW Pressure rising. DW pressure is measured in gauge (Pgauge=Pabsolute - PatmOSPhere)'

For Condition 2, a trip of a Drywell Recirc Fan will result in Drywell Pressure and Temperature rising.

RAP*C3f also has the applicant verify proper DW Explanation cooler operation.

B is Incorrect but plausible. For Condition 1, N2 added to the DW will result in OW Pressure rise without a change in DW Temp, however for Condition 2, a trip of a TBCCW pump will not cause OW Temperature to lower.

C is Incorrect but plausible. For Condition 1, a Recirc Pump trip will result in OW Press and Temp lowering.

D is Incorrect but plausible. For Condition 1, a trip of all Fuel Pool Cooling pumps will result in reduced RBCCW temperature due to heat no longer being removed from the Fuel Pool Heat Exchanger. DW Press and Temp will lower. The applicant may believe this will have to affect on OW Temp, however DW Pressure would not have risen.

ILT 10-1 NRC & AUDIT EXAM Page: 184 of 218 26 May 2011

EXAMINATION ANSWER KEY ILT 10-1 NRC RO Exam References to be None provided during exam:

Lesson Plan 2621.828.0.0032, Primary Containment Learning PCS-432, Interpret given control room and/or local Objectivel Primary Containment system indications and evaluate them in terms of limits and trends, using available data.

Question Source (New, Modified, Bank) I Modified If Bank or Modified:

VISION System/Question ID 666827 Question Source ILT 08-1 RO Audit Exam Memory or X Comprehension Fundamental 2:RI or Analysis Cognitive Knowledge Level NUREG 1021 Appendix B: Recognizing Interaction between systems (plural), including consequences and implications 55.41 I 5 I 55.43 I Facility operating characteristics during steady state 10CRF55 and transient conditions, including coolant chemistry, Content causes and effects of temperature, pressure and reactivity changes, effects of load changes, and operating limitations and reasons for these operating characteristics.

Justification for LORT questions with N/A KIA values < 3.0 Time to Complete: 1-2 minutes I Point Value: 1 System 10 No.: 295010 PRA: I NO Safety [8J Initial License Level 5

Function: D LORT ILT 10*1 NRC & AUDIT EXAM Page: 185 of 218 26 May 2011

EXAMINATION ANSWER KEY ILT 10-1 NRC RO Exam 62 10: 10-1 NR062 Points: 1.00 The reactor was at rated power when the following annunciator came into alarm:

  • TURBINE VAC/SEALS - COND VAC LO 25 INCHES The reactor operator lowered recirculation flow as directed by the associated RAP/ABN.

Condenser vacuum has now recovered to 25.8 in Hg and is steady. The Unit Supervisor then directs you to restore RPV pressure to the pre-event value by adjusting the EPR.

Which of th4~ following lists the required action and its effect?

Take the EPR RELAY POSITION control switch to (1) position which will cause MWeto (2) ill A. LOWER (t%) lower B. LOWER (t%) rise C. RAISE U%) rise D. RAISEU%) lower Answer: D IAnswer Explanation QID: 10*1 NR062 Question # I 62 I Developer / Date: JJR /7-11-11 Knowledge and Ability Reference Information Importance Rating K&A RO SRO 295002 Loss of Main Condenser Vac /3 AA1.06 - Ability to operate and/or monitor the following as they apply to LOSS OF MAIN 3.0 3.1 CONDENSER VACUUM: Reactor/turbine pressure regulating system Level I RO I Tier I 1 I Group I 2 ILT 10-1 NRC & AUDIT EXAM Page: 186of218 26 May 2011

EXAMINATION ANSWER KEY ILT 10-1 NRC RO Exam General 315.5 RAP..Q3c ABN..14 References 202.1 D is Correct. As power is reduced, the EPR relay position also goes down (proportional to turbine load). To raise RPV pressure back up, the turbine control valves must close down some. Lowering the EPR relay position even further will do this (Raise UOk>>. As the TCV close down some, RPV pressure will rise. Therefore, the EPR relay position must be taken to the RAISE position, which will cause turbine control valves to close further, causing RPV Explanation pressure to rise, control valves close and electrical output lowers.

All distractors are Incorrect but plausible since they either manipulate the switch in the incorrect direction or the plant effect is incorrect. The applicant may also confuse where RPV pressure was at following the power reduction compared to its initial value which will result in an incorrect answer.

References to be None provided durin~ exam:

Lesson Plan 2621.828.0.0051, Turbine Controls Learning TCS-10446, Identify and explain system operating Objective/ controls / indications under all plant operating conditions.

Question Source (New, Modified, Bank) I Bank If Bank or Modified:

VISION System/Question ID 606550 Question Source ILT 05-1 RO NRC Exam Memory or X Comprehension Fundamental 3:PEO Cognitive or Analysis Knowledge Level NUREG 1021 Appendix B: fredict an Event or

.outcome 55.41 I 7 I 55.43 I 10CRF55 Design, components, and functions of control and Content safety systems, including instrumentation, signals, interlocks, failure modes, and automatic and manual features.

ILT 10-1 NRC & AUDIT EXAM Page: 187 of 218 26 May 2011

EXAMINATION ANSWER KEY ILT 10-1 NRC RO Exam Justification for LORT questions with N/A KIA value~; < 3.0 Time to Complete: 1-2 minutes I Point Value: 1 System 10 No.: 295002 PRA: I NO Safet;, ~ Initial License Level 3

Function: o LORT ILT 10-1 NRC & AUDIT EXAM Page: 188of218 26 May 2011

EXAMINATION ANSWER KEY ILT 10-1 NRC RO Exam 63 ID: 10-1 NR063 Points: 1.00 Given the following conditions:

  • Plant is at rated power
  • 'A' CRD pump is NOT available for operation
  • 'B' CRD pump trips and CANNOT be restarted
  • Annunciator CHAR WTR PRESS LO is in alarm Which of the following conditions requires a Reactor Scram?

A. Two or more CRD high temperature alarms are received.

B. Two or more CRD accumulator trouble alarms are received.

C. Five minutes after the 'B' CRD pump trips one CRD pump is still NOT operating.

D. Five minutes after the 'B' CRD pump trips one CRD accumulator trouble alarm is received.

Answer: B IAnswer Explanation QID: 10-1 NR063 Question # I 63 I Developer 1 Date: JJR 1 7-11-11 Knowledge and Ability Reference Information Importance Rating K&A RO SRO 295022 Loss of CRD Pumps 11 AA2.01

  • Ability to determine and/or interpret 3.5 3.6 the following as they apply to LOSS OF CRD PUMPS: Accumulator pressure Level I RO I Tier 1 I Group I 2 Genera~

RAP-H2c References ILT 10-1 NRC & AUDIT EXAM Page: 189 of 218 26 May 2011

EXAMINATION ANSWER KEY ILT 10-1 NRC RO Exam B is Correct. lAW RAP-H2c, if Reactor pressure is >

850 psig (stem states plant is at rated power), CRD flow cannot be immediately established, and two or more CRD annumulator pressure alarms are received (accumulator pressure alarm will be received for loss of both CRD pumps), then manually scram the reactor. The applicant must be able to determine that two accumulator pressure alarms Explanation have been received in order to execute the procedure correctly. Two accumulator alarms are signified by the accumulator trouble annunciator accompanied by an accumulator rod block.

All distractors are Incorrect but plausible if the applicant does not recall actions required for a trip or both CRD pumps.

References to be None provided durin II exam:

Lesson Plan 2621.828.0.0011, CRD & Hydraulics Learning CRD-10449, State the function and interpretation of Objective/ system alarms, alone and in combination, as applicable in accordance with the system RAPS.

Question Source (New, Modified, Bank) I Bank If Bank or Modified:

VISION System/Question 10 505739 Question Source CRD-23 Memory or X Comprehension Fundamental Cognitive 1:P or Analysis Knowledge Level NUREG 1021 Appendix B: frocedure steps and cautions 10CRF55 55.41 I 10 I 55.43 I Content Administrative, normal, abnormal, and emergency operating procedures for the facility.

Justification for LORT que!stions with N/A KIA values < 3.0 Time to C,omplete: 1-2 minutes I Point Value: 1 System I[) No.: 295022 PRA: I NO Safety 1:8] Initial License Level 1

Function: o LORT ILT 10-1 NRC & AUDIT EXAM Page: 1900f218 26 May 2011

EXAMINATION ANSWER KEY ILT 10-1 NRC RO Exam 64 10: 10-1 NR064 Points: 1.00 Complete the following sentences:

The NE Corner Room water level can be CONFIRMED at the MAX SAFE value by (1) .

(2) are not considered OPERABLE at water level greater than MAX SAFE in the NE Corner Room.

A. (1) a valid 1-7 Sump HI LEVEL alarm.

(2) System 1 Core Spray Pumps B. (1) an EO reporting water level is at the RED LINE.

(2) System 1 Core Spray Pumps C. (1) a valid 1-7 Sump HI LEVEL alarm.

(2) System 1 Containment Spray Pumps D. (1) an EO reporting water level is at the RED LINE.

(2) System 1 Containment Spray Pumps Answer: D IAnswer Explanation QID: 10-1 NR064 Question # I 64 I Developer I Date: JJR 17-11-11 Knowledge and Ability Reference Information Importance Rating K&A RO SRO 295036 Secondary Containment High SumplArea Water Levell 5 2.4.35 - Emergency Procedures I Plan:

3.8 4.0 Knowledge of local auxiliary operator tasks during emergency and the resultant operational effects.

Level I RO I Tier 1 I Group I 2 General EOP User's SCC EOP References Guide ILT 10-1 NRC & AUDIT EXAM Page: 191 of 218 26 May 2011

EXAMINATION ANSWER KEY ILT 10-1 NRC RO Exam o is Correct. lAW the EOP User's Guide, the MAX SAFE level of 16" in the Comer Rooms is signified by a RED LINE (painted) on the wall in the room. An EO must be dispatched to verify level visually in the Corner Rooms. System 1 Containment Spray Pumps are located in the NE Corner Room.

Explanation All distractors are Incorrect but plausible. The 1-7 Sump Hi Level Alarm in New Radwaste signifies the water may exist in the comer rooms and requires entry into Secondary Containment Control, however this is a Max Normal value, not Max Safe. In addition, the operator may be confused by which System 1 components are in the NE Comer Room (Containment Spray Pumps or Core Spray Pumps).

References to be None provided durin;J exam:

Lesson Plan 2621.845.0.0057, Secondary Containment Control Learning SCC-1667, Based upon specific plant parameters Objectiv'e/ and conditions, determine if entry conditions into EOPs have been met and determine which EOPs are applicable to the conditions provided.

Question Source (New, Modified, Bank) I New If Bank or Modified: N/A VISION System/Question 10 Question Source Memory or X Comprehension Fundamental 1:P Cognitive or Analysis Knowledge 1:S Level NUREG 1021 Appendix B: f.rocedure steps and cautions; s.tructures and locations 10CRF55 55.41 I 10 I 55.43 I content.t; rallve, nonnal, abnonnal, and emergency erating procedures for the facility.

Justification for LORT questions with N/A KIA values < 3.0 Time to Complete: 1*2 minutes I Point Value: 1 System 10 No.: 295036 PRA: I NO Safety 5

121 Initial License Level Function: D LORT ILT 10-1 NRC & AUDIT EXAM Page: 192 of 218 26 May 2011

EXAMINATION ANSWER KEY ILT 10-1 NRC RO Exam 65 ID: 10-1 NR065 Points: 1.00 The plant was at 50% power when an event resulted in the crew inserting a manual scram due to lowering RPV water level. Plant conditions include the following:

  • RPV water level continues to lower lAW ABN-1, Reactor Scram, as RPV level continues to lower, at which point, if any, is the crew REQUIRED to perform the following actions below?
  • Exit ABN-1 (ie. stop controlling RPV water level lAW ABN-1) AND;
  • Enter and use the RPV Control- no ATWS EOP for level control A. When directed by the US ONLY.

B. When RPV water level is less than 138" ONLY.

C. ABN-1 will always be performed concurrently with EOPs.

D. When RPV water level is less than 138" AND directed by the US.

Answer: 0 IAnswer Explanation QID: 10-1 NR065 Question # I 65 I Developer I Date: JJR 17-11-11 Knowledge and Ability Reference Information Importance Rating K&A RO SRO 295009 Low Reactor Water Levell 2 2.4.8 - Emergency Procedures I Plan:

Knowledge of how abnormal operating 3.8 4.5 procedures are used in conjunction with EOP's.

Level I RO I Tier 1 I Group I 2 General!

ABN-1 References ILT 10-1 NRC & AUDIT EXAM Page: 193 of 218 26 May 2011

EXAMINATION ANSWER KEY ILT 10-1 NRC RO Exam o is Correct. lAW ABN-1, if RPV water level goes below 138" post scram, Notify the US, EXIT ABN-1, and Enter EOP level control when directed by the US.

Explanatic)n All distractors are Incorrect but plausible if the applicant is not familiar with the procedural requirements to transition from ABN-1 level control to level control lAW EOP support procedures.

References to be None provided durin ~ exam:

Lesson Plan 2621.882.0.0001, Reactor Scram Learning ABN-1, Perform actions required by ABN-1:

Objective/ Reactor Scram Question Source (New, Modified, Bank) I New If Bank or Modified: N/A VISION System/Question 10 Question Source Memory or X Comprehension Fundamental Cognitive 1:P or Analysis Knowledge Level NUREG 1021 Appendix B: erocedure steps and cautions 10CRF55 55.41 I 10 I 55.43 I Content Administrative, normal, abnormal, and emergency operating procedures for the facility.

Justification for LORT questions with N/A KIA value!, < 3.0 Time to Complete: 1-2 minutes I Point Value: 1 System ID No.: 295009 PRA: I NO Safety 2

IZI Initial License Level Function: D LORT ILT 10-1 NRC & AUDIT EXAM Page: 194 of 218 26 May 2011

EXAMINATION ANSWER KEY ILT 10-1 NRC RO Exam 66 ID: 10-1 NR066 Points: 1.00 A reactor startup is in progress with the following conditions:

  • The 'A' Feed String is in service on the 'A' Low Flow Reg Valve
  • One Turbine Bypass Valve is open An overload trip of the 'A' Reactor Feed Pump then occurs. lAW station procedures, which of the following actions are required by the URO?

A. Immediately insert a manual scram, then inform the US the scram was inserted due to approaching a scram setpoint.

B. Announce the 'A' Reactor Feed Pump tripped, pause to ensure the US received the communication, then immediately perform a Rapid Power Reduction.

C. Announce the 'A' Reactor Feed Pump tripped, immediately start another feed pump, then update the US on critical plant parameters once RPV level has stabalized.

D. Announce the 'A' Reactor Feed Pump tripped and intention to scram, pause to ensure the US received the communication, then immediately insert a Manual Scram.

Answer: D IAnswer Explanation QID: 10-1 NR066 Question # I 66 I Developer 1 Date: JJR 17-11-11 Knowledge and Ability Reference Information Importance Rating K&A RO SRO 2.1.39 - Knowledge of conservative decision 3.6 4.3 making p."actices" Level I RO I Tier 3 I Category I COO General OP-OC-101-111 ABN-17 ABN-1 References 1001 ILT 10-1 NRC & AUDIT EXAM Page: 195 of 218 26 May 2011

EXAMINATION ANSWER KEY ILT 10-1 NRC RO Exam o is Correct. The question stem provides a condition where a plant startup is in progress with one feed pump in operation. An overload trip of the feed pump will required a manual scram due to lowering RPV water level approaching the Low Level Scram Setpoint. ABN-17, Feedwater System Abnormal Conditions will direct inserting a manual scram. OP-OC-101-111-1001, Strategies For Successful Transient Mitigation, provides further conservative decision making guidance and directs the operator to announce that the 'A' Feed Pump has tripped, pause to ensure the US received the communication, then immediately insert a manual scram. ABN-1, Reactor Scram, also implies that a manual scram should be inserted BEFORE reaching Explanaticm an automatic scram setpoint which provides positive operator control over the plant.

A is Incorrect but plausible if the applicant does not recall the conservative decision making guidance provided in Strategies For Successful Transient Mitigation.

B is Incorrect. This distractor is plausible this this is the correct action for a single feed pump trip, however this situation implies the reactor had all 3 feed pumps running and was above 700/0 power.

C is Incorrect. This distractor is plausible if the applicant does not recall immediate actions of ABN 17 for feed pump trips.

References to be None provided durin;:J exam:

Lesson Plan 2621.830.0.0017, Conduct of Operations - Admin Learning 2.1.39, Knowledge of conservative decision making Objective/ practices.

Question Source (New, Modified, Bank) I New If Bank or Modified: N/A VISION System/Question 10 Question Source Memory or Cognitive X Comprehension Fundamental Level 1:P or Analysis Knowledge ILT 10-1 NRC & AUDIT EXAM Page: 196 of 218 26 May 2011

EXAMINATION ANSWER KEY ILT 10-1 NRC RO Exam NUREG 1021 Appendix B: Procedure steps and cautions 10CRF55 55.41 I 10 I 55.43 I Content Administrative, normal, abnormal, and emergency operating procedures for the facility.

Justification for LORT questions with N/A KIA values < 3.0 Time to Complete: 1-2 minutes I Point Value: 1 System ID No.: N/A PRA: I NO Safety [g] Initial License Level N/A Function: D LORT ILT 10-1 NRC & AUDIT EXAM Page: 197 of 218 26 May 2011

EXAMINATION ANSWER KEY ILT 10-1 NRC RO Exam 67 10: 10-1 NR067 Points: 1.00 The plant was at rated power when an event occurred. Indications and investigations revealed the following:

  • Battery Charger MG Set A Breaker has opened.
  • Battery A Main Breaker has opened.

Which of the following states the proper function of a DC Distribution System Automatic Transfer Switch under the given conditions?

The power to 125 VDC Bus (1) has automatically transferred to 125 VDC Bus (2) .

ill (2)

A. DC-F DC-C B. DC-1 DC-C C. DC-2 DC-B D. DC-E DC-B Answer: D IAnswer Explanation QID: 10-1 NR067 Question # I 67 I Developer 1 Date: JJR 17-11-11 Knowledge and Ability Reference Information Importance Rating K&A RO SRO 2.1.28

  • Knowledge of the purpose and function of major system components and 4.1 4.1 controls.

Level I RO I Tier 3 I Category I COO General RAP-9XF4e References ILT 10-1 NRC & AUDIT EXAM Page: 198 of 218 26 May 2011

EXAMINATION ANSWER KEY ILT 10-1 NRC RO Exam o is Correct. The question stem describes a loss of power to 125 VDC Bus DC-A (both the battery charger and battery become disconnected from the Bus). When this bus de-energizes, then automatic transfer switch DC-E swaps from DC-A as the source of input power to 125 VDC Bus DC-B.

A is Incorrect but plausible if the applicant does not Explanation recall Bus DC*F normally receives power from Bus DC*C, which is not affected by the loss of DC-A.

B is Incorrect but plausible if the applicant does not recall Bus DC-1 normally receives power from Bus DC-B, which is not affected by the loss of DC-A.

C is Incorrect but plausible if the applicant does not recall Bus DC-2 normally receives power from Bus DC-C, which is not affected by the loss of DC*A.

References to be None provided durin;J exam:

Lesson Plan 2621.830.0.0017, Conduct of Operations - Admin Learning 2.1.28, Knowledge of the purpose and function of Objective/ major system components and controls.

Question Source (New, Modified, Bank) I Bank If Bank or Modified:

VISION System/Question ID 606388 Question Source ILT 07*1 RO NRC Exam Memory or Comprehension X Fundamental Cognitive or Analysis 3:SPK Knowledge Level NUREG 1021 Appendix B: Solve a Problem using Knowledge and its meaning 55.41 I 7 I 55.43 I 10CRF55 Design, components, and functions of control and Content safety systems, including instrumentation, signals, interlocks, failure modes, and automatic and manual features.

Justification for LORT questions with N/A KIA values < 3.0 Time to Complete: 1*2 minutes I Point Value: 1 System 10 No.: N/A PRA: I NO Safety ~ Initial License Level N/A Function: o LORT ILT 10-1 NRC & AUDIT EXAM Page: 199 of 218 26 May 2011

EXAMINATION ANSWER KEY ILT 10-1 NRC RO Exam 68 10: 10-1 NR068 Points: 1.00 The reactor is in COLD SHUTDOWN and pre-startup evolutions are in progress.

The 'B' Reactor Recirculation Pump is being placed in service and is aligned as follows:

  • The MG set drive motor breaker is shut
  • The scoop tube is positioned at 100%
  • The WARM light has just illuminated Which one of the following describes what happens when the STRTINORM pushbutton is depressed?

A. The field breaker will close immediately and the scoop tube will remain at 100%.

B. The field breaker will close immediately and the scoop tube will start running back.

C. The scoop tube will start running back and the field breaker will close when the scoop tube reaches the low speed position.

D. The scoop tube will start running back and the field breaker will close when the scoop tube passes through the 40% to 30% range.

Answer: D IAnswer Explanation QID: 10-1 NR068 Question # I 68 I Developer I Date: JJR 17-11-11 Knowledge and Ability Reference Information Importance Rating K&A RO SRO 2.2.1 - Ability to perform pre-startup procedures for the facility, including operating those controls associated with plant 4.5 4.4 equipment that could affect reactivity.

Level I RO I Tier 3 I Category I EQC General 301.2 Referencles ILT 10-1 NRC & AUDIT EXAM Page: 200 of 218 26 May 2011

EXAMINATION ANSWER KEY ILT 10-1 NRC RO Exam D is Correct. The question stem provides a condition where a plant pre-startup evolutions are in progress with the 'B' Recirc Pump being placed in service. As soon as the STRT/NORM pushbutton is depressed the scoop tube begins to run back. When it reaches the 40-30% position, the field breaker will Explanation close and the recirc pump will start. The scoop tube will continue to run back to the low speed position.

All distractors are Incorrect but plausible if the applicant does not recall the correct startup sequence for Recirculation Pumps.

References to be None provided during exam:

Lesson Plan 2621.830.0.0018, Equipment Control- Admin Learning 2.2.1, Ability to perform pre-startup procedures for Objective/ the facility, including operating those controls associated with plant equipment that could affect reactivity.

Question Source (New, Modified, Bank) I Bank If Bank or Modified:

VISION System/Question ID 510894 Question Source ILT 05-1 RO NRC Exam Memory or X Comprehension Fundamental Cognitive 1:P or Analysis Knowledge Level NUREG 1021 Appendix B: J:!rocedure steps and cautions 10CRF55 55.41 I 10 I 55.43 I Content Administrative, normal, abnormal, and emergency operating procedures for the facility.

Justification for LORT questions with N/A KIA values < 3.0 Time to Complete: 1-2 minutes I Point Value: 1 System ID No.: N/A PRA: I NO Safety N/A 181 Initial License Level Functic)n: o LORT ILT 1Q..1 NRC & AUDIT EXAM Page: 201 of 218 26 May 2011

EXAMINATION ANSWER KEY ILT 10-1 NRC RO Exam 69 ID: 10-1 NR069 Points: 1.00 The plant is at rated power. You have just come in for day shift turnover and plant status includes the following:

Which of the following maintenance activities, if it resulted in tripping the breaker, will DIRECTLY impact the LCO for Tech Spec 3.2.C?

A. MCC 1B21 B. MCC 1B22 C. MCC 1B23 D. MCC 1B24 Answer: A IAnswer Explanation QID: 10-1 NR069 Question # I 69 I Developer 1 Date: JJR 17-11-11 Knowledge and Ability Reference Information Importance Rating K&A RO SRO 2.2.36 - Ability to analyze the effect of maintenance activities, such as degraded 3.1 4.2 power sources, on the status of limiting conditions for operations.

Level I RO I Tier 3 I Category I EQC Genera II Tech Spec 3.2.C BR 3004 She 3 References ILT 10-1 NRC & AUDIT EXAM Page: 202 of 218 26 May 2011

EXAMINATION ANSWER KEY ILT 10-1 NRC RO Exam A is Correct. The question stem provides a condition where the 'A' Standby Liquid Control (SLC)

Pump was removed from service and the plant is in a 7 day LCO. The 'B' SLC Pump is powered from MCC 1B21. A loss of this MCC will result in a loss of all SLC pumps and the plant must be brought to a COLD SHUTDOWN condition within 24 hrs. It is not Explanation required knowledge for the RO applicant to know what the LCO is, however it is RO required knowledge to recognize that the LCO was impacted by the loss of the redundant SLC pump.

All distractors are Incorrect but plausible if the applicant doesn't recall the correct power supply to the B SLC pump.

References to be None provided durin ~ exam:

Lesson Plan, 2621.850.0.0090, Overview and Highlights of Technical Specifications Learning Objective/ TSX-1920, Given various plant indications, evaluate the indications to determine plant status with respect to operating license and technical specifications.

Question Source (New, Modified, Bank) I New If Bank or Modified: N/A VISION System/Question ID Question ~Source Memory or Comprehension X Fundamental Cognitive or Analysis 2:DR Knowledge Level NUREG 1021 Appendix B: Describing or recognizing Relationships 10CRF55 55.41 I 10 I 55.43 I Content Administrative, normal, abnormal, and emergency operating procedures for the facility.

Justification for LORT questions with N/A KIA values < 3.0 Time to Complete: 1-2 minutes I Point Value: 1 System I[) No.: N/A PRA: I NO Safety N/A 181 Initial License Level Function: o LORT ILT 10-1 NRC & AUDIT EXAM Page: 203 of 218 26 May 2011

EXAMINATION ANSWER KEY ILT 10-1 NRC RO Exam 70 ID: 10-1 NR070 Points: 1.00 Given the following plant conditions:

  • The plant is operating at 100% power.
  • Operators note rising Off-Gas Radiation Levels on RN-05E & 05F [Off Gas Channel 1 & 2 radiation monitors on 1R].
  • At 10:30 both monitors read 100 mr/hr
  • The readings are rising at the rate of 150 mr/hr every 5 minutes Based on these conditions, when will the augmented off-gas (AOG) system automatically isolate? (Use actual setpoint values)

DmeofAOG Isolation A. 11:00 B. 11 :15 C. 11:30 D. 12:15 Answer: B IAnswer Explanation QID: 10-1 NR070 Question # I 70 I Developer I Date: JJR 17-11-11 Knowledge and Ability Reference Information Importance Rating K&A RO SRO 2.3.15 - Knowledge of radiation monitoring systems, such as fixed radiation monitors and 2.9 3.1 alarms, portable survey instruments, personnel monitoring equipment, etc.

Level I RO I Tier 3 I Category I RPT General RAP*10F1c References ILT 10-1 NRC & AUDIT EXAM Page: 204 of 218 26 May 2011

EXAMINATION ANSWER KEY ILT 10-1 NRC RO Exam B is Correct. V-7-31/29 and OG-AOV-001A(B) isolate the off-gas system at the stack after a 14-15 minute time delay with coincident upscale trips of both channels at 1000 mr/hr (Off Gas Hi-Hi alarm). 1000 mr/hr is reached at 1100, at 1115 AOG isolation occurs. Increasing AlE off-gas radiation levels is an Explanation indication of leaking fuel (cladding failure). (TS value for high radiation in off-gas is 2000 mr/hr).

All distractors are Incorrect but plausible if the applicant doesn't recall the correct setpoint value for the Off Gas Hi-Hi alarm or doesn't recall the isolation logic.

References to be None provided during exam:

Lesson Plan 2621.828.0.033A, Plant Radiation Monitoring System Learning Objectivel RAD-10453, Explain or describe how this system is interrelated with other plant systems.

Question Source (New, Modified, Bank) I Bank If Bank or Modified:

VISION System/Question ID 607956 Question Source ILT 07-1 RO Comp #1 Memory or Fundamental Comprehension X or Analysis 2:RI Cognitive Knowledge Level NUREG 1021 Appendix B: Recognizing Interaction between systems (plural), including consequences and implications 55.41 I 7 I 55.43 I 10CRF55 Design, components, and functions of control and Content safety systems, including instrumentation, signals, interlocks, failure modes, and automatic and manual features.

Justification for LORT questions with N/A KIA values < 3.0 Time to Complete: 1-2 minutes I Point Value: 1 System 10 No.: N/A PRA: I NO Safety N/A I:8l Initial License Level Function: D LORT ILT 10-1 NRC & AUDIT EXAM Page: 205 of 218 26 May 2011

EXAMINATION ANSWER KEY ILT 10-1 NRC RO Exam 71 10: 10..1 NR071 Points: 1.00 The plant was at power when it was determined that an Operator needed to enter an area inside the RCA where the dose rate was 1100 mrem/hr. (This was a non emergency evolution.)

lAW RP-AA-460, Controls for High and Very High Radiation Areas, which of the following ane correct in order to enter the room to isolate the leak?

  • Prior to entry, a briefing must be conducted by (1) .
  • The key for entry will be issued by (2) .

(1) (2)

A. OPS OPS B. RP RP C. RP OPS D. OPS RP Answer: B IAnswer Explanation QID: 10-1 NR071 Question # I 71 I Developer 1 Date: JJR 17-11-11 Knowledge and Ability Reference Information Importance Rating K&A RO SRO 2.3.12 - KI10wiedge of Radialogical Safety Principles pertaining to licensed operator duties, such as containment entry 3.2 3.7 requirements, fuel handling responsibilities, access to locked high-radiation areas, aligning filters, etc.

Level I RO I Tier 3 I Category I RPT General RP*AA-460 References ILT 10-1 NRC & AUDIT EXAM Page: 206 of 218 26 May 2011

EXAMINATION ANSWER KEY ILT 10-1 NRC RO Exam B is Correct. lAW the reference, the area will be classified as a locked high radiation area ( .?!1000 mrem at 30 cm in 1 hour1.157407e-5 days <br />2.777778e-4 hours <br />1.653439e-6 weeks <br />3.805e-7 months <br />). Also, the procedure requires a briefing by RP, with keys issued by RP.

The procedure does account for master keys to be Explanation used in times of an emergency, but the question stem states that this is not an emergency.

All distractors are Incorrect but plausible. They provide either the incorrect area designation, incorrect briefing provider or incorrect key issuer.

References to be None provided durin ~ exam:

Lesson Plan 2621.830.0.0015, Radiation Control- Admin Learning 2.3.12, Knowledge of Radialogical Safety Principles Objectivel pertaining to licensed operator duties, such as containment entry requirements, fuel handling responsibilities, access to locked high-radiation areas, aligning filters, etc.

Question Source (New, Modified, Bank) I Bank If Bank or Modified:

VISION S)fstem/Question 10 667552 Question Source ILT 08-1 RO Audit Exam Memory or X Comprehension Fundamental Cognitive 1:P or Analysis Knowledge Level NUREG 1021 Appendix B: erocedure steps and cautions 10CRF55 55.41 I 12 I 55.43 I Content Radiological safety principles and procedures.

Justification for LORT questions with N/A KIA values < 3.0 Time to Complete: 1-2 minutes I Point Value: 1 System ID No.: N/A PRA: I NO Safety ~ Initial License Level N/A Functhln: D LORT ILT 10-1 NRC & AUDIT EXAM Page: 207 of 218 26 May 2011

EXAMINATION ANSWER KEY ILT 10-1 NRC RO Exam 72 ID: 10-1 NR072 Points: 1.00 Which of the following states the definition of the following EOP symbol?

This designates a Support Procedure ....

A. as high Importance.

B. may override plant Interlocks.

C. could cause an Increase in off-site release rate.

D. to be completed Immediately, and without delay, once started.

Answer: B IAnswer Ex,planation QID: 10-1 NR072 Question # I 72 I Developer 1 Date: JJR 17..11-11 Knowledge and Ability Reference Information Importance Rating K&A RO SRO 2.4.19 .. Knowledge of EOP layout, symbols, 3.4 4.1 and icons.,

Level I RO I Tier I 3 I Category I EOP ILT 10-1 NRC & AUDIT EXAM Page: 208 of 218 26 May 2011

EXAMINATION ANSWER KEY ILT 10-1 NRC RO Exam General EOP User's References Guide B is Correct. lAW the EOP User's Guide, the symbol designates that a support procedure will override plant interlocks.

Explanatilon All distractors are Incorrect but plausible if the applicant does not recall the correct definition for this EOP symbol.

References to be None provided durin ~ exam:

Lesson Plan 2621.830.0.0016, Emergency Procedures / Plan ..

Admin Learning Objective/ 2.4.19, Knowledge of EOP layout, symbols, and icons.

Question Source (New, Modified, Bank) I Bank If Bank or Modified:

VISION System/Question 10 608088 Question Source ILT 07-1 RO Audit Exam Memory or X Comprehension Cognitive Fundamental 1:0 or Analysis Level Knowledge NUREG 1021 Appendix B: D.efinitions 10CRF55 55.41 I 10 I 55.43 I Content Administrative, normal, abnormal, and emergency operating procedures for the facility.

Justification for LORT questions with N/A KIA values < 3.0 Time to Complete: 1..2 minutes I Point Value: 1 System 10 No.: N/A PRA: I NO Safety 1:81 Initial License Level N/A Function: D LORT ILT 10-1 NRC & AUDIT EXAM Page: 209 of 218 26 May 2011

EXAMINATION ANSWER KEY ILT 10-1 NRC RO Exam 73 10: 10-1 NR073 Points: 1.00 The plant was at rated power when RPV pressure spiked to 1055 psig. The following plant conditions currently exist:

  • RPV water level indicates 155"
  • The Operator has depressed both MANUAL SCRAM push buttons, and has placed the REACTOR MODE SELECTOR switch in SHUTDOWN
  • APRMs indicate> 2% power lAW OP-OC-101-111-1001, Strategies for Successful Transient Mitigation, which of the following acltions is immediately required under the conditions given, and the correct reason for the action?

A. Place ALL Recirculation MG Set DRIVE MOTOR switches to STOP to reduce reactor power.

B. Depress the ALT ROD INJECTION INITIATION pushbutton to energize the ARI solenoids.

C. Place the SLC keylock switch in FIRE SYS 1 (or 2) to initiale the Standby Liquid Contrtol System.

D. Place the ROPS switch in BYPASS since over-fill protection will NOT function as designed under the conditions that currently exist.

Answer: B IAnswer Explanation QID: 10-1 NR073 Question # I 73 I Developer 1 Date: JJR 17-11-11 Knowledge and Ability Reference Information Importance Rating K&A RO SRO 2.4.1 - Kn()wledge of EOP entry conditions and 3.6 4.3 immediate action steps.

Level I RO I Tier 3 I Category I EOP General OP-OC-101-111- EOP User's References 1001 Guide ILT 10-1 NRC & AUDIT EXAM Page: 210 of 218 26 May 2011

EXAMINATION ANSWER KEY ILT 10-1 NRC RO Exam B is Correct. The plant was at power when an RPV over-pressure event occurred. The reactor should have scrammed at 1045 psig, but indications show that an ATWS is in-progress. lAW OP-OC-101-111 1001, and RPV Control - with A TWS EOP, the Panel 4F Operator shall depress the scram button, place the mode switch in shutdown, and initiate ARlo The ARI solenoids are de-energized and are inter-locked de-energized until manually initiated, or automatically from RPV high pressure (1090 psig) or from RPV water level 10-10 (90"). When ARI is initiated, vent valve solenoids and the ARI isolation valve solenoids energize to isolate/vent the scram air header. This allows the scram valves to open to Explanation allow the CRO hydraulics to insert more control rods.

A is Incorrect but plausible. The same reference directs the panel operator to take recirculation flow to minimum - not to trip the pumps/MG.

C is Incorrect but plausible. Initiating SLC is only directed from the A TWS EOP.

o is Incorrect but plausible. The reference also directs the Operator to place ROPS in BYPASS, but the EOP Users Guide states this is done to prevent an inadvertent FW pump trip - not because ROPS is not designed during an ATWS.

References to be None

_provided during exam:

Lesson Plan 2621.845.0.0053, RPV Control- with ATWS Learning EWA-3052, State the plant conditions requiring Objective/ entry into RPV Control - with A TWS.

Question Source (New, Modi'fied, Bank) I Bank If Bank or Modified:

VISION System/Question 10 718212 Question Source ILT 09-1 RO Audit Exam Memory or X Comprehension Fundamental Cognitive 1:P or Analysis Knowledge Level NUREG 1021 Appendix B: erocedure steps and cautions ILT 10-1 NRC & AUDIT EXAM Page: 211 of218 26 May 2011

EXAMINATION ANSWER KEY ILT 10-1 NRC RO Exam 10CRF55 55.41 I 10 I 55.43 I Content Administrative, normal, abnormal, and emergency operating procedures for the facility.

Justification for LORT quelstions with N/A KIA values < 3.0 Time to Complete: 1-2 minutes I Point Value: 1 System 10 No.: N/A PRA: I NO Safety N/A 181 Inilial License Level Function: o LORT ILT10-1 NRC & AUDIT EXAM Page: 212 of 218 26 May 2011

EXAMINATION ANSWER KEY ILT 10-1 NRC RO Exam 74 10: 10*1 NR074 Points: 1.00 A plant startup is in-progress with the following conditions:

  • The REACTOR MODE switch is in STARTUP, with control rod withdrawals in progress.
  • IRMs 11,12,15,16,18 read 72-74 % of scale on Range 1.
  • IRMs 13, 14, and 17 read 10 % of scale on Range 2.

A malfunction in the IRM drive circuitry caused IRM 13 to withdraw to the full-out position.

Which of the following states the effect on the plant and the required Operator actions to continue withdrawing control rods?

A. There are panel annunciators ONLY; withdrawing control rods may continue without any other control panel manipulations.

B. There are panel annunciators and a rod block from IRM downscale ONLY; bypassing the IRM is required to continue withdrawing control rods.

C. There are panel annunciators and a rodblock from IRM downscale AND IRM detector position; bypassing the IRM is required to continue withdrawing control rods.

D. There are panel annunciators, a rodblock and a 1/2 scram; bypassing the IRM and resetting the 1/2 scram is required to continue withdrawing control rods.

Answer: C IAnswer Explanation QID: 10-1 NR074 Question # I 74 I Developer 1 Date: JJR 17-11-11 Knowledge and Ability Reference Information Importance Rating K&A RO SRO 2.2.2 - Ability to manipulate the console controls as required to operate the facility 4.6 4.1 between shutdown and designated power levels.

ILT 10-1 NRC & AUDIT EXAM Page: 213 of 218 26 May 2011

EXAMINATION ANSWER KEY ILT 10-1 NRC RO Exam Level I RO I Tier 3 I Category I EQC General RAP*H7a 402.4 References C is Correct. The following IRM parameters provide roblocks only (no scram input): IRM downscale (in REFUAL and STAR1"UP; bypassed in Range 1 or in RUN), detector not fully inserted (bypassed in RUN),

and IRM high (bypassed in RUN). When the IRM comes off the full-in position, a rodblock is instituted (plus panel annunciators). It is expected that the IRM will also go downscale as it drives to the fully withdrawn position (downscale also gives a rodblock except in Range 1). There are no 1/2 scrams from these conditions. Therefore, to Explanati(m continue to move control rods, IRM 13 (which is instituting a rodblock both from downscale and IRM position) must be bypassed.

A is Incorrect but plausible if the applicant does not recall that rod blocks should exist.

B is Incorrect but plausible if the applicant does not recall that a rodblock should exist for IRM detector position also.

o is Incorrect but plausible if the applicant does not recall that there should not be a 1/2 scram for these conditions.

References to be None provided durin~ exam:

Lesson Plan 2621.828.0.0029, Nuclear Instrumentation Learning NIS-10445, Given a set of system indications or Objective/ data, evaluate and interpret them to determine limits, trends and system status.

Question Source (New, Modified, Bank) I Bank If Bank or Modified:

VISION System/Question 10 510857 Question Source ILT 05-1 RO NRC Exam Memory or Comprehension X Fundamental Cognitive or Analysis 3:PEO Knowledge Level NUREG 1021 Appendix B: f.redict an Event or Outcome ILT 10-1 NRC & AUDIT EXAM Page: 214 of 218 26 May 2011

EXAMINATION ANSWER KEY ILT 10-1 NRC RO Exam 10CRF55 55.41 I 6 I 55.43 I Content Design, components, and functions of reactivity control mechanisms and instrumentation.

Justification for LORT questions with N/A KIA values < 3.0 Time to Complete: 1-2 minutes I Point Value: 1 System ID No.: N/A PRA: I NO Safety ~ Initial License Level N/A Function: o LORT ILT 10-1 NRC & AUDIT EXAM Page: 215 of 218 26 May 2011

EXAMINATION ANSWER KEY ILT 10-1 NRC RO Exam 75 10: 10-1 NR075 Points: 1.00 The plant was at rated power when a large break LOCA and A TWS occur. Plant conditions include the following:

  • RPV water level is -18" and lowering Which ONE of the following sources of water does the RPV Control - with ATWS EOP recommend as the LOWEST priority (LAST alternative) for makeup to the RPV AND lAW the EOP User's Guide, what is a basis for this priority?

A. Fire Water via the Core Spray system due to its corrosive affect on core components.

B. Condensate Transfer via the Core Spray system due to its low discharge head and flow rate.

C. The Feedwater/Condensate system since it is secured while Terminating and Preventing Injection.

D. The Core Spray System since it will result in large quantities of cold, unborated water injecting inside the core shroud.

Answer: D IAnswer Explanation QID: 10-1 NR075 Question # I 75 I Developer I Date: JJR /7-11-11 Knowledge and Ability Reference Information Importance Rating K&A RO SRO 2.4.22 - Knowledge of the bases for prioritizing safety functions during abnormal/emergency 3.6 4.4 operations.

Level I RO I Tier 3 I Category I EOP General RPVC -with EOP User's References ATWSEOP Guide ILT 10-1 NRC & AUDIT EXAM Page: 216 of 218 26 May 2011

EXAMINATION ANSWER KEY ILT 10*1 NRC RO Exam D is Correct. lAW the EOP User's Guide, the Core Spray System is used only after all other sources of injection have proven inadequate for restoring and maintaining RPV level above -20 in. The Core Spray System has two significant drawbacks:

1. Injection into the RPV occurs inside the shroud, not outside the shroud, where the relatively cold, unborated water injected by Core Spray would not have an opportunity to mix with the warm, borated water in the lower plenum before reaching the core.
2. Since the Core Spray injection valves are unable to be remotely throttled, injection into the RPV cannot be readily controlled, resulting in large Explanaticm quantities of relatively cold, un borated water entering the core region directly from the Core Spray sparger.

The combination of these two factors makes the choice of operating the Core Spray System the least desirable alternative for providing RPV injection when the Reactor may be critical or just barely subcritical. However, the undesirable consequences of prolonged uncovering of the core and loss of adequate core cooling requires operation of the Core Spray System even at the risk of a Reactor power excursion.

All distractors are Incorrect but plausible due to being actual sources of makeup water. The applicant may not recall the bases for makeup water priority or confuse the priority.

References to be None provided during exam:

Lesson Plan 2621.845.0.0053, RPV Control - with ATWS Learning EWA-3054, Describe the parameters monitored and Objective/ controlled by the RPV Control EOP.

Question Source (New, Modified, Bank) I New If Bank or Modified: N/A VISION System/Question ID Question Source ILT 10-1 NRC & AUDIT EXAM Page: 217 of 218 26 May 2011

EXAMINATION ANSWER KEY ILT 10-1 NRC RO Exam Memory or X Comprehension Cognitive Fundamental 1:B or Analysis Level Knowledge NUREG 1021 Appendix B: Bases or purpose 10CRF55 55.41 I 10 I 55.43 I Content Administrative, normal, abnormal, and emergency operating procedures for the facility.

Justification for LORT questions with N/A KIA values < 3.0 Time to Complete: 1-2 minutes I Point Value: 1 System ID No.: N/A PRA: I NO Safety ~ Initial License Level N/A Function: D LORT ILT 10-1 NRC & AUDIT EXAM Page: 218 of 218 26 May 2011

EXAMINATION ANSWER KEY ILT 10-1 NRC SRO Exam 1 ID: 10-1 NSR01 Points: 1.00 The plant was at 50% power when an event occurred due to electrical bus losses.

Present plant conditions are as follows:

  • All Af~PM drawers have failed downscale
  • GD1 and GC1 are open
  • Both Isolation condensers are in-service
  • RPV Pressure is 1000 psig and stable
  • Drywell pressure is 3.1 psig and steady
  • Torus water temperature is 90 OF and steady Which of the following actions is required?

A. Place the ADS TIMER switches to BYPASS per RPV Control - with A TWS EOP Level/Power Leg.

B. Transfer RPV pressure control to the EMRVs ONLY per RPV Control with ATWS EOP Pressure Leg.

C. Place one Containment Spray System in the Torus Cooling Mode per Primary Containment Control EOP Torus Temperature Leg.

D. Maintain RPV water level 138" - 175" with Feedwater/Condensate and/or CRD per RPV Control - no ATWS EOP Level Leg.

Answer: A Answer Explanation I QID: 10-1 NSR01 Question # I 1 I Developer I Date: JJR I 7-11-11 ILT 10-1 NRC & AUDIT EXAM Page: 1 of 81 26 May 2011

EXAMINATION ANSWER KEY ILT 10-1 NRC SRO Exam Knowledge and Ability Reference Information Importance Rating K&A RO SRO 295006 SCRAM /1 AA2.05

  • Ability to determine and/or interpret the following as they apply to SCRAM :

4.6 4.6 Whether a reactor SCRAM has occurred Level I SRO I Tier 1 I Group I 1 General RPVC - with EOP User's ReferenCE!S A TWS EOP Guide A is Correct. The question stem describes an event where a scram should have occurred (loss of power to all APRMs gives INOPs to both RPS 1 and 2; also indications that the generator has tripped), but both primary indicators of the reactor shutdown, APRMs and control rod position indication, are lost. But with both isolation condensers in-service and reactor pressure constant at 1000 psig, then all steam generated is going to the ICs (6%) and the RPV is not cooling down. Therefore, reactor power is 6%. Thus, an ATWS has occurred with power at 6%. Placing ADS in bypass is required in the level/power leg of the ATWS EOP.

B is Incorrect but plausible. The A TWS EOP allows Explanation RPV pressure with ICs or EMRVs, or other systems, but there is no requirement to transfer from one method that is working OK to another.

C is Incorrect but plausible. The stem also provides entry into the primary containment control EOP on DW pressure. Torus cooling is initiated to maintain torus temperature < 95 of. The stem says it is 90 OF and steady - thus there is no need to start it.

D is Incorrect. The RPV water level (answer D) is found both in the RPV Control- w/o ATWS EOP, and in the ATWS EOP. To control in the normal band in the ATWS EOP, reactor power must be < 2%. As stated, power is above this valve.

References to be None provided during exam:

ILT 10-1 NRC & AUDIT EXAM Page: 2 of 81 26 May 2011

EXAMINATION ANSWER KEY ILT 10-1 NRC SRO Exam

.Lesson Plan 2621.845.0.0053, RPV Control - with ATWS Learning EWA-2257, Given the EOP, describe in detail each Objective/ step/statement, including the technical basis, and how to verify or perform each step.

Question Source (New, Modified, Bank) I Bank If Bank or Modified:

VISION System/Question 10 609216 Question Source ILT 07-1 SRO Audit Exam Memory or X Comprehension Fundamental 3:SPK Cognitive or Analysis Knowledge Level NUREG 1021 Appendix B: Solve a eroblem using Knowledge and its meaning 55.41 I I 55.43 I 5 10CRF55 Assessment of facility conditions and selection of Content appropriate procedures during normal, abnormal, and emergency situations.

Justification for LORT quelstions with N/A KIA values < 3.0 Time to Complete: 1-2 minutes I Point Value: 1 System 10 No.: 295006 PRA: I NO Safety 1

r8:I Initial License Level Function: D LORT ILT 10-1 NRC & AUDIT EXAM Page: 30f81 26 May 2011

EXAMINATION ANSWER KEY ILT 10-1 NRC SRO Exam 2 ID: 10*1 NSR02 Points: 1.00 The plant was at rated power when an event occurred. The Operator reports the following observations:

  • BUS 1B CNTRL DC LOST has alarmed
  • BUS 1D CNTRL DC LOST has alarmed
  • ALL Isolation Condenser A valves on Panel 1F/2F indicate green light on
  • Annunciator DC-D PWR XFER has NOT alarmed Which ONE of the following is the cause for the indications listed above AND which of the following shall the SRO direct?

NOTE:

ABN-53 is DC A and Panel Failures ABN-54 is DC Band Panel/MCC Failures ABN-55 is DC C and Panel/MCC Failures A. lAW ABN-54, direct an Operator to manually align DC-1 transfer switch to DC-A.

B. lAW ABN-53, direct an Operator to manually align DC-E transfer switch to DC-B.

C. lAW ABN-54, direct an Operator to manually align DC-D transfer switch to DC-A.

D. lAW ABN-55, direct an Operator to manually operate supply and load breakers at DC-2 as required.

Answer: C Answer Explanation I QID: 10-1 NSR02 Question # I 2 I Developer I Date: JJR 17*11*11 Knowledge and Ability Reference Information Importance Rating K&A RO I SRO ILT 10-1 NRC & AUDIT EXAM Page: 40f81 26 May 2011

EXAMINATION ANSWER KEY ILT 10-1 NRC SRO Exam 295004 Partial or Total Loss of DC Pwr' 6 AA2.01 - Ability to determine and/or interpret the following as they apply to PARTIAL OR 3.2 3.6 COMPLETIE LOSS OF D.C. POWER: Cause of partial or complete loss of D.C. pOwer Level I SRO I Tier 1 I Group I 1 General R References ABN 54 C is Correct. The plant was at rated power when DC control power was lost to Bus 1B & 1D. This DC power comes from 125 VDC Bus DC-B. Thus, there is a loss of DC-B and ABN 54 applies. The provided R

information also states that DC-D & DC-E did not transfer to their alternate DC supply. Of these, only DC-D is fed from DC-B. The question stem also states that the valve positions for Isolation Condenser A indicate their normal positions. Two of the valves are powered by DC-1, which is fed by DC B. Since the indications do show valve positions, then DC-1 has transferred to its alternate DC supply (DC-A). Thus, DC-B has been lost and DC-D did not Explanaticm auto transfer. lAW ABN-54, manually performing the transfer of DC-D is correct.

A is Incorrect but plausible. ABN-54 does direct manually transferring the power supply for DC-1 if it didn't auto transfer. From the indications provided, it did auto transfer.

B is Incorrect but plausible. DC-E is normally powered from DC-A and the alternate supply is DC-B.

But since DC-A has not lost power, performing actions lAW ABN-53 is not appropriate.

D is Incorrect but plausible since the action in answer D is correct lAW ABN-55, but this ABN will not be entered under the given conditions since DC C has not lost power.

References to be None provided during exam:

Lesson Plan 2621.828.0.0012, DC Distribution Learning DCO-10445, Given a set of system indications or Objective' data, evaluate and interpret them to determine limits, trends and system status.

I Question Source (New, Modified, Bank) Bank ILT 10-1 NRC & AUDIT EXAM Page: 5 of 81 26 May 2011

EXAMINATION ANSWER KEY ILT 10-1 NRC SRO Exam If Bank or Modified:

VISION System/Question 10 718366 Question Source ILT 09-1 SRO NRC Exam Memory or X Comprehension Fundamental 3:SPK Cognitive or Analysis Knowledge Level NUREG 1021 Appendix B: S.olve a Problem using Knowledge and its meaning 55.41 I I 55.43 I 5 10CRF55 Assessment of facility conditions and selection of Content appropriate procedures during normal, abnormal, and emergency situations.

Justification for LORT questions with N/A KIA values < 3.0 Time to Complete: 1-2 minutes I Point Value: 1 System 10 No.: 295004 PRA: I NO Safety ~ Initial License Level 6

Functi()n: D LORT ILT 10-1 NRC & AUDIT EXAM Page: 6 of 81 26 May 2011

EXAMINATION ANSWER KEY ILT 10-1 NRC SRO Exam 3 ID: 10-1 NSR03 Points: 1.00 The plant is at rated power. An event then occurred and control room indications now include the following:

  • All SOV 'A' Train valve indications are extinguished
  • All STABALIZER VALVES SELECT NC19 valve position indications are extinguished
  • All HOTWELL LEVEL meters indicate downscale Which ONE of the following is the cause for the indications listed AND what shall the SRO direct?

NOTE:

ABN-54 is DC Band Panel/MCC Failures ABN-55 is DC C and Panel/MCC Failures ABN-58 is Instrument Power Failures The cause for the indications listed is a loss of ...

A. VACP-1. lAW ABN-58 direct manually starting the Standby Gas Treatment System.

B. DC-D. lAW ABN-54, direct manually aligning Auto Transfer Switch for DC-D to DC-A.

C. IP-4A. lAW ABN-58, direct adjusting Reactor Recirculation Flow for required power changes.

D. DC-2. lAW ABN-55 direct manually operating Isolation Condenser 'B' DC valves if required for operation.

Answer: C Answer Explanation I QID: 10-1 NSR03 Question # I 3 I Developer 1 Date: JJR 17-11-11 Knowledge and Ability Reference Information ILT 10-1 NRC & AUDIT EXAM Page: 7 of 81 26 May 2011

EXAMINATION ANSWER KEY ILT 10~1 NRC SRO Exam Importance Rating K&A RO SRO 295003 Partial or Complete Loss of AC / 6 AA2.05

  • Ability to determine and/or interpret the following as they apply to PARTIAL OR 3.9 4.2 COMPLE1'E LOSS OF A.C. POWER: Whether a partial or complete loss of A.C. power has occurred Level I SRO I Tier 1 I Group I 1 General ABN*58 References C is Correct. The question stem provides indications of a loss of IP-4A. lAW ABN-58, Instrument Power Failures, reactor power will be required to be manipulated using Reactor Recirculation Flow due to the Reactor Manual Control System losing power. In order to examine the KIA, the applicant must correctly diagnose a loss of an AC bus has occurred, not DC. The SRO must Explanation then direct an action required by each ABN.

All distractors are Incorrect but plausible. It is RO knowledge to diagnose which bus is lost, however it is SRO Only knowledge to know what action to direct lAW the applicable ABN. A choice of two Vital AC buses and two DC buses are given as choices in order for the applican to determine whether a partial or complete loss of AC power has occurred lAW the KIA.

References to be None provided durin~ exam:

Lesson Plan 2621.828.0.0056, Vital AC Distribution Learning VAC-10445, Given a set of system indications or Objective/ data, evaluate and interpret them to determine limits, trends and system status.

Question Source (New, Modified, Bank) I New If Bank or Modified: N/A VISION System/Question 10 Question Source ILT 10-1 NRC & AUDIT EXAM Page: 8 of 81 26 May 2011

EXAMINATION ANSWER KEY ILT 10-1 NRC SRO Exam Memory or X Comprehension Fundamental 3:SPK Cognitive or Analysis Knowledge Level NUREG 1021 Appendix B: Solve a eroblem using Knowledge and its meaning 55.41 I I 55.43 I 5 10CRF55 Assessment of facility conditions and selection of Content appropriate procedures during normal, abnormal, and emergency situations.

Justification for LORT questions with N/A KIA values < 3.0 Time to Complete: 1-2 minutes I Point Value: 1 System ID No.: 295003 PRA: I NO Safety [81 Initial License Level 6

Function: o LORT ILT 10-1 NRC & AUDIT EXAM Page: 9 of 81 26 May 2011

EXAMINATION ANSWER KEY ILT 10-1 NRC SRO Exam 4 10: 10-1 NSR04 Points: 1.00 A plant startup is in progress with the following:

  • EMRV testing is in progress
  • Torus average water temperature is rising Which one of the following describes the Torus average water temperature limits during and after the test, in accordance with Technical Specifications?

Torus average water temperature shall NOT exceed (1) during performance of the test and must be reduced below the normal power operation limit within (2)

Ul A. 12 hours1.388889e-4 days <br />0.00333 hours <br />1.984127e-5 weeks <br />4.566e-6 months <br />

8. 12 hours1.388889e-4 days <br />0.00333 hours <br />1.984127e-5 weeks <br />4.566e-6 months <br />
c. 24 hours2.777778e-4 days <br />0.00667 hours <br />3.968254e-5 weeks <br />9.132e-6 months <br /> D. 24 hours2.777778e-4 days <br />0.00667 hours <br />3.968254e-5 weeks <br />9.132e-6 months <br /> Answer: D Answer Explanation I QID: 10-1 NSR04 Question # I 4 I Developer / Date: JJR / 7-11-11 Knowledge and Ability Reference Information Importance Rating K&A RO SRO 295026 Suppression Pool High Water Temp./ 5 2.2.38 - Knowledge of conditions and 3.6 4.5 limitations in the facility license.

Level I SRO I Tier 1 I Group I 1 General TS 3.5.A.1 References ILT 10-1 NRC & AUDIT EXAM Page: 10 of 81 26 May 2011

EXAMINATION ANSWER KEY ILT 10-1 NRC SRO Exam o is Correct. lAW TS 3.5.A.1.c.(2), During testing which adds heat to the suppression pool, the water temperature shall not exceed 10F above the normal Power Operation limit (95F). In connection with such testing, the pool temperature must be reduced below the Power Operation limit within 24 hours2.777778e-4 days <br />0.00667 hours <br />3.968254e-5 weeks <br />9.132e-6 months <br />.

A is Incorrect. This distractor is plausible if the applicant does not recall the Max power limit value of 95F and that temperature is allowed to be 10F above that limit. In addition, 24 hrs are allowed to Explanation return pool temperature below the Power Operation limit, not 12 hrs.

B is Incorrect. This distractor is plausible if the applicant does not recall that 24 hrs are allowed to return pool temperature below the Power Operation limit, not 12 hrs.

C is Incorrect. This distractor is plausible if the applicant does not recall the Max power limit value of 95F and that temperature is allowed to be 10F above that limit.

References to be None provided durin ~ exam:

Lesson Plan 2621.850.0.0090, Overview/Highlights of Technical Specifications Learning Objective/ TSX-1661, Using the Tech Specs, determine if LCO requirements are/are not being met and determine the appropriate plant/operator response and state the basis for the response.

Question Source (New, Modified, Bank) I Bank If Bank or Modified:

VISION System/Question 10 SRO Question #80 Question :Source 2010 NMP-1 SRO NRC Exam Memory or X Comprehension Fundamental Cognitive 1:P or Analysis Knowledge Level NUREG 1021 Appendix B: Procedure steps and cautions 10CRF55 55.41 I I 55.43 I 1 Content Conditions and limitations in the facility license.

Justification for LORT questions with N/A KIA values < 3.0 ILT 10-1 NRC & AUDIT EXAM Page: 11 of 81 26 May 2011

EXAMINATION ANSWER KEY ILT 10-1 NRC SRO Exam Time to Complete: 1-2 minutes I Point Value: 1 System 10 No.: 295026 PRA: I NO Safet:v 5

IZI Initial License Level Functi(m: D LORT ILT 10-1 NRC & AUDIT EXAM Page: 12 of 81 26 May 2011

EXAMINATION ANSWER KEY IlT 10-1 NRC SRO Exam 5 10: 10..1 NSR05 Points: 1.00 A plant startup is in progress with all IRMs on Range 8. An event then occurred and plant indications now include the following:

  • Panel4F IRM 12 ON SCl OR INOP white light is illuminated
  • All Panel 4F RPS SCRAM SOlENIOD white lights are illuminated
  • AIIIRM indications on Panel4F are stable Which ONE of the following actions shall the SRO direct NEXT?

A. lAW ABN-39, RPS Failures, direct manually inserting a half scram on RPS I.

B. lAW RAP-G1e, IRM HI-HI/INOP, direct bypassing IRM 12 lAW procedure 402.4, IRM Bypass Operation.

C. lAW ABN-39, RPS Failures, direct placing the RPS I Sub Channel Test Keylocks in the TRIP position.

D. lAW RPV Control- with ATWS EOP, following Immediate Failure to Scram Actions, direct placing both ADS Timers in BYPASS.

Answer: A Answer Explanation I QID: 10-1 NSR05 Question # I 5 I Developer I Date: JJR 17-11-11 Knowledge and Ability Reference Information Importance Rating K&A RO SRO 295037 SCRAM Conditions Present and Reactor Power Above APRM Downscale or Unknown 1 1 2.4.8 .. Emergency Procedures I Plan: 3.8 4.5 Knowledge of how abnormal operating procedures are used in conjunction with EOP's.

Level I SRO I Tier I 1 I Group I 1 ILT 10-1 NRC & AUDIT EXAM Page: 13 of 81 26 May 2011

EXAMINATION ANSWER KEY ILT 10-1 NRC SRO Exam General ABN*39 References A is Correct. The question stem provides a condition where IRM 12 has failed INOP. With the Rod Block and IRM Hi-Hi/INOP alarm in, a 1/2 scram on RPS I should've been received and was not. lAW ABN-39, RPS Failures, a subsequent action in the ABN is to manually insert a 1/2 scram on RPS I.

B is Incorrect. This distractor is plausible since this is an action the crew will perform, however conservative decision making will require the SRO to direct FIRST inserting a manaul 1/2 scram on RPS I.

The action to bypass IRM 12 (and continue with the plant startup) would come from plant management.

C is Incorrect. This distractor is plausible since this Explanation is a subsequent action lAW ABN-39, however the action to insert (or attempt to insert) a manual 1/2 scram on RPS I is before placing the Sub Channel Test Keylocks in TRIP.

o is Incorrect. This distractor is plausible since it is an action the crew would take, but only if a valid scram signal is received on both RPS systems. In this case, a valid scram signal was received on RPS system I only.

This question examines the SRO's ability to interpret plant indications and prioritize ABN-39 over RPV Control - with ATWS EOP in this case. If actions of ABN*39 are not successful, then entry into the EOP is required.

References to be None provided durin ~ exam:

Lesson Plan 2621.828.0.0039, Reactor Protection System Learning RPS-10450, Describe and interpret procedure Objectivel sections and steps for plant emergency or off*

normal conditions that involve this system including personnel allocation and equipment operation lAW applicable ABN, SDRP, EOP & EOP support procedures and EP Procedures.

Question Source (New, Modified, Bank) I New If Bank or Modified: N/A VISION System/Question ID Question Source ILT 10-1 NRC & AUDIT EXAM Page: 14 of 81 26 May 2011

EXAMINATION ANSWER KEY ILT 10-1 NRC SRO Exam Memory or Comprehension X

Fundamental 2:RI or Analysis Cognitive Knowledge Level NUREG 1021 Appendix B: Recognizing Interaction between systems (plural), including consequences and implications 55.41 I I 55.43 I 5 10CRF55 Assessment of facility conditions and selection of Content appropriate procedures during normal, abnormal, and emergency situations.

Justification for LORT questions with N/A KIA values < 3.0 Time to Complete: 1*2 minutes I Point Value: 1 System 10 No.: 295037 PRA: I NO Safety ~ Initial License Level 1

Function: D LORT ILT 10-1 NRC & AUDIT EXAM Page: 15 of 81 26 May 2011

EXAMINATION ANSWER KEY ILT 10-1 NRC SRO Exam 6 ID: 10-1 NSR06 Points: 1.00 The plant is cooling down for a refuel outage with the following current conditions:

  • RPV water level is 160" and steady
  • RPV pressure is 85 psig and lowering slowly

Annunciator/Indication Required Action A. Annunciator OW PRESS Raise RPV water level to >

HIILO is alarming 185" lAW ABN-3, Loss of Shutdown Cooling B. Annunciator SHUT ON CLG Establish an RPV alternate ISOL VALVES OPEN clearing cooldown using the Turbine (NOT in alarm) Bypass Valves lAW ABN-3, Loss of Shutdown Cooling C. Annunciator SO HX PUMP Confirm SOC automatic RM TEMP HI is alarming isolation lAW the Secondary Containment Control EOP O. RPV pressure indication rises Bypass the isolation and to 90 psig restore SOC lAW 305, Shutdown Cooling System Operation Answer: B Answer Explanation I QID: 10-1 NSR06 Question # I 6 I Developer 1 Date: JJR 17-11-11 E

Knowledge and Ability Reference Information K&A Importance Rating RO I SRO ILT 10-1 NRC & AUDIT EXAM Page: 16 of 81 26 May 2011

EXAMINATION ANSWER KEY ILT 10-1 NRC SRO Exam 295021 Loss of Shutdown Cooling 14 2.4.45 - Ability to prioritize and interpret the 4.1 4.3 significance of each annunciator or alarm.

Level I SRO I Tier 1 I Group I 1 General ABN-3 References B is Correct. The plant is cooling down with SOC.

Annunciator ISOL VALVES OPEN would be in the alarm state with SOC in service. This annunciator alarms when ever SOC V-17-19 or V-17-54 is NOT in the full closed position. When this annunciator clears, then both valves are in the full closed position. With these valves closed, all SOC flow is lost. lAW ABN*3, Loss of Shutdown Cooling, if SOC is lost, then re-establish alternate cooling lAW Attachment ABN-3-3. At 85 psig, RPV coolant temperature is about 327F, and the Attachment allows the use of the main turbine bypass valves to cooldown.

A is Incorrect but plausible. SOC will isolate on high OW pressure (3.0 psig). But the annunciator Explanation provided in answer A alarms at 1.4 psig and thus there is no isolation signal. The action listed in answer A is correct for a high OW pressure isolation of SOC.

C is Incorrect but plausible. If a primary leak were to occur in the SOC Room, then the Secondary Containment Control EOP would require isolation of the leak, which could be performed by isolating the SOC system. This would result in a total loss of SOC flow. But, the alarm provided, which does indicate a SOC leak, does not result in an automatic SOC isolation. The SOC system isolation is manual.

o is Incorrect but plausible. SOC will isolate if recirculation pump loop temperature reaches 350F.

But at 90 pslg, the temperature is only about 331 F, and no automatic isolation occurs.

References to be Attachment 203-2 provided during exam:

Lesson Plan 2621.828.0.0045, Shutdown Cooling System Learning SOC*10445, Given a set of system indications or Objective/ data, evaluate and interpret them to determine limits, trends and system status.

ILT 10-1 NRC & AUDIT EXAM Page: 17 of 81 26 May 2011

EXAMINATION ANSWER KEY ILT 10-1 NRC SRO Exam Question Source {New, Modified, Bank} I Bank If Bank or Modified:

VISION System/Question 10 718253 Question Source ILT 09-1 SRO Audit Exam Memory or Comprehension X

Fundamental 3:SPR Cognitive or Analysis Knowledge Level NUREG 1021 Appendix B: Solve a E!roblem using References 55.41 I I 55.43 I 5 10CRF55 Assessment of facility conditions and selection of Content appropriate procedures during normal, abnormal, and emergency situations.

Justification for LORT questions with N/A KIA values < 3.0 Time to Complete: 1-2 minutes I Point Value: 1 System I[JI No.: 295021 PRA: I NO Safety ~ Initial License Level 4

Functictn: o LORT ILT 10-1 NRC & AUDIT EXAM Page: 18 of 81 26 May 2011

EXAMINATION ANSWER KEY ILT 10-1 NRC SRO Exam 7 ID: 10-1 NSR07 Points: 1.00 The reactor was at rated power when an event occurred. Current plant conditions are as follows:

  • RPV water level lowered to 130" and has recovered to 182"
  • RPV pressure is 900 psig and steady
  • Drywell temperature is 225 OF and steady
  • Dryw1ell pressure is 2 psig and steady
  • Torus water level is 120" and steady
  • Torus water temperature is 158 OF and rising slowly CONTAINMENT SPRAY INITIATION LIMIT 600 550 B c 500

- I (7,550)  !

450

- Ir I 400 ORYWEll TEMPERATURE 350 (OF) 300 - I

, I 250 200

- I 150 100

- lA I I

o 1.8 5 10 15 20 25 30 35 40 DRYWEll PRESSURE (PSI G)

ILT 10-1 NRC & AUDIT EXAM Page: 19 of 81 26 May 2011

EXAMINATION ANSWER KEY ILT 10-1 NRC SRO Exam TORUS LOW LEVEL 240 230 220 210 200 TORUS TEMPERATURE 190 (1:)

180 170 TORUS LEVEL 100

-143 150 120 110 140 0 100 200 300 400 500 600 700 aoo 900 1000 1100 RPV PRESSURE (PSIG)

Which of the following actions shall the SRO direct?

A. Line-up and spray the Drywell per the Primary Containment Control EOP.

B. Emergency Depressurize the RPV per the Primary Containment Control EOP.

C. Lower RPV pressure with Turbine Bypass valves per the RPV Control noATWS EOP.

D. Lower RPV pressure with the Isolation Condensers per the RPV Control

- no ATWS EOP.

Answer: C

~ Answer Explanation QID: 10-1 NSR07 ILT 10-1 NRC & AUDIT EXAM Page: 20 of 81 26 May 2011

EXAMINATIC~N ANSWER KEY ILT 10-1 NRC SRO Exam I Question # 7 I Developer/Date: JJR/7*11*11 Knowledge and Ability Reference Information Importance Rating K&A RO SRO 295030 LClIW Suppression Pool Water Level/ 5 EA2.03

  • Ability to determine and/or interpret the following as they apply to LOW 3.7 3.9 SUPPRESSION POOL WATER LEVEL:

Reactor pressure Level I SRO I Tier 1 I Group I 1 General EOP User's PCC EOP References Guide ILT 10-1 NRC & AUDIT EXAM Page: 21 of 81 26 May 2011

EXAMINATION ANSWER KEY ILT 10-1 NRC SRO Exam C is Correct. A loss of Orywell cooling, with controlled Drywell venting can result in the Drywell conditions listed. A Torus leak combined with EMRV leakage, with no Torus cooling can result in the Torus indications listed.

From the conditions in the question stem, it is given that the reactor has scrammed. The plant has entered RPV Control - No ATWS EOP (RPVC-NA EOP) on low RPV water level, and Primary Containment Control EOP (PCC EOP) due to low Torus water level, high Torus water temperature, and high OW temperature.

There are no parameters that require an emergency depressurization. Currently, the Heat Capacity Temperature Limit Curve is not violated but will be violated if RPV pressure is maintained constant and Torus temperature continues to rise. If Torus temperature and RPV water level cannot be maintained below HCTL, ED will be required. lAW the RPV Control- No ATWS EOP, if Torus Explanation temperature cannot be maintained below HCTL, then maintain RPV pressure below HCTL. This action will prevent the need to ED. Because RPV water level is >

180", the Isolation Condensers cannot be used to reduce RPV pressure. The EMRVS can be used to lower RPV pressure.

A is Incorrect. This distractor is plausible if the applicant does not recognize that spraying the Drywell is not appropriate since Drywell is < 12 psig, and since Orywell parameters are on the bad side of the Containment Spray Initiation Limit Curve.

B is Incorrect. This distractor is plausible if the applicant does not recognize that the HCTL is not currently violated and lowering RPV pressure can prevent the need to ED.

D is Incorrect. This distractor is plausible if the applicant does not recognize that since RPV water is

> 180", it precludes the use of the Isolation Condensers.

ILT 10-1 NRC & AUDIT EXAM Page: 22 of 81 26 May 2011

EXAMINATION ANSWER KEY ILT 10-1 NRC SRO Exam References to be None provided durin ~ exam:

Lesson Plan 2621.845.0.0056, Primary Containment Control Learning PCC..3000, Using EMG-3200.02, evaluate the Objective/ technical basis for each step in the procedure and apply this evaluation to determine correct courses of action under emergency conditions.

Question Source (New, Modified, Bank) I Bank If Bank or Modified:

VISION System/Question ID 609460 Question Source ILT 07-1 SRO NRC Exam Memory or X Comprehension Fundamental 3:SPK Cognitive or Analysis Knowledge Level NUREG 1021 Appendix B: Solve a f.roblem using Knowledge and its meaning 55.41 I I 55.43 I 5 10CRF55 Assessment of facility conditions and selection of Content appropriate procedures during normal, abnormal, and emergency situations.

Justification for LORT questions with N/A KIA values < 3.0 Time to Complete: 1-2 minutes I Point Value: 1 System 10 No.: 295030 PRA: I NO Safety C8J Initial License Level 5

Function: D LORT ILT 10-1 NRC & AUDIT EXAM Page: 23 of 81 26 May 2011

EXAMINATION ANSWER KEY ILT 10-1 NRC SRO Exam 8 ID: 10..1 NSR08 Points: 1.00 The reactor was at rated power when an event occurred, which required a manual scram. The following plant conditions exist:

  • The REACTOR MODE SELECTOR switch is in SHUTDOWN
  • All APRM/LPRM DNSCL lights are OFF
  • RPV pressure is being maintained 800 - 1000 psig with the Isolation Condensers
  • Drywell pressure indicates 2 pSig and rising slowly
  • Torus water temperature indicates 80 0 F and steady
  • RPV water level indicates 140" and steady
  • Drywell temperature indicates 220 0 F and rising slowly
  • Radiation elements C3 and C6 indicate 25 Mr/hr and 16 Mr/hr, respectively, and are rising slowly
  • The following Panel 1OR ISO COND RM TEMPS indicate between 189 0 F and 202 0 F, and rising slowly:
  • IB-06-A, SOUTH COLUMN BY CONDENSERS ELEV. 95 FT.
  • IB-06-B, NORTH COLUMN BY CONDENSERS ELEV. 95 FT.
  • IB-06-C, CEILING BY EAST VALVES ELEV. 75 FT.
  • IB-06-0, CEILING BY WEST VALVES ELEV. 75 FT.

Which one of the following actions must be taken for these conditions?

A. Maintain RPV water level between 138" - 175", lAW RPV Control - with ATWS.

B. Initiate the Liquid Poison System lAW Support Procedure 22, Initiating The Liquid Poison System.

C. Initiate one Containment Spray System in the Torus Cooling Mode lAW Primary Containment Control.

D. Isolate the Isolation Condensers and use EMRVs for pressure control, lAW Secondary Containment Control.

Answer: 0 Answer Explanation I QID: 10-1 NSR08 Question # I 8 I Developer 1 Date: JJR 17-11-11 Knowledge and Ability Reference Information IlT 10-1 NRC & AUDIT EXAM Page: 24 of 81 26 May 2011

EXAMINATION ANSWER KEY ILT 10-1 NRC SRO Exam Importance Rating K&A RO SRO 295033 High Secondary Containment Area Radiation Levels /9 EA2.01 - Ability to determine and/or interpret the following as they apply to HIGH 3.8 3.9 SECONDARY CONTAINMENT AREA RADIATION LEVELS: Area radiation levels Level I SRO I Tier 1 I Group I 2 General References EOP Users SCCEOP Guide D is Correct. Two indications are given that show a leak in the area of the ICs: rising temperatures and radiation levels. Both temperature and radiation levels are above the Max Normal valves in Secondary Containment Control EOP. Since it appears that one/both of the ICs are discharging into the RB, they should be isolated, and RPV pressure control should be augmented with another system, such as EMRVs.

The section that directs isolating the IC's from the SCC EOP is not provided due to being a direct lookup.

A is Incorrect but plausible. Because reactor power is above 2% (all dn scl/inop lights off), then RPV Explanation water level should be lowered to below 30", lAW the ATWS EOP.

B is Incorrect but plausible. The Liquid Poison System would be required to be initiated due to power oscillations or to prevent exceeding the BIIT curve.

C is Incorrect but plausible. The question stems shows that an ATWS occurred and ICs are controlling RPV pressure due to the failure of the TBVs. Because torus water temperature is 80° F and steady, there is no need to initiate torus cooling.

This portion of PCC is not provided since it would be a direct lookup.

References to be None provided during exam:

ILT 10-1 NRC & AUDIT EXAM Page: 25 of 81 26 May 2011

EXAMINATION ANSWER KEY ILT 10-1 NRC SRO Exam Lesson Plan 2621.845.0.0057, Secondary Containment Control Learning SCC-3082, Using Procedure 3200.11, evaluate the Objectivel technical basis for each step and apply this evaluation to determine the correct course of action under emergency conditions.

Question Source (New, Modified, Bank) I Bank If Bank or Modified:

VISION System/Question 10 608983 Question Source ILT 07-1 SRO Comp #3 Memory or X Comprehension Fundamental 3:SPK Cognitive or Analysis Knowledge Level NUREG 1021 Appendix B: Solve a eroblem using Knowledge and its meaning 55.41 I I 55.43 I 5 10CRF55 Assessment of facility conditions and selection of Content appropriate procedures during normal, abnormal, and emergency situations.

Justificatic)n for LORT quelstions with N/A KIA values. < 3.0 Time to Cc,mplete: 1-2 minutes I Point Value: 1 System 10 No.: 295033 PRA: I NO Safety [8j Initial License Level 9

Function: o LORT ILT 10-1 NRC & AUDIT EXAM Page: 26 of 81 26 May 2011

EXAMINATION ANSWER KEY ILT 10-1 NRC SRO Exam 9 10: 10-1 NSR09 Points: 1.00 The plant is at rated power when an event occurred. The current plant conditions are as follows:

  • RPV pressure is being maintained at 900 - 1000 psig
  • Drywell pressure is 10 psig and rising slowly
  • Torus pressure is 9 psig and rising slowly
  • Torus water temperature is 135" F and rising slowly
  • Torus water level is 170" and rising slowly Refer to portions of "Primary Containment Control" below.

"Primary Containment Control" TORUS LOAD Urv11T TORUS WATER LEVEL (IN.)

B 1~+---+-'--+---+---+---+---+---+---+---+---+---~~

150+1---r--+---~~---r--4---+-~---+---r--++~

/~

/)-/

O+~~~"~~r'~~~~T~~~+~r~~~~~~~~~'~~

o 100 200 300 400 500 600 700 800 900 1000 1100 1200 RPV PRESSURE (PSIG)

ILT 10-1 NRC & AUDIT EXAM Page: 27 of 81 26 May 2011

EXAMINATION ANSWER KEY ILT 10-1 NRC SRO Exam "Primary Containment Control" UNTIL IT HAS BEEN DETERMINED THAT TORUS WATER TEMPERATURE AND RPV PRESSURE CANNOT BE MAINTAINED BELOW IG.F HCT


9 EMERGENCY DEPRESSURIZATION IS REQUIRED CONCURRENTLY VVITH THIS PROCEDURE TORUS HIGH LEVEL 230 220 210 +-~

TORUS TEMPERATURI: 190 +-....~~...-.....-.......

(I)

TORUS LEVEL

--144

-154

-164

-174 1~+---+----+---+---+---+---+---+-~~--+---~~ --180

~~~~..,...........,~~ -188 o 100 200 300 400 500 eoo 700 800 900 1000 1100 RPV PRESSURE (PSIG)

ILT 10-1 NRC & AUDIT EXAM Page: 28 of 81 26 May 2011

EXAMINATION ANSWER KEY ILT 10-1 NRC SRO Exam Which of the following is the correct action and the reason for this action?

A. Lower RPV pressure to allow all low pressure systems to inject per RPV Control - no A TWS Level Restoration Leg.

B. Place the ADS Timers in BYPASS to prevent an uncontrolled cooldown per RPV Control - no ATWS Level Restoration Leg.

C. Lower RPV pressure to prevent over-stressing the EMRV quenchers in the event that an EMRV lifts per RPV Control- no ATWS Pressure Leg.

D. Emergency Depressurize the RPV to prevent exceeding the Heat Capacity Temperature Limit on a LOCA per Primary Containment Control Torus Temperature Leg.

Answer: C Answer Explanation I QID: 10-1 NSR09 Question # I 9 I Developer I Date: JJR 17-11-11 Knowledge and Ability Reference Information Importance Rating K&A RO SRO 295029 High Suppression Pool Water Levell 5 2.1.32 - Conduct of Operations: Ability to 3.8 4.0 explain and apply all system limits and precautions.

Level I SRO I Tier 1 I Group I 2 General RPV Control - EOP User's References noATWS Guide ILT 1()"1 NRC & AUDIT EXAM Page: 29 of 81 26 May 2011

EXAMINATION ANSWER KEY ILT 10-1 NRC SRO Exam C is Correct. From the given conditions, the combination of RPV pressure and torus level (and slowly rising) place it very close to the torus load limit curve (PPC EOP). Since torus level is rising, the next question asked is if RPV pressure can be maintained below Torus Load Limit (TLL). Violating TLL when an EMRV opens can over-stress EMRV tailpiece and related components (quencher). So, the only correct answer is to ensure RPV pressure stays below the TLL as torus water level rises. There is also an override in RPVC - No ATWS which says that if torus level cannot be maintained below TLL, then reduce RPV pressure. The question ensures the applicant understands and applies the correct actions to maintain plant parameters below the TLL.

A is Incorrect. In the level leg of RPV Control, with level above 61", the Level Restoration portion has not been entered. When asked if level can be Explanation maintained/restored above 61", the answer is yes which bring you back to the top of the level leg.

Lowering RPV pressure is in the restoration leg which has not been entered. This distractor is plausible if the applicant does not correctly interpret level conditions.

B is Incorrect. Placing the ADS Timers in bypass is in the RPV water level restoration leg, which has not even been entered yet. This distractor is plausible if the applicant does not correctly interpret level conditions.

o is Incorrect but plausible. In the torus temperature leg of PCC EOP, ED is the last step when it has been determined that torus temperature and RPV pressure cannot be maintained below HCTL. The given torus temperature is below the relevant HCTL curve, and, RPV pressure can be reduced. ED is not required now from this parameter.

References to be None provided during exam:

ILT 10-1 NRC & AUDIT EXAM Page: 30 of 81 26 May 2011

EXAMINATION ANSWER KEY ILT 10-1 NRC SRO Exam Lesson Plan 2621.845.0.0052, RPV Control - no ATWS Learning ENA-3055, Given a copy of RPV Control, describe Objective/ in detail each step or conditional statement, including technical basis, and how to perform each step as required.

Question Source (New, Modified, Bank) I Bank If Bank or Modified:

VISION System/Question ID 510927 Question Source ILT 05-1 SRO Audit Exam Memory or X Comprehension Fundamental 3:SPK Cognitive or Analysis Knowledge Level NUREG 1021 Appendix B: Solve a eroblem using Knowledge and its meaning 55.41 I I 55.43 I 5 10CRF55 Assessment of facility conditions and selection of Content appropriate procedures during normal, abnormal, and emergency situations.

Justification for LORT questions with N/A KIA values < 3.0 Time to Complete: 1-2 minutes I Point Value: 1 System ID No.: 295029 PRA: I NO Safety ~ Initial License Level 5

Functicm: D LORT ILT 10-1 NRC & AUDIT EXAM Page: 31 of 81 26 May 2011

EXAMINATION ANSWER KEY ILT 10-1 NRC SRO Exam 10 10: 10*1 NSR010 Points: 1.00 The plant is at 25% power when a pneumatic supply line failure to outboard MSIV NS 048 results in plant conditions including the following:

  • Panel 11 F NS-048 indicates GREEN light ON and RED light OFF Which of the following describes (1) the plant impact, if any, and (2) what procedural actions must be taken by the SRO?

A. (1) An automatic half scram on RPS 2 will occur ONLY.

(2) Reset the half scram lAW RAP-G1c, SCRAM CONTACTOR OPEN, when the cause is corrected and conditions permit.

8. (1) An automatic half scram on RPS 2 will occur ONLY.

(2) Enter ABN-1, Reactor Scram, and restore RPV level between 138 160"; stabalize RPV pressure pressure below 1045#.

C. (1) An automatic full scram will occur.

(2) Enter ABN-1, Reactor Scram, and restore RPV level between 138 160"; stabalize RPV pressure pressure below 1045#.

D. (1) NEITHER a half scram or full scram will occur.

(2) Continue Power Operations lAW 202.1; direct Work Support correct pneumatic supply failure.

Answer: A Answer Explanation I QID: 10-1 NSR010 Question # I 10 I Developer 1Date: JJR 17-11-11 Knowledge and Ability Reference Information Importance Rating K&A RO SRO 295020 Inadvertent Cont. Isolation I 5 & 7 2.2.44 - Equipment Control: Ability to interpret control room indications to verify the status and operation of a system, and understand 4.2 4.4 how operator actions and directives effect plant and system conditions.

Level I SRO I Tier I 1 I Group I 2 ILT 10-1 NRC & AUDIT EXAM Page: 32 of 81 26 May 2011

EXAMINATION ANSWER KEY ILT 10-1 NRC SRO Exam General RAP-J2a References A is Correct. The question stem provides a condition where the applicant must interpret that NS 04B (outboard MSIV) has failed closed. Since the reactor is at 25% power, this will only result in a half scram on RPS 2. If power was higher, the resulting RPV pressure rise would result in a full reactor scram. The SRO must then direct actions in RAP J2a, MSIV CLOSED II, to reset the half scram once the cause of the pneumatic failure is corrected and plant conditions permit.

B is Incorrect. This distractor is plausible if the Explanation applicant believes the reactor should be manually scrammed due to this condition.

C is Incorrect. This distractor is plausible if the applicant believes the plant reached an automatic scram setpoint. An MSIV in both RPS systems must be < 90% open OR RPV pressure must rise above the high scram setpoint (which will not occur at the lower power in the question stem).

o is Incorrect. This distractor is plausible if the applicant does not recall that one MSIV <90% open will result in a half scram on RPS.

References to be None provided c::Iurin~ exam:

Lesson Plan 2621.828.0.0037, Reactor Protection System Learning RPS-10441 , Given the system logic/electrical Objective/ drawings, describe the system trip signals, setpoints and expected system response including power loss or failed components.

Question Source (New, Modified, Bank) I Modified If Bank or Modified:

VISION System/Question 10 N/A Question Source Peach Bottom Dec 2009 SRO Exam ILT 10-1 NRC & AUDIT EXAM Page: 33 of 81 26 May 2011

EXAMINATION ANSWER KEY ILT 10-1 NRC SRO Exam Memory or X Comprehension Fundamental 2:RI or Analysis Cognitive Knowledge Level NUREG 1021 Appendix B: Recognizing Interaction between systems (plural), including consequences and implications 55.41 I I 55.43 I 5 10CRF55 Assessment of facility conditions and selection of Content appropriate procedures during normal, abnormal, and emergency situations.

Justification for LORT questions with N/A KIA values < 3.0 Time to Complete: 1-2 minutes I Point Value: 1 System 10 No.: 295020 PRA: I NO Safety 5&7 181 Initial License Level Function: D LORT ILT 10-1 NRC & AUDIT EXAM Page: 34 of 81 26 May 2011

EXAMINATION ANSWER KEY ILT 10-1 NRC SRO Exam 11 ID: 10-1 NSR011 Points: 1.00 The plant was at rated power when an event occurred. Current conditions are as follows:

  • 4160 BUS 1B indicates 0 AC AMPERES
  • RPV water level is 40" and lowering
  • Feedwater is being injected into the RPV at 2.5 x 106 Ib/hr lAW Support Procedure 8, Lineup for Condensate Injection
  • Core Spray System 1 is lined up for injection lAW Support Procedure 9, Lineup for Gore Spray System Injection
  • RPV pressure is 400 psig and lowering
  • Drywell pressure is 17.9 psig and rising
  • The leak rate into the primary containment has been quantified at 2.9 x 106 1b/hr Which of the following states the RPV water level control strategy the SRO shall direct AT THIS TIME?

A. Lower RPV pressure as necessary to allow low pressure systems to inject into the RPV.

B. Manually raise feedwaterflowto > 2.9 x 106 1b/hr lAW Support Procedure 8, Lineup for Condensate Injection.

C. Line-up and commence injection with Core Spray 2 lAW Support Procedure 9, Lineup for Core Spray System Injection.

D. Wait until RPV water level lowers to the top of active fuel, and then direct an ED to allow low pressure systems to inject into the RPV.

Answer: A Answer Explanation I QID: 10-1 NSR011 Question # I 11 I Developer I Date: JJR I 7*11-11 Knowledge and Ability Reference Information Importance Rating K&A RO I SRO ILT 10-1 NRC & AUDIT EXAM Page: 35 of 81 26 May 2011

EXAMINATION ANSWER KEY ILT 10-1 NRC SRO Exam 259002 Reactor Water Level Control System A2.04

  • Ability to (a) predict the impacts of the following on the REACTOR WATER LEVEL CONTROL, SYSTEM and (b) based on those 3.0 3.1 prediction,s, use procedures to correct, control, or mitigate the consequences of those abnormal operation: RFP runout condition:

Plant-Specific Level I SRO I Tier 2 I Group I 1 General RPV Control* EOP User's References noATWS Guide ILT 10-1 NRC & AUDIT EXAM Page: 36 of 81 26 May 2011

EXAMINATION ANSWER KEY ILT 10-1 NRC SRO Exam A is Correct. The stem shows that a leak into the PC has occurred, and that FW pumps Band C are not available, due to the Bus 1Bloss. RPV water level is 40" and lowering, and current RPV pressure is above all low pressure systems discharge head, and the reactor has scrammed. Because FW runout protection will cap flow through the one remaining FW pump at 2.67 x 106 1b/hr, FW flow cannot be raised to greater than the leak size (and other FW pumps are unavailable). Because DW pressure is >

2.9 psig, core spray has started and is running on minimum flow and NOT discharging into the RPV (which is at 400 psig). The EOP step should be to lower RPV pressure to allow low pressure systems (ie., core spray) to inject. This question also requires the SRO to choose the correct strategy between RPV Control - no ATWS level control and the Level Restoration contingency.

Explanatit)n B is Incorrect. This distractor is plausible if the applicant does not recognize that current RPV pressure is above all low pressure systems discharge head, and the reactor has scrammed.

Because FW runout protection will cap flow through the one remaining FW pump at 2.67 x 106 1b/hr, FW flow cannot be raised to greater than the leak size (and other FW pumps are unavailable).

C is Incorrect. This distractor is plausible if the applicant does not recognize that RPV pressure is greater than the discharge pressure of the Core Spray pumps. Injection will not be possible until RPV pressure lowers to < 310 psig.

D is Incorrect. Because core spray has started normally, when the SRO directs a lowered RPV pressure to allow core spray to inject, THEN the SRO will decide if this action can keep water level above 0".

References to be None provided during exam:

ILT 10-1 NRC & AUDIT EXAM Page: 37 of 81 26 May 2011

EXAMINATION ANSWER KEY ILT 10-1 NRC SRO Exam Lesson Plan 2621.845.0.0052, RPV Control - no ATWS Learning EWA-3055, Given a copy of the EOP, describe each Objective/ step/statement, including the technical basis and how to verify or perform each step.

Question Source (New, Modified, Bank) I Bank If Bank or Modified:

VISION System/Question 10 609226 Question Source ILT 07-1 SRO Audit Exam Memory or X Comprehension Fundamental 3:SPK Cognitive or Analysis Knowledge Level NUREG 1021 Appendix B: S.olve a Problem using Knowledge and its meaning 55.41 I I 55.43 I 5 10CRF55 Assessment of facility conditions and selection of Content appropriate procedures during normal, abnormal, and emergency situations.

Justificati40n for LORT questions with N/A KIA values < 3.0 Time to Complete: 1-2 minutes I Point Value: 1 System 10 No.: 259002 PRA: I NO Safety [g] Initial License Level 2

Function: o LORT ILT 10-1 NRC & AUDIT EXAM Page: 38 of 81 26 May 2011

EXAMINATION ANSWER KEY ILT 10-1 NRC SRO Exam 12 10: 10*1 NSR012 Points: 1.00 The plant was at rated power with activities being performed in the Spent Fuel Pool, when an event occurred on the refuel floor. The following conditions have existed for 5 minutes:

  • ALL refuel floor radiation monitors indicate between 70 and 90 mRlhr
  • REACTOR BUILDING VENT MANIFOLD NO.1 and NO.2 radiation monitors indicate 3 - 4 mRlhr
  • RX BLDG DIFFERENTIAL PRESS indicates -0.25 inches of water

".. IF mEN ~

Rx BLDG VENTIlATION EXHAUST CONFIRM SECONDARY OONTAlNMENT RADIATION LEVEL IS XMRIHR INmATIONS AND ISOLATIONS PER AOOVE SlJPPORTPROC49 I. Rx BLDG VENTILATION ISOLATES OR IS SHUTDOWN AND

2. DRYWELL IS NOT BEING VENTED THROUGHTHE Rx BlDG SUPPLY FANS AND
3. Rx BLDG VENTILATION EXHAUST RADIATION LEVEL IS BELOW X MRlHR OR OPERATEAVA1LABLE Rx BL.DG VENTILATION PER Sll PPORTPRQC.SO Rx.BLDG PRESSURE IS AOOVE 0 IN. OF WATER

-.... ANDA GROUND LEVEL RELEASE IS IMMINENT OR IN IROGRESS ~

c.urrent Status Future Status A. RBVS is operating RBVS is operating SGTS is in standby SGTS is in stand by B. RBVS is operating RBVS is shutdown SGTS is in standby SGTS is operating C. RBVS is shutdown/isolated RBVS is operating SGTS is operating SGTS is shutdown D. RBVS is shutdown/isolated RBVS is shutdown/isolated SGTS is operating SGTS is operating Answer: C IAnswer Explanation ILT 10-1 NRC & AUDIT EXAM Page: 39 of 81 26 May 2011

EXAMINATION ANSWER KEY ILT 10-1 NRC SRO Exam QID: 10-1 NSR012 Question # I 12 I Developer I Date: JJR I 7-11-11 Knowledge and Ability Reference Information Importance Rating K&A RO SRO 261000 SGTS A2.15 - Ability to (a) predict the impacts of the following on the STANDBY GAS TREATMENT SYSTEM; and (b) based on those predictions, 3.0 3.4 use procedures to correct, control, or mitigate the consequences of those abnormal conditions or operations: High area radiation by refuel bridge: Plant-Specific Level I SRO I Tier 2 I Group I 1 General EOP User's References Guide see EOP ILT 10-1 NRC & AUDIT EXAM Page: 40 of 81 26 May 2011

EXAMINATION ANSWER KEY ILT 10-1 NRC SRO Exam C is Correct. The question stem describes an event that occurred on the refuel floor. Floor area radiation monitors are alarming. Radiation monitors C9 and B9, when their setpoint is exceeded, will start a 2*

minute timer. At the end of 2 minutes, RBVS will trip and isolate and SGT will auto start.

As provided in an override in Secondary Containment Control EOP: 1) if RB ventilation isolates or is shutdown (which it has); AND 2)

Drywell is not being vented through the RB supply fans (which it isn't); AND 3) RB vent exhaust radiation levels are below 9 mr/hr (which they are)

OR RB pressure is above 0" of water and a ground release is imminent or in.progress (which it isn't),

THEN operate available RB ventilation lAW SP*50.

The question stem provides enough information to recognize that RBVS has tripped/isolated and SGTS has started. When SP-50, Reactor Building Ventilation Restart, is performed, this will stop SGTS and re-start RBVS.

The SRO must direct the correct override, either SP Explanation 50 or SP*49, in order to obtain the correct answer.

The actual setpoint values in each override have been changed to an "X" to eliminate a direct lookup.

A is Incorrect. "rhis distractor is plausible if the applicant believes the SRO should direct SP-50 but the RBVS has NOT tripped yet.

B is Incorrect. This distractor is plausible if the applicant does not recognize that refuel floor radiation levels have exceeded the point where RBVS has tripped and SGTS has started. In addition, this is plausible if the SRO believes they should direct SP-49 be performed instead of SP*50. In this case, the Future Status will have the SGTS operating and RBVS will be secured.

D is Incorrect. This is plausible if the SRO believes they should direct SP-49 be performed instead of SPa

50. In this case, the Future Status will have the SGTS operating and RBVS will be secured.

References to be None provided during exam:

ILT 10-1 NRC & AUDIT EXAM Page: 41 of 81 26 May 2011

EXAMINATION ANSWER KEY ILT 10-1 NRC SRO Exam Lesson Plan 2621.845.0.0057, Secondary Containment Control Learning SCC-3082, Using procedure 3200.11, evaluate the Objectiv'e/ technical basis for each step and apply this evaluation to determine the correct course of action under emergency conditions.

Question Source (New, Modified, Bank) I Bank If Bank or Modified:

VISION System/Question 10 666199 Question Source ILT 08-1 SRO Audit Exam Memory or X Comprehension Fundamental 3:SPK Cognitive or Analysis Knowledge Level NUREG 1021 Appendix B: Solve a Problem using Knowledge and its meaning 55.41 I I 55.43 I 5 10CRF55 Assessment of facility conditions and selection of Content appropriate procedures during normal, abnormal, and emergency situations.

Justification for LORT questions with N/A KIA values < 3.0 Time to Complete: 1-2 minutes I Point Value: 1 S-'Ystem 10 No.: 261000 PRA: I NO Safety [gJ Initial License Level 9

Function: D LORT ILT 10..1 NRC & AUDIT EXAM Page: 42 of 81 26 May 2011

EXAMINATION ANSWER KEY ILT 10-1 NRC SRO Exam 13 ID: 10..1 NSR013 Points: 1.00 The plant was at rated power when a LOCA occurred. Plant conditions include the following:

  • RPV Pressure indicates 890 psig and lowering
  • RPV Water Level indicates 58" and lowering
  • Drywell Pressure indicates 8 psig and rising
  • Drywell Temperature indicates 190°F and rising
  • Torus Pressure indicates 7 psig and rising
  • Torus Temperature indicates 190°F and rising
  • Containment Oxygen indicates 3% on both H2/02 monitors For the above conditions, which ONE of the following must the SRO direct NEXT?

Assume ALL Immediate Actions required by OP-OC-101-111-1001, Strategies For Successful Transient Mitigation, have been performed by the crew.

A. Bypass ADS Timers lAW RPV Control - Level Restoration B. Trip all Recirculation Pumps lAW RPV Control- with ATWS - Power Leg C. Line-up AND Initiate Drywell Sprays lAW Primary Containment Control Pressure Leg D. Exit all EOPs and enter SAMGs lAW Primary Contaiment Control Combustable Gas Leg Answer: A Answer Explanation I QID: 10-1 NSR013 Question # I 13 I Developer I Date: JJR 17-11-11 Knowledge and Ability Reference Information Importance Rating K&A RO I SRO ILT 10-1 NRC & AUDIT EXAM Page: 43 of 81 26 May 2011

EXAMINATION ANSWER KEY ILT 10-1 NRC SRO Exam 218000 A[)S 2.4.9 - Emergency Procedures / Plan:

Knowledge of low power / shutdown 3.8 4.2 implications in accident (e.g., loss of coolant accident or loss of residual heat removal) mitigation strategies.

Level I SRO I Tier 2 I Group I 1 RPV Control General EOP User's Level References Guide Restoration A is Correct. The question stem provides a condition where a LOCA occurred and the plant is now shutdown. lAW the EOP User's Guide, the reactor can be considered shutdown under all conditions without boron if all rods, except one, are full in. Since RPV water level is 58" and lowering, RPV Control - Level Restoration (entry at 61") has been entered. The first action the SRO must direct to the crew is to Bypass ADS Timers.

B is Incorrect but plausible. If the applicant believes the crew is taking actions lAW RPV Control - with ATWS, the next action the SRO would direct would be to trip all recirculation pumps from the Power Leg (or bypass ADS Timers in the Level/Power Leg which is not given as a choice). Tripping recirc pumps is Explanation the next action since the stem states that all Immediate Actions per the Transient Mitigation document has been completed by the crew. This would not be required anyway since all recirc pumps would have tripped on RPV Lo-Lo level <86".

C is Incorrect but plausible. It is true the the direction to Lineup Drywell Sprays lAW PCC could be ordered, however the order to initiate sprays must wait until Drywell or Torus Pressure is > 12 psig.

D is Incorrect but plausible. This would be the correct action if Containment Hydrogen was> 2.5%.

The stem states that Containment Oxygen is 3%.

The H2I02 analyzer monitors and the crew will record both values when determining Containment Combustable Gas levels.

References to be None provided during exam:

ILT 10-1 NRC & AUDIT EXAM Page: 44 of 81 26 May 2011

EXAMINATION ANSWER KEY ILT 10-1 NRC SRO Exam Lesson Plan 2621.845.0.0052, RPV Control - no ATWS Learning ENA-3055, Given a copy of RPV Control, describe Objective/ in detail each step or conditional statement, including technical basis, and how to perform each step as required.

Question Source (New, Modified, Bank) I New If Bank or Modified: N/A VISION System/Question ID Question Source Memory or Comprehension X Fundamental Cognitive or Analysis 3:SPK Knowledge Level NUREG 1021 Appendix B: Solve a froblem using Knowledge and its meaning 55.41 I I 55.43 I 5 10CRF55 Assessment of facility conditions and selection of Content appropriate procedures during normal, abnormal, and emergency situations.

Justification for LORT questions with N/A KIA values < 3.0 Time to Complete: 1-2 minutes I Point Value: 1 System ID No.: 218000 PRA: I NO Safety [8J Initial License Level 3

Function: D LORT ILT 10-1 NRC & AUDIT EXAM Page: 45 of 81 26 May 2011

EXAMINATION ANSWER KEY ILT 10-1 NRC SRO Exam 14 ID: 10-1 NSR014 Points: 1.00 The reactor was at rated power when an event occurred. Plant conditions include the following:

  • All red scram lights are ON
  • RPV water level indicates 120" TAF and rising slowly
  • Drywell pressure is 2.2 psig and rising very slowly
  • Drywell temperature is 170 OF and rising very slowly
  • Torus water temperature is 100 OF and rising
  • Annunciator ROPS BYPASSED is in alarm For the above conditions, which ONE of the following shall the SRO direct NEXT?

A. Close the MSIVs lAW ABN-40, Stuck Open EMRV B. Initiate drywell sprays lAW Primary Containment Control C. Vent the scram air header lAW RPV Control- with ATWS D. Perform scram reset and scram lAW RPV COlltrol- with ATWS Answer: D Answer Explanation I QID: 10*1 NSR014 Question # I 14 I Developer 1 Date: JJR 17-11-11 Knowledge and Ability Reference Information Importance Rating K&A RO SRO 215005 APRM I LPRM 2.4.46 - Ernergency Procedures I Plan: Ability 4.2 4.2 to verify that the alarms are consistent with the plant conditions.

Level I SRO I Tier I 2 I Group I 1 ILT 10-1 NRC & AUDIT EXAM Page: 46 of 81 26 May 2011

EXAMINATION ANSWER KEY ILT 10-1 NRC SRO Exam General RPV Control* EOP User's References with ATWS Guide D is Correct. The indications provided show that an electromatic relief valve (EMRV) is open (EMRV open and not closed alarms) and that a reactor scramed occurred (scram contactor open alarm and scram lights on). It also shows that the reactor is not shutdown and that power is greater than 4% (APRM downscale alarm not in-applicant verifies this is consistent with other plant conditions), and alternate rod insertion (ARI initiated alarm) has been initiated.

The initial conditions show that reactor overfill protection (ROPS) is bypassed and that all reactor recirculation pumps have been manually tripped (green-flagged switches). The next action in RPV Control* With ATWS is to insert control rods given a hydraulic ATWS exists (since all red scram lights are on, then all scram valves have opened and the ATWS is not electric). A possible method to insert control rods is to reset the scram, allow the scram discharge volume time to drain, and to scram again.

Explanation A is Incorrect but plausible. This is an action in ABN-40 to close MSIVs to limit cooldown, however MSIVs should never be closed during an ATWS (unless directed by EOPs) due to removing the largest heat sink available. This action is also directed following a successful manual scram, which has not occurred.

B is Incorrect but plausible. Even though drywell sprays could be initiated now in the drywell temperature leg, temperature is far away from 281 F, and other actions are of higher priority and would not be the NEXT action.

C is Incorrect but plausible if the applicant does not recognize that a hydraulic ATWS exists, not an electric ATWS. This is a correct action for the SRO to direct if an electric ATWS was present.

References to be None provided during exam:

ILT 10-1 NRC & AUDIT EXAM Page: 47 of 81 26 May 2011

EXAMINATION ANSWER KEY ILT 10-1 NRC SRO Exam Lesson Plan 2621.845.0.0053, RPV Control- with ATWS Learning EWA-10445, Given a set of system indications or Objective/ data, evaluate and interpret them to determine limits, trends and system status.

Question Source (New, Modified, Bank) I Modified If Bank or Modified:

VISION System/Question 10 510960 Question Source ILT 05-1 SRO NRC Exam Memory or X Comprehension Fundamental 3:SPK Cognitive or Analysis Knowledge Level NUREG 1021 Appendix B: Solve a Problem using Knowledge and its meaning 55.41 I I 55.43 I 5 10CRF55 Assessment of facility conditions and selection of Content appropriate procedures during normal, abnormal, and emergency situations.

Justification for LORT questions with N/A KIA values < 3.0 Time to Complete: 1-2 minutes I Point Value: 1 System 10 No.: 215005 PRA: I NO Safety [gI Initial License Level 7

Functic)n: o LORT ILT 10-1 NRC & AUDIT EXAM Page: 48 of 81 26 May 2011

EXAMINATION ANSWER KEY ILT 10-1 NRC SRO Exam 15 ID: 10-1 NSR015 Points: 1.00 A plant startup is in progress with the following conditions:

  • RPV pressure is 700 psig and rising slowly
  • RPV water level is in the normal band
  • Feedwater Pump A is in service An event then occurred. Plant conditions now include the following:
  • RPV water level swelled to 181" and is slowly rising Based on the above conditions, which of the following RPV pressure control strategies shall the SRO direct?

A. Use EMRVs lAW RPV Control - no A TWS B. Adjust the MPR setpoint lAW 201, Plant Startup

c. Use the Isolation Condensers lAW RPV Control- no ATWS D. Use the Bypass Valve Opening Jack lAW 201, Plant Startup Answer: A Answer Explanation I QID: 10-1 NSR015 Question # I 15 I Developer 1 Date: JJR 17-11-11 Knowledge and Ability Reference Information Importance Rating K&A RO SRO 212000 RPS A2.01 - Ability to (a) predict the impacts of the following on the REACTOR PROTECTION SYSTEM; and (b) based on those predictions, 3.7 3.9 use procedures to correct, control, or mitigate the consequences of those abnormal conditions or operations: RPS motor-generator set failure ILT 10-1 NRC & AUDIT EXAM Page: 49 of 81 26 May 2011

EXAMINATION ANSWER KEY ILT 10¥1 NRC SRO Exam Level J SRO I Tier 2 I Group I 1 General RPV Control - EOP User's 237E566 Sh. 3 References noAWTS Guide A is Correct. Under the conditions in the stem, with RPV pressure less that 825 psig (TS value), a single RPS Bus loss will result in a full reactor scram and closure of the MSIVs. With the closure of the MSIVs, changing the MPR setpoint or the Bypass Valve Opening Jack will have no impact on RPV pressure.

The EMRVs can be still used to control RPV pressure. Even though the use of EMRVs require Torus water level above 90", the event started at normal level of approximately 150" and can be Explanation assumed to be the same.

Band D are Incorrect but plausible if the applicant does not recall that MSIVs will go closed on a loss of a single RPS bus with RPV pressure < 825 psig.

C is Incorrect. This distractor is plausible if the applicant does not recognize that the Isolation Condensers are not available due to RPV level>

160". IC DC valves are also isolated at 180".

References to be None provided durin ~ exam:

Lesson Pian 2621.828.0.0037, Reactor Protection System Learning RPS-10445, Given a set of system indications or Objective/ date, evaluate and interpret them to determine limits, trends, and system status.

Question Source (New, Modified, Bank) I Bank If Bank or Modified:

VISION System/Question ID 609465 Question Source ILT 07-1 SRO NRC Exam Memory or X Comprehension Fundamental 3:SPK Cognitive or Analysis Knowledge Level NUREG 1021 Appendix B: Solve a froblem using Knowledge and its meaning 55.41 J I 55.43 j 5 10CRF55 Assessment of facility conditions and selection of Content appropriate procedures during normal, abnormal, and emergency situations.

ILT 10-1 NRC & AUDIT EXAM Page: 50 of 81 26 May 2011

EXAMINATION ANSWER KEY ILT 10-1 NRC SRO Exam Justification for LORT questions with N/A KIA values < 3.0 Time to Complete: 1*2 minutes I Point Value: 1 System 10 No.: 212000 PRA: I NO Safety 7

181 Initial License Level Function: o LORT ILT 10-1 NRC & AUDIT EXAM Page: 51 of 81 26 May 2011

EXAMINATION ANSWER KEY ILT 10-1 NRC SRO Exam 16 10: 10-1 NSR016 Points: 1.00 The plant is at 70% power. An event then occurred resulting in the following indications:

  • Conductivity recorder CR-423-11, Point 1, A NORTH HOTWELL, indicates hotwell conductivity of 1.1 IJmho/cm and steady
  • Conductivity recorder IJ1 0 indicates REACTOR WATER conductivity of 1.0 IJmho/cm and steady
  • Reactor coolant chloride concentration is 0.21 ppm as confirm by Chemistry Department Which of the following states the required action and the Technical Specifications bases on chloride ion concentration?

Action Chloride TS Bases A. Raise reactor power to raise To minimize chloride induced the steaming rate pitting corrosion of reactor internals B. Immediately initiate an orderly To minimize stress corrosion shutdown cracking of stainless steel components C. Backwash the A North To minimize stress corrosion condenser section cracking of stainless steel components D. Immediately isolate the A To minimize chloride enhanced North hotwell by closing the zirconium cracking CW inlet/outlet valves Answer: C Answer Explanation I QID: 10-1 NSR016 Question # I 16 I Developer 1 Date: JJR 17-11-11 c== Knowled e and Abili c== K&A ILT 10-1 NRC & AUDIT EXAM Page: 52 of 81 26 May 2011

EXAMINATION ANSWER KEY ILT 1O~1 NRC SRO Exam RO SRO 256000 Reactor Condensate A2.15

  • Ability to (a) predict the impacts of the following on the REACTOR CONDENSATE SYSTEM; and (b) based on those predictions, 2.8 3.3 use procedures to correct, control, or mitigate the consequences of those abnormal conditions or operations: Abnormal water quality Level I SRO J Tier 2 I Group I 2 General TS 3.3.E RAP-K7a References C is Correct. The question describes a high conductivity event introduced in the A North hotwell and its effect on reactor water chlorides. lAW the TS bases, limits on chlorides is to prevent stress corrosion cracking of stainless steel components.

RAP*K7a (HOTWELL CONDUCT HI) directs a backwash if hotwell conductivity is > 1 tJmho/cm.

A and D are Incorrect but plausible since they list the Explanation incorrect bases. The applicant may not recall the correct action to take either.

B is Incorrect but plausible. The TS allows the unit to remain at power for 72 hours8.333333e-4 days <br />0.02 hours <br />1.190476e-4 weeks <br />2.7396e-5 months <br /> when chlorides exceed 0.2 ppm (but < 0.5 ppm) and if conductivity exceeds 1 tJmho/cm (but < 10 tJmho/cm). TS does not require an immediate SID at the given chloride rate or conductivity.

References to be None provided during exam:

Lesson Plan 2621.828.0.0017, Feed and Condensate System Learning CFW-10445, Given a set of system indications or Objectivel data, evaluate and interpret them to determine limits, trends and system status.

Question Source (New, Modified, Bank) I Bank If Bank or Modified:

VISION System/Question ID 609232 Question Source ILT 07-1 SRO Audit Exam ILT 10-1 NRC & AUDIT EXAM Page: 53 of 81 26 May 2011

EXAMINATION ANSWER KEY ILT 10-1 NRC SRO Exam Memory or Comprehension X Fundamental Cognitive or Analysis 3:SPK Knowledge Level NUREG 1021 Appendix B: S.olve a E!roblem using Knowledge and its meaning 10CRF55 55.41 I I 55.43 I 2 Content Facility operating limitations in the technical specifications and their bases.

Justification for LORT questions with N/A KIA values < 3.0 Time to Complete: 1*2 minutes I Point Value: 1 System 10 No.: 256000 PRA: I NO Safety ~ Initial License Level 2

Functicln: D LORT ILT 10-1 NRC & AUDIT EXAM Page: 54 of 81 26 May 2011

EXAMINATION ANSWER KEY ILT 10-1 NRC SRO Exam 17 10: 10-1 NSR017 Points: 1.00 The plant is at rated power with the following conditions:

  • Reactor Engineering has determined adequate Shutdown Margin is available for continued operation A fault then occurs in the Reactor Manual Control System resulting in another control rod being declared INOPERABLE.

Complete the following sentence regarding whether condinued plant operation is allowed and the bases for that decision lAW Technical Specifications:

Continued plant operation is...

A. NOT allowed. The plant must be placed in the shutdown condition since this could be indicative of a generiC problem.

B. allowed. The reactor may remain in operation provided that ONLY the three (3) rods NOT at position 00 are defined as INOPERABLE.

C. allowed. The reactor may remain in operation provided that this new INOPERABLE control rod is NOT at position 48 AND adequate Shutdown Margin can be demonstrated.

D. NOT allowed. The plant must be placed in the shutdown condition since under any circumstance, the reactor CANNOT demonstrate adequate Shutdown Margin under this condition.

Answer: A Answer Explanation I QID: 10-1 NSR017 Question # I 17 I Developer 1 Date: JJR 17-11-11 Knowledge and Ability Reference Information Importance Rating K&A RO I SRO ILT 10-1 NRC & AUDIT EXAM Page: 55 of 81 26 May 2011

EXAMINATION ANSWER KEY ILT 10-1 NRC SRO Exam 201002 RMCS 2.2.40 - Ability to apply Technical 3.4 4.7 Specifications for a system.

Level I SRO I Tier 2 I Group I 2 General TS 3.2.B.4 References A is Correct. lAW Tech Spec 3.2.B.4, in no case shall the number of inoperable control rods valved out of service be greater than six during the power operation. If this specification is not met, the reactor shall be placed in the shutdown condition. The bases states the number of inoperable control rods permitted to be valved out of service could be many Explanation more than six allowed by the specification, particularly late in the operating cycle; however, the occurrence of more than six could be indicative of a generic problem and the reactor will be shut down.

All distractors are Incorrect but plausible if the applicant does not recall the Tech Spec action requirement or correct bases for the action.

References to be None provided durin ~ exam:

Lesson Plan 2621.850.0.0050, Overview/Highlights of Technical Learning Specifications Objective/

TSX-1920, Given various plant indications (and their values) or copies of control room/plant logs, evaluate the indications to determine plant status with respect to operating license and technical specifications.

Question Source (New, Modified, Bank) I Bank If Bank or Modified:

VISION System/Question ID 507130 Question Source ILT Bank #378 Memory or X Comprehension Fundamental 3:SPK Cognitive or Analysis Knowledge Level NUREG 1021 Appendix B: Solve a Problem using Knowledge and its meaning 10CRF55 55.41 I I 55.43 I 2 Content Facility operating limitations in the technical specifications and their bases.

ILT 10-1 NRC & AUDIT EXAM Page: 56 of 81 26 May 2011

EXAMINATION ANSWER KEY ILT 10-1 NRC SRO Exam Justification for LORT questions with N/A KIA values < 3.0 Time to Complete: 1*2 minutes I Point Value: 1 System 10 No.: 201002 PRA: I NO Safety 1

181 Initial License Level Function: D LORT ILT 10-1 NRC & AUDIT EXAM Page: 57 of 81 26 May 2011

EXAMINATION ANSWER KEY ILT 10-1 NRC SRO Exam 18 10: 10..1 NSR018 Points: 1.00 Following core power distribution checks, the # 3 TIP Ball Valve did NOT automatically close due to a malfunctioning in-shield limit switch.

What action is required?

Restore the inoperable TIP Ball Valve to operable status within (1) hours or (2) .

A. (1) 4 (2) actuate the respective TIP Shear Valve B. (1) 4 (2) de-energize the affected TIP Ball Valve in the closed position

c. (1) 48 (2) de-energize the affected TIP Ball Valve in the closed position D. (1) 48 (2) the reactor shall be placed in the Cold Shutdown condition within 24 hours2.777778e-4 days <br />0.00667 hours <br />3.968254e-5 weeks <br />9.132e-6 months <br /> Answer: C Answer Explanation I QID: 10-1 NSR018 Question # I 18 I Developer 1 Date: JJR 17-11-11 Knowledge and Ability Reference Information Importance Rating K&A RO SRO 215001 Tri8versing In-core Probe 2.1.32 - Ability to explain and apply system 3.8 4.0 limits and precautions.

Level I SRO I Tier 2 I Group I 2 General 405.2 TS 3.5.A.3 References ILT 10-1 NRC & AUDIT EXAM Page: 58 of 81 26 May 2011

EXAMINATION ANSWER KEY ILT 10-1 NRC SRO Exam C is Correct. In order to maintain Primary Containment Integrity (which is required during core power distribution checks ... reactor critical, etc.), and with an inoperable TIP ball valve (automatic containment isolation valve), Tech Spec 3.S.A.3 requires: maintaining an operable isolation valve in the affected penetration (the shear valve meets this requirement), and within 48 hours5.555556e-4 days <br />0.0133 hours <br />7.936508e-5 weeks <br />1.8264e-5 months <br /> (TIP) either restore the inoperable TIP ball valve, or isolate the penetration by use of a deactivated automatic isolation valve secured in the isolated position, or by Explanation use of a closed manual valve. The way to deactivate the TIP Ball Valve is to de-energize it.

A & B are Incorrect due to the time allowed. This distractor is plausible if the applicant does not recall the correct time the TIP must be returned to operable status.

D is Incorrect. This distractor is plausible since if these provisions cannot be met, the reactor shall be placed in COLD SHUTDOWN within 24 hours2.777778e-4 days <br />0.00667 hours <br />3.968254e-5 weeks <br />9.132e-6 months <br />.

References to be None provided during exam:

ILT 10-1 NRC & AUDIT EXAM Page: 59 of 81 26 May 2011

EXAMINATION ANSWER KEY ILT 10-1 NRC SRO Exam Lesson Plan 2621.828.0.0029, Nuclear Instrumentation Learning NIS-10451, Referencing plant Technical Objective/ Specifications (* from memory for Initial Candidates) and given a set of plant conditions, determine, as applicable, the:

a) Definitions*

b) Safety Limits and Bases*

c) Limiting Safety System Settings and Bases*

d) Limiting Conditions for Operation and Applicability e) LCO Action Requirements (SRO ONLy) f) Surveillance Requirements (SRO ONLy) g) Design Features, Containment, Auxiliary Equipment, Administrative Controls, and Appendix B Environmental Technical Specifications (SRO ONLY) h) Bases for Surveillance Requirements, Design Features, Containment, Auxiliary Equipment, Administrative Controls, and Appendix B Environmental Technical Specifications. (SRO ONLy)*

Question Source (New, Modified, Bank) I Bank If Bank or Modified:

VISION System/Question 10 506190/ NI-05 Question Source ILT Bank Memory or X Comprehension Fundamental Cognitive 1:P or Analysis Knowledge Level NUREG 1021 Appendix B: erocedure steps and cautions 10CRF55 55.41 I I 55.43 I 2 Content f':Cility operating limitations in the technical ecifications and their bases.

Justification for LORT questions with N/A KIA values < 3.0 Time to Complete: 1-2 minutes I Point Value: 1 System I[) No.: 215001 PRA: I NO Safety 181 Initial License Level Function:

7 o LORT ILT 10-1 NRC & AUDIT EXAM Page: 60 of 81 26 May 2011

EXAMINATION ANSWER KEY ILT 10-1 NRC SRO Exam 19 ID: 10-1 NSR019 Points: 1.00 The plant is shutdown for a refuel outage. Control rod 30-35 is to be replaced. Which of the following lists, in the correct order, the steps to prepare the cell to remove the control rod from the core?

A. 1. Remove fuel bundles A and B

2. Insert double blade guide
3. Remove fuel bundles C and D
4. Uncouple control rod
5. Withdraw control rod to position 48 B. 1. Remove fuel bundles A and C
2. Insert double blade guide
3. Remove fuel bundles Band D
4. Withdraw control rod to position 48
5. Uncouple control rod C. 1. Remove fuel bundles A and B
2. Remove fuel bundles C and D
3. Insert double blade guide
4. Uncouple control rod
5. Withdraw control rod to position 48 D. 1. Remove fuel bundles A and insert single blade guide
2. Remove fuel bundles B and insert single blade guide
3. Remove fuel bundles C and insert single blade guide
4. Remove fuel bundles D and insert single blade guide
5. Withdraw control rod to position 48
6. Uncouple control rod ILT 10-1 NRC & AUDIT EXAM Page: 61 of 81 26 May 2011

EXAMINATION ANSWER KEY ILT 10-1 NRC SRO Exam Answer: B Answer Explanation I QID: 10*1 NSR019 Question # I 19 I Developer / Date: JJR /7*11*11 Knowledge and Ability Reference Information Importance Rating_

K&A RO SRO 2.1.36 - Knowledge of procedures and 3.0 4.1 limitations involved in core alterations.

Level I SRO I Tier 3 I Category I COO General 205.0 205.5 References B is Correct. Procedures 205.0 (Reactor Refueling) and 205.5 (Rod Withdrawal/Insertion During Refueling) provide the general guidance to remove a control rod from the core: 1. remove 2 opposite bundles; 2. insert blade guide; 3. remove last 2 Explanation bundles; 4 withdraw rod to 48; 5. Uncouple.

All distractors are Incorrect but plausible if the applicant is not familiar with the control rod removal process during refuel activities.

References to be None provided durin tI exam:

Lesson Plan 2621.812.0.0003, Reactor Refueling Learning RFL-7442, Describe, in general, refueling and fuel Objective/ handling procedures to include precautions and limitations per Procedure 205 series.

Question Source (New, Modified, Bank) I Bank If Bank or Modified:

VISION System/Question ID 609011 Question Source ILT 07*1 SRO Comp #3 Memory or X Comprehension Fundamental Cognitive Knowledge 1:P or Analysis Level NUREG 1021 Appendix B: erocedure steps and cautions ILT 10-1 NRC & AUDIT EXAM Page: 62 of 81 26 May 2011

EXAMINATION ANSWER KEY ILT 1O~1 NRC SRO Exam 10CRF55 55.41 I I 55.43 I 7 Content Fuel handling facilities and procedures.

Justification for LORT questions with N/A KIA values < 3.0 Time to Complete: 1-2 minutes I Point Value: 1 System 10 No.: N/A PRA: I NO Safety ~ Initial License Level N/A Function: o LORT ILT 10*1 NRC & AUDIT EXAM Page: 63 of 81 26 May 2011

EXAMINATION ANSWER KEY ILT 10-1 NRC SRO Exam 20 ID: 10*1 NSR020 Points: 1.00 The Control Room has been evacuated due to a Control Room fire. ABN-30, Control Room Evacuation, is being executed. The following conditions exist:

  • The I~EACTOR MODE SELECTOR switch is in SHUTDOWN and all control rods verified full-in
  • RPV water level is steady at 150" and adequate core cooling is assured
  • RPV pressure is 900 psig and lowering
  • The control room has been evacuated
  • All Core Spray Pumps and all EMRV's have been disabled lAW ABN-30 Based on the conditions given. which of the following actions must be met, and bases, to comply with Technical Specifications?
1. Reduce RPV pressure to < 110 pSig within 24 hours2.777778e-4 days <br />0.00667 hours <br />3.968254e-5 weeks <br />9.132e-6 months <br />.
2. Place the reactor in COLD SHUTDOWN within 30 hours3.472222e-4 days <br />0.00833 hours <br />4.960317e-5 weeks <br />1.1415e-5 months <br />.

Action Bases A. 1 ONLY ADS requirements NOT met B. 2 ONLY Core Spray requirements NOT met C. 1 and 2 ADS and Core Spray requirements NOT met D. NEITHER 1 .or 2 All ADS and Core Spray requirements are met Answer: C Answer Explanation J QID: 10-1 NSR020 Question # I 20 I Developer 1 Date: J ..IR 17-11-11 Knowledge and Ability Reference Information Importance Rating K&A RO I SRO ILT 10-1 NRC & AUDIT EXAM Page: 64 of 81 26 May 2011

EXAMINATION ANSWER KEY ILT 10-1 NRC SRO Exam 2.2.22 - Knowledge of limiting conditions for 4.0 4.7 operations and safety limits.

Level I SRO I Tier 3 I Category I EQC General TS 3.4.A.2 References C is Correct. Both Tech Spec action statements must be met**.with the EMRV's disabled (lAW Attachment ABN-30-8) the ADS function is also disabled. Tech Spec 3.4.B (ADS) requires reactor pressure to be reduced to less than 110 psig within 24 hours2.777778e-4 days <br />0.00667 hours <br />3.968254e-5 weeks <br />9.132e-6 months <br /> if ADS operability requirements are not met.

Table 3.4.1 (Core Spray) allows reduced Core Spray capability, provided several things are met: one is Explanation that the RPV be maintained < 2120 F (currently at 900 psig). Since the requirements of the Table cannot be met, then 3.4.A.2 applies: place in Cold Shutdown within 30 hours3.472222e-4 days <br />0.00833 hours <br />4.960317e-5 weeks <br />1.1415e-5 months <br />.

All distractors are Incorrect but plausible if the applicant does not recall the Tech Spec LCO or Bases for ADS disabled.

References to be None provided during exam:

Lesson Plan 2621.830.0.0018, Equipment Control- Admin Learning 2.2.22, Knowledge of limiting conditions for Objective/ operations and safety limits.

Question Source (New, Modified, Bank) I Bank If Bank or Modified:

VISION System/Question 10 608914 Question Source ILT 07..1 SRO Comp #2 Memory or X Comprehension Cognitive Fundamental 1:B or Analysis Level Knowledge NUREG 1021 Appendix B: Bases or purpose 10CRF55 55.41 I I 55.43 I 2 Content Facility operating limitations in the technical specifications and their bases.

Justification for LORT questions with N/A KIA value~; < 3.0 Time to Complete: 1-2 minutes I Point Value: 1 ILT 10-1 NRC & AUDIT EXAM Page: 65 of 81 26 May 2011

EXAMINATION ANSWER KEY ILT 10-1 NRC SRO Exam System 10 No.: N/A PRA: I NO Safety [8] Initial License Level N/A Function: D LORT ILT 10-1 NRC & AUDIT EXAM Page: 66 of 81 26 May 2011

EXAMINATION ANSWER KEY ILT 10-1 NRC SRO Exam 21 10: 10-1 NSR021 Points: 1.00 The plant was at rated power when an event occurred.

20 minutes late.r. the following plant conditions exist:

  • Offgas Radiation Monitors have risen and continue to rise
  • Several Turbine Building AND Reactor Building Area Radiation Monitors are in alarm (but on-scale)
  • Turbine Building ap is positive
  • The Shift Manager has declared An Alert due to Radiological Effluent Which of the following actions is required?

A. Close the MSIVs lAW the Radioactivity Release Control EOP B. Close the MSIVs lAW ABN-26, High Main Steam/Offgas/Stack Effluent Activity C. Emergency Depressurize the RPV lAW the Radioactivity Release Control EOP D. Emergency Depressurize the RPV lAW the Secondary Containment Control EOP Answer: A Answer Explanation I QID: 10-1 NSR021 Question # I 21 I Developer I Date: JJR 17-11-11 Knowledge and Ability Reference Information Importance Rating K&A RO SRO 2.3.11 - Ability to control radiation releases. 3.8 4.3 Level I SRO I Tier 3 I Category I RPT General EOP User's RREOP References Guide ILT 10-1 NRC & AUDIT EXAM Page: 67 of 81 26 May 2011

EXAMINATION ANSWER KEY ILT 10-1 NRC SRO Exam A is Correct. The question states that an event had occurred. The conditions show that MSL and offgas radiation has increased, TB ARMs are in alarm and that TB AP is positive. These indicate a primary steam leak in the TB. The stem also shows that an alert emergency condition has been declared due to radiological effluents. This is an entry condition into the Radioactivity release Control EOP. The first step is to isolate primary systems discharging outside the primary and secondary containments. Closing the MSIVs would stop the leak into the TB.

Explanation B is Incorrect but plausible since ABN-26 requires closing the MSIVs when MSL radiation is > 800 mr/hr and the stem shows only 500 and rising slowly.

C is Incorrect but plausible since the Radioactivity Release Control EOP does require ED, but only after a GE is declared.

D is Incorrect but plausible since ED is also required in the Secondary Containment Control EOP, but the MAX SAFE must first be exceeded (with a primary leak in the RB) in 2 areas first.

References to be ABN*26 provided durin ~ exam:

Lesson Plan 2621.830.0.0015, Radiation Control - Admin Learning 2.3.11, Ability to control radiation releases Objectivel Question Source (New, Modified. Bank) I Bank If Bank or Modified:

VISION Sy'stem/Question ID 667779 Question Source ILT 08-1 SRO Audit Exam Memory or Comprehension X Fundamental Cognitive or Analysis 3:SPR Knowledge Level NUREG 1021 Appendix B: Solve a froblem using References 55.41 I I 55.43 I 4 10CRF55 Radiation hazards that may arise during normal and Content abnormal situations, including maintenance activities and various contamination conditions.

ILT 1()"1 NRC & AUDIT EXAM Page: 68 of 81 26 May 2011

EXAMINATION ANSWER KEY ILT 10-1 NRC SRO Exam Justification for LORT questions with N/A KIA values < 3.0 Time to Ciomplete: 1-2 minutes I Point Value: 1 System ID No.: N/A PRA: I NO Safety [81 Initial License Level N/A Function: o LORT lLT 10-1 NRC & AUDIT EXAM Page: 69 of 81 26 May 2011

EXAMINATION ANSWER KEY ILT 10*1 NRC SRO Exam 22 ID: 10-1 NSR022 Points: 1.00 The reactor was at rated power when a LOCA occurred. Plant conditions include the following:

  • Reactor has been scrammed and all rods at "00"
  • RPV pressure is 159 psig and lowering due to the leak
  • RPV water level was just raised to 60" TAF, and rising slowly
  • 1 Condensate Pump is still injecting
  • Torus water level is 184" and rising TORUS LOAD LIMIT 180 +--~I--

TORUS WATER \

LEVEL 170 (IN.)

B 160 i

(1125.156)

I, 150 L

/V

t~ 1J 0 100 200 300 400 500 600 700 800 900 1000 1100 1200 RPV PRESSURE (PSIG)

ILT 1Q...1 NRC & AUDIT EXAM Page: 70 of 81 26 May 2011

EXAMINATION ANSWER KEY ILT 10-1 NRC SRO Exam Which of th,e following EOP actions is required?

A. Emergency Depressurize per the RPV Control and Emergency Depressurization - no A TWS EOPs.

B. Terminate RPV injection with Condensate and inject with Core Spray per the Primary Containment Control EOP.

C. Lower RPV pressure with the Turbine Bypass Valves (exceeding 1000 F/hr is allowed) per the RPV Control- no ATWS EOP.

D. Anticipate Emergency Depressurization and rapidly reduce RPV pressure by opening the Turbine Bypass Valves per the RPV Control noATWS EOP.

Answer: B Answer Explanation I QID: 10-1 NSR022 Question # I 22 I Developer/Date: JJR/7-11-11 Knowledge and Ability Reference Information Importance Rating K&A RO SRO 2.4.6 - Knowledge of EOP mitigation 3.6 4.7 strategies.

Level I SRO I Tier 3 I Category I EOP General EOP User's References pec EOP Guide ILT 10-1 NRC & AUDIT EXAM Page: 71 of 81 26 May 2011

EXAMINATION ANSWER KEY ILT 10-1 NRC SRO Exam B is Correct. Under the given conditions, condensate is injecting and out through the break into the torus. Torus water is currently below the Torus load Limit. The only action which will prevent exceeding *rll and the necessity of ED, is to stop condensate (an outside containment injection source) and start core spray. Since only 1 condensate pump is currently running, 2 core spray pumps can pump more than this and will be more than able to ensure adequate core cooling.

Explanation A & D are Incorrect but plausible. ED is not required until it has been determined that the Tll cannot be maintained. Also, since there is some action that can be done to prevent exceeding Tll and ED, then anticipating ED is not correct.

C is Incorrect but plausible since lowering RPV pressure in distractor D is from a conditional statement in RPV Control will do nothing since the Tll curve is flat at the current RPV pressure and below.

References to be None provided durin t:J exam:

Lesson Plan 2621.830.0.0016, Emergency Procedures/Plan -

Admin Learning Objective/ 2.4.6, Knowledge of EOP mitigation strategies.

Question Source (New, Modified, Bank) I Bank If Bank or Modified:

VISION System/Question ID 608431 Question Source IlT 07-1 SRO Comp #2 Memory or Comprehension X Fundamental Cognitive or Analysis 3:SPK Knowledge Level NUREG 1021 Appendix B: Solve a f.roblem using Knowledge and its meaning 10CRF55 55.41 I I 55.43 I 5 Content Administrative, normal, abnormal, and emergency operating procedures for the facility.

Justification for lORT questions with N/A KIA values < 3.0 IlT10*1 NRC & AUDIT EXAM Page: 72 of 81 26 May 2011

EXAMINATION ANSWER KEY ILT 10-1 NRC SRO Exam Time to Complete: 1-2 minutes I Point Value: 1 System 10 No.: N/A PRA: I NO Safety rgj Initial License Level N/A Function: o LORT ILT 10-1 NRC & AUDIT EXAM Page: 73 of 81 26 May 2011

EXAMINATION ANSWER KEY ILT 10-1 NRC SRO Exam 23 10: 10-1 NSR023 Points: 1.00 Which of the following refuel activities REQUIRES a Licensed SRO to DIRECTLY supervise?

1. Withdrawal of fuel from the vessel.
2. Control rod removal from the reactor core.
3. Insertion of fuel into the vessel.
4. Withdrawal of a fuel support piece from an empty cell.
5. Insertion of spent fuel into a Fuel Pool rack.

A. 2 ONLY B. 1 and 3 ONLY C. 1,2 and 3 ONLY D. 1, 2, 3, 4 and 5 Answer: C Answer Explanation I QID: 10-1 NSR023 Question # I 23 I Developer 1 Date: JJR 17-11-11 Knowledge and Ability Reference Information Importance Rating K&A RO SRO 2.1.37 - Knowledge of procedures, guidelines, or limitations associated with reactivity 4.3 4.6 management.

Level I SRO I Tier 3 I Category I COO General TS 1.21 205.0 OP-AA-300-1520 References TS 6.2.2.2.e ILT 1()..1 NRC & AUDIT EXAM Page: 74 of 81 26 May 2011

EXAMINATION ANSWER KEY ILT 10-1 NRC SRO Exam C is Correct. TS definition 1.21 provides the following definition for core alteration: A core alteration is the addition, removal, relocation or other manual movement of fuel or controls in the reactor core. Control rod movement with the control rod drive hydraulic system is not defined as a core alteration.

TS 6.2.2.2.e provides the following: All CORE ALTERATIONS shall be directly supervised by either a licensed Senior Reactor Operator or Senior Explanation Reactor Operator Limited to Fuel Handling who has no other concurrent responsibilities during this operation.

OP-AA-300-1520, Reactivity Management - Fuel Handling, Storage, and Refueling, requires the core alterations be performed lAW approved procedures.

All distractors are Incorrect but plausible since they are all activites a Licensed SRO would likely supervise, however Answer B is the only choice where a Licensed SRO is REQUIRED to supervise.

References to be None provided durinr;, exam:

Lesson Plan 2621.830.0.0017, Conduct of Operations - Admin Learning 2.1.37, Knowledge of procedures, guidelines, or Objective/ limitations associated with reactivity management.

Question Source (New, Modified, Bank) I Modified If Bank or Modified:

VISION System/Question 10 667775 Question Source ILT 08-1 SRO Audit Exam Memory or X Comprehension Fundamental Cognitive 1:P or Analysis Knowledge Level NUREG 1021 Appendix B: Procedure steps and cautions 55.41 I I 55.43 I 6 10CRF55 Procedures and limitations involved in initial core Content loading, alterations in core configuration, control rod programming, and determination of various internal and external effects on core reactivity.

ILT 10-1 NRC & AUDIT EXAM Page: 75 of 81 26 May 2011

EXAMINATION ANSWER KEY ILT 10-1 NRC SRO Exam Justification for LORT questions with N/A KIA values < 3.0 Time to Complete: 1-2 minutes I Point Value: 1 System ID No.: N/A PRA: I NO Safety [gI Initial License Level N/A Function: o LORT ILT 10-1 NRC & AUDIT EXAM Page: 76 of 81 26 May 2011

EXAMINATION ANSWER KEY ILT 10-1 NRC SRO Exam 24 ID: 10-1 NSR024 Points: 1.00 Given the following:

  • A Site Area Emergency has been declared at Oyster Creek
  • The Technical Support Center (TSC) and Emergency Operations Facility (EOF) are activated with command and control functions transferred accordingly A worker is required to enter the Reactor Building under emergency conditions to close a manual valve to terminate a radioactive release. Details of this entry are as follows:
  • The worker's current annual exposure is 150 mRem
  • The general area radiation levels at the valve is 25 Rem/hr
  • It will take 20 min for the worker to close the manual valve
  • NEGLECT any dose the worker will receive transiting to and from the valve According tel EP-AA-113 "Personnel Protective Actions", who must authorize the emergency exposure the worker is expected to receive?

A. The Oyster Creek Site Vice President B. The Shift Manager in the Control Room C. The Station Emergency Director in the TSC D. The Corporate Emergency Director in the EOF Answer: C Answer Explanation I QID: 10-1 NSR024 Question # I 24 I Developer I Date: JJR I 7-11-11 Knowledge and Ability Reference Information Importance Rating K&A RO SRO 2.3.4

  • Knowledge of radiation exposure limits 3.2 3.7 under normal or emergency conditions.

Level I SRO I Tier 3 I Category I RPT General EP*AA*113 RP-AA-203 References ILT 10-1 NRC & AUDIT EXAM Page: 77 of 81 26 May 2011

EXAMINATION ANSWER KEY ILT 10-1 NRC SRO Exam C is Correct. Per EP-AA-1007 (among others),

emergency exposure controls are non-delegable responsibilities that remain with the Station Emergency Director. Since the TSC is activated, the Shift Manager (Shift Emergency Director) has transferred this responsibility to the Station Emergency Director. Per EP-AA-113, the Station Emergency Director (TSC) authorizes emergency exposures greater than 5 Rem TEDE. The dose the worker will receive is 8.3 Rem (25R1hr x 20min = 8.3 R). The applicant must recognize this above the limit the Site Vice President is authorized to approve and the Emergency Director with current command and control must authorize this exposure.

Explanation A is Incorrect but plausible since the Site Vice President approves all exposure up to the Federal Limit.

B is Incorrect but plausible since the Shift Manager is the person who authorizes emergency exposure when the Control Room has ERO command and control.

D is Incorrect. This distractor is plausible if the applicant does not recall that emergency exposure is a non-delegable responsibility and will be authorized by either the Station ED or Shift Manager depending who has command and control on site.

References to be None provided durin ~ exam:

Lesson Plan 2621.830.0.0015, Radiation Control- Admin Learning 2.3.4, Knowledge of radiation exposure limits under Objective/ normal or emergency conditions.

Question Source (New, Modified, Bank) I Modified If Bank or Modified:

N/A VISION System/Question ID Peach Bottom 2009 SRO NRC Question Source Exam ILT 10-1 NRC & AUDIT EXAM Page: 78 of 81 26 May2011

EXAMINATION ANSWER KEY ILT 10-1 NRC SRO Exam Memory or X Comprehension Fundamental Cognitive 1:P or Analysis Knowledge Level NUREG 1021 Appendix B: f.rocedure steps and cautions 55.41 I I 55.43 J 4 10CRF55 Radiation hazards that may arise during normal and Content abnormal situations, including maintenance activities and various contamination conditions.

Justification for LORT questions with N/A KIA values < 3.0 Time to Complete: 1*2 minutes I Point Value: 1 System 10 No.: N/A PRA: I NO Safety [gI Initial License Level N/A Function: o LORT ILT 1()"'1 NRC & AUDIT EXAM Page: 79 of 81 26 May 2011

EXAMINATION ANSWER KEY ILT 10-1 NRC SRO Exam 25 ID: 10*1 NSR025 Points: 1.00 Which one of the following activities requires a Temporary Configuration Change (TCC) per CC-AA-112, Temporary Configuration Changes?

A. Installation and removal of a jumper in accordance with an approved surveillance test procedure.

B. Changing a Control Room alarm setpoint that is NOT in direct support of a Maintenance Work Order.

C. Installation and removal of Measurement and Test Equipment (M&TE) in accordance with an approved surveillance test procedure.

D. A temporary configuration change included with an Operations Clearance that does NOT affect the system beyond the clearance boundary.

Answer: B Answer Explanation I QID: 10-1 NSR025 Question # I 25 I Developer 1 Date: JJR 17-11-11 Knowledge and Ability Reference Information Importance Rating K&A RO SRO 2.2.11 - Knowledge of the process for 2.3 3.3 controlling temporary design changes.

Level I SRO I Tier 3 I Category I EQC General CC-AA-112 References B is Correct. lAW the reference, temporary setpoint changes (ie. alarm setpoint changes) is not an excluded activity and therefore requires a Temporary Configuration Change (TCC), unless it is part of an Explanation approved Work Order or approved Clearance All distractors are Incorrect but plausible if the applicant does not recall activities that require TeCs per the procedure.

References to be None provided during exam:

ILT 10-1 NRC & AUDIT EXAM Page: 80 of 81 26 May 2011

EXAMINATION ANSWER KEY ILT 10-1 NRC SRO Exam Lesson Plan 2621.830.0.0018, Equipment Control* Admin Learning 2.2.11, Knowledge of the process for controlling Objective/ temporary design changes.

Question Source (New, Modified, Bank) I New If Bank or Modified:

N/A VISION System/Question 10 Question Source Memory or X Comprehension Fundamental Cognitive 1:P or Analysis Knowledge Level NUREG 1021 Appendix B: Procedure steps and cautions 55.41 I I 55.43 I 3 10CRF55 Facility licensee procedures required to obtain Content authority for design and operating changes in the facility.

Justification for LORT questions with N/A KIA values < 3.0 Time to Complete: 1*2 minutes I Point Value: 1 System 10 No.: N/A PRA: I NO Safety I2$j Initial License Level N/A Function: o LORT ILT 10-1 NRC & AUDIT EXAM Page: 81 of 81 26 May 2011